Sie sind auf Seite 1von 107

BMS

SPECIAL BOOKLET
(Inclusive of questions from previous years CAT papers and series of preparatory questions)

TABLE OF CONTENT
UNIT 1: QUANTITATIVE APTITUDE ____________________________ 03
Number System, Simplification, Percentage ___________________________04
Profit and Loss, Ratio & Proportion ___________________________________05
Averages __________________________________________________________06
Simple Interest, Compound Interest, Time, Speed & Distance ___________07
Time and Work ____________________________________________________08
Clocks and Calendars, Linear Equations, Quadratic Equations ___________09
Progression, Probability, Permutation & Combination __________________10
Geometry _________________________________________________________11
Mensuration _______________________________________________________12
Heights and Distance, Coordinate Geometry, Function _________________13
Set Theory, Trigonometry Ratio _____________________________________14
Differentiation, Integration, Application of Derivative __________________15
Differential Equation, Vectors _______________________________________16
3D, Logarithms, Data Interpretation _________________________________17

UNIT 2: LOGICAL REASONING _________________________________ 21


Coding & Decoding, Family Tree _____________________________________22
Arrangements and Puzzles __________________________________________23
Groups & Conditionalities ___________________________________________24
Directions ________________________________________________________26
Venn Diagram _____________________________________________________27
Syllogism _________________________________________________________28
Ranking & Series __________________________________________________29
Data Sufficiency ___________________________________________________30
Statement and Conclusion __________________________________________31
Visual Reasoning ___________________________________________________32
Cubes & Dices _____________________________________________________33
Input Output ____________________________________________________34
Statement & Assumptions __________________________________________35
Statement & Course of Action ______________________________________36
Statement & Argument, Assertion & Reason __________________________37
Inference from Passage _____________________________________________38

UNIT 3: VERBAL ABILITY ____________________________________ 40


Synonyms, Antonyms _______________________________________________41
Analogies, Idioms __________________________________________________42
Sentence Completion _______________________________________________43
Sentence Improvement _____________________________________________44
Sentence Correction________________________________________________45
Jumbled Sentences _________________________________________________48
Reading Comprehension ____________________________________________49

ANSWERS & EXPLANATIONS __________________________________ 67


PRATHAM: BMS Special Booklet

1
QUANTITATIVE APTITUDE

NUMBER SYSTEM:
1. Suppose you have a currency, named Miso, in three denominations: 1 Miso, 10 Misos and 50 Misos. In how many
ways can you pay a bill of 107 Misos?
(CAT 2007)
(a) 16
(b) 18
(c) 15
(d) 19
2. The sum of four consecutive two-digit odd numbers, when divided by 10, becomes a perfect square. Which of the
following can possibly be one of these four numbers?
(CAT 2006)
(a) 21
(b) 25
(c) 41
(d) 67
3. If a, a + 2 and a + 4 are prime numbers, then the number of possible solutions for a is(CAT 2003)
(a) One
(b) Two
(c) Three
(d) More than three
4. The owners of a local jewellery store hired 3 watchmen to guard his diamonds, but a thief still got in and stole some
1
diamonds. On the way out, the thief met each watchman, one at a time. To each he gave 2 of the diamonds he had
then and 2 more besides. He escaped with one diamond. How many diamonds did he steal originally? (CAT 2002)
(a) 40
(b) 36
(c) 34
(d) 45
5. ABC is a three-digit number in which A > 0. The value of ABC is equal to the sum of the factorials of its three digits.
What is the value of B?
(CAT 1997)
(a) 9
(b) 7
(c) 4
(d) 2

SIMPLIFICATION:
1

6. The value of 1 + +1
1
+5

1+

1
+2

1+
1
+5

1
+3

(a) 1 +
(b)
4
3
7. If r = -2, then r + 2r + 3r2 + r = ?
(a) 8
(b) 4

8. Let

= and (x y) =
+

(+ )

(+)

1+

1
+4

is
(c) x +

1
+5

(c) 0
, then x is equal to

(d)

+5
+1

(d) 10

()

(a)
(b)
(c)
(d) None of these
9. Assuming that in Harappan era, rocks, stones and pebbles were used for money. The following used to be the
currency valuations: 1 rock = 7 stones, 1 stone = 7 pebbles. If a person used 6 rocks to purchase a cave that costs 5
rocks, 2 stones and 3 pebbles, then how much would the change be?
(a) 4 stones, 4 pebbles
(b) 5 stones, 4 pebbles
(c) 1 rock, 5 stones, 4 pebbles
(d) 5 stones, 5 pebbles
10. The term 2 +

1
2

approximately equal to r +

1/2

85
?
(a) 9.06

(b) 9.34

.
2

Which of the following is the closest approximation to


(c) 9.22

(d) 9.28

PERCENTAGE:
Directions for Questions 11 and 12: In an examination, there are 100 questions divided into three groups A, B and C
such that each group contains at least one question. Each question in group A carries 1 mark, each question in group B
carries 2 marks and each question in group C carries 3 marks. It is known that the questions in group A together carry at
least 60% of the total marks.
(CAT 2004)
11. If group B contains 23 questions, then how many questions are there in group C?
(a) 1
(b) 2
(c) 3
(d) Cannot be determined
12. If group C contains 8 questions and group B carries at least 20% of the total marks, which of the following best
describes the number of questions in group B?
(a) 11 or 12
(b) 12 or 13
(c) 13 or 14
(d) 14 or 15

PRATHAM: BMS Special Booklet

QUANTITATIVE APTITUDE
13. At the end of year 1998, Shepherd bought nine dozen goats, henceforth, every year he added p% of the goats at the
beginning of the year and sold q% of the goats at the end of the year where p>0 and q>0. If Shepherd had nine
dozen goats at the end of year 2002, after making the sales for that year, which of the following is true? ( CAT 2003)
(a) p = q
(b) p < q
(c) p > q
(d) p = q/2
14. One bacteria splits into eight bacterias of the next generation. But due to environment, only 50% of one generation
can produce the next generation. If the seventh generation number is 4096 million, what is the number in first
generation?
(CAT 1998)
(a) 1 million
(b) 2 million
(c) 4 million
(d) 8 million
15. The number of votes not cast for the Praja Party increased by 25% in the National General Election over those not
cast for it in the previous Assembly Polls and the Praja Party lost by a majority twice as large as that by which it had
won the Assembly Polls. If a total 2,60,000 people voted each time, how many voted for the Praja Party in the
previous Assembly Polls?
(CAT 1994)
(a) 1,10,000
(b) 1,50,000
(c) 1,40,000
(d) 1,20,000

PROFIT AND LOSS:


Directions for Questions 16 and 17: Answer the questions based on the following information.
(CAT 1999)
The following table presents the sweetness of different items relative to sucrose, whose sweetness is taken to be 1.00
Lactose
Maltose
Glucose
Sucrose
Fructose
Saccharin

0.16
0.32
0.74
1.00
1.70
675.00

16. What is the minimum amount of sucrose (to the nearest gram) that must be added to one gram of saccharin to make
a mixture that will be at least 100 times as sweet as glucose?
(a) 7
(b) 8
(c) 9
(d) 100
17. Approximately how many times sweeter than sucrose is a mixture consisting of glucose, sucrose and fructose in the
ratio of 1 : 2 : 3?
(a) 1.3
(b) 1.0
(c) 0.6
(d) 2.3
18. Using only 2, 5, 10, 25 and 50 paise coins, what will be the minimum number of coins required to pay exactly 78
paise, 69 paise and Re 1.01 to three different persons?
(CAT 2003)
(a) 19
(b) 20
(c) 17
(d) 18
19. Instead of walking along two adjacent sides of a rectangular field, a boy took a short cut along the diagonal and saved
a distance equal to half the longer side. Then, the ratio of the shorter side to the longer side is
(CAT 2002)
1
2
1
3
(a)
(b)
(c)
(d)
2
3
4
4
20. Mayank, Mirza, Little and Jaspal bought a motorbike for $60,000. Mayank paid one half of the sum of the amounts
paid by the other boys, Mirza paid one third of the sum of the amounts paid by the other boys; and Little paid one
fourth of the sum of the amounts paid by the other boys. How much Jaspal had to pay?
(CAT 2002)
(a) $15,000
(b) $13,000
(c) $17,000
(d) None of these

RATIO & PROPORTION:


21. Which is the larger among two numbers if they are in the ratio of 6 :13 and their least common multiple is 312 ?
(a) 52
(b) 26
(c) 24
(d) 12

PRATHAM: BMS Special Booklet

22. The ratio between the number of passengers travelling by I and II class between the two railway stations is 1: 50,
whereas the ratio of I and II class fares between the same stations is 3 : 1. If on a particular day, Rs. 1325 were
collected from the passengers travelling between these stations, then what was the amount collected from the II class
passengers ?
(a) Rs. 750
(b) Rs. 850
(c) Rs. 1000
(d) Rs. 1250
1
1
23. It is given that y 3 .For x = 2, value of y is .If x = 1, then the value of y will be :

6
(a) 1
(b) 0
(c) -1
(d) None of these
24. Suppose y varies as the sum of two quantities of which one varies directly as x and the other varies inversely as x. If y
1
= 6 when x = 4 and y = 3 when x = 3, then the relation between x and y is :
4

(a) y = x + x
(b) y = - 2x +
(c) y = 2x + x
(d) y = 2x -
25. Which of the following is the ratio between a number and the number obtained by adding one-fifth of that number to
it ?
(a) 6 : 5
(b) 5 : 6
(c) 5 : 4
(d) 4 : 5

AVERAGES:
26. A shipping clerk has five boxes of different but unknown weights each weighing less than 100 kg. The clerk weighs
the boxes in pairs. The weights obtained are 110, 112, 113, 114, 115, 116, 117, 118, 120 and 121 kg. What is the
weight of the heaviest box?
(CAT 2000)
(a) 60 kg
(b) 62 kg
(c) 64 kg
(d) Cannot be determined
27. Consider the set S = {2, 3, 4, , 2n + 1}, where n is a positive integer larger than 2007. Define X as the average of
the odd integers in S and Y as the average of the even integers in S. What is the value of X Y?
(CAT 2007)
1
n+1
(a) 1
(b) 2 n
(c) 2n
(d) (n + 1)
Directions for Questions 28 30: Answer the questions based on the following information.
(CAT 1997)
There are 60 students in a class. These students are divided into three groups A, B and C of 15, 20 and 25 students each.
The groups A and C are combined to form group D.
28. What is the average weight of the students in group D?
(a) More than the average weight of A
(b) More than the average weight of C
(c) Less than the average weight of C
(d) Cannot be determined
29. If one student from group A is shifted to group B, which of the following will be true?
(a) The average weight of both the groups increases
(b) The average weight of both the groups decreases
(c) The average weight of the class remains the same
(d) Cannot be determined
30. If all the students of the class have the same weight, then which of the following is false?
(a) The average weight of all the four groups is the same
(b) The total weight of A and C is twice the total weight of B
(c) The average weight of D is greater than the average weight of A
(d) The average weight of all the groups remains the same even if a number of students are shifted from one group
to another.

PRATHAM: BMS Special Booklet

QUANTITATIVE APTITUDE

SIMPLE INTEREST:
31. A person closes his account in an investment scheme by withdrawing Rs 10000. One year ago, he had withdrawn
Rs 6000. Two years ago he had withdrawn Rs 5000. Three years ago he had not withdrawn any money. How much
money had he deposited approximately at the time of opening the account 4 years ago, if the annual simple interest is
10 %?
(a) Rs 15600
(b) Rs 16500
(c) Rs 17280
(d) None of these
32. Vikram borrowed Rs 6450 at 5 percent simple interest repayable in 4 equal installments. What will be the annual
instalment payable by him ?
(a) Rs 1710
(b) Rs 1810
(c) Rs 1910
(d) Rs 1860
33. Subbarao was approached by two neighbours for loan. He had Rs 2540, a part of which he lent to one person at
12% interest per annum, and the other part was lent to the second person at 12.5 %. At the end of a year, Subbarao
received Rs 311.60 as interest on the total loan. Calculate the amount of money lent by him at 12% interest.
(a) Rs 1360
(b) Rs 1340
(c) Rs 1240
(d) Rs 1180
34. A sum of money doubles itself in 12 years if invested at simple interest. What is the rate of interest allowed on the
investment ?
(a) 9.5%
(b) 8.25%
(c) 8.5%
(d) 8.33%
35. Ram purchased a flat at Rs. 1 lakh and Prem purchased a plot of land worth Rs. 1.1 lakh. The respective annual rates
at which the prices of the flat and the plot increased were 10% and 5%. After two years, they exchanged their
belongings and one paid the other the difference. Then:
(CAT 1995)
(a) Ram paid Rs. 275 to Prem
(b) Ram paid Rs. 476 to Prem
(c) Ram paid Rs. 375 to Prem
(d) Prem paid Rs. 475 to ram

COMPOUND INTEREST:
36. A bank offers 10 % interest rate compounded annually. A person deposits Rs. 10,000 every year in his account. If he
does not withdraw any amount, then how much balance will his account show after four years ?
(a) Rs. 51051
(b) Rs. 45095
(c) Rs. 36410
(d) Rs. 51000
37. A sum is invested for 3 years compounded at 5%, 10% and 20 % respectively. In three years if the sum amounts to
Rs. 16,632, then find the sum.
(a) Rs. 11000
(b) Rs. 12000
(c) Rs. 13000
(d) Rs. 14000
38. The value of a fixed asset depreciates at the rate of 10 % on the value at the beginning of each year. If the value of
the asset two years ago, was Rs 12,000 more than the value of the asset one year ago, then find the present value of
the asset, given that the asset was bought two years ago.
(a) Rs. 14520
(b) Rs. 96,000
(c) Rs. 97,200
(d) Rs. 17,520
39. Anuj has deposited certain amount in the bank to earn compound interest at 10 % p.a. The difference in the interest
on the amount between the 3rd and 2nd years is Rs. 1,100. What amount has Anuj deposited?
(a) Rs. 10000
(b) Rs. 11000
(c) Data inadequate
(d) None of these
40. Divide Rs. 1301 between A and B, so that the amount of A after 7 years is equal to the amount of B after 9 years, the
interest being compounded at 4% per annum.
(a) Rs. 676 and Rs. 625
(b) Rs. 650 and Rs. 651
(c) Rs. 670 and Rs. 631 (d) Rs. 660 and Rs. 630

TIME, SPEED & DISTANCE:


Directions for Questions 41 42: Answer the questions based on the following information.
(CAT 2001)
The petrol consumption rate of a new model car Palto depends on its speed and may be described by the adjoining
graph:

PRATHAM: BMS Special Booklet

41. Manasa makes the 200 km trip from Mumbai to Pune at a steady speed of 60 km/h. What is the amount of petrol
consumed for the journey?
(a) 12.5 L
(b) 13.33 L
(c) 16 L
(d) 19.75 L
42. Manasa would like to minimize the fuel consumption for the trip by driving at the appropriate speed. How should she
change the speed?
(a) Increase the speed
(b) Decrease the speed
(c) Maintain the speed at 60 km/h
(d) Cannot be determined
43. Three runners A, B and C run a race, with runner A finishing 12 m ahead of runner B and 18 m ahead of runner C,
while runner B finishes 8 m ahead of runner C. Each runner travels the entire distance at a constant speed. What was
the length of the race?
(CAT 2001)
(a) 36 m
(b) 48 m
(c) 60 m
(d) 72 m
3
44. A train approaches a tunnel AB. Inside the tunnel a cat is located at a point that is 8 of the distance AB measured from
the entrance A. When the train whistles, the cat runs. If the cat moves to the entrance of the tunnel A, the train
catches the cat exactly at the entrance. If cat moves to the exit B, the train catches the cat at exactly the exit. The
speed of the train is greater than the speed of the cat by what order?
(CAT 2002)
(a) 3 : 1
(b) 4 : 1
(c) 5 : 1
(d) None of these
45. At a bookstore, MODERN BOOK STORE is flashed using neon lights. The words are individually flashed at
1
1
1
intervals of 2 , 4 , 5 seconds respectively, and each word is put off after a second. The least time after which the
2
4
8
full name of the bookstore can be read again, is(CAT 2002)
(a) 49.5 s
(b) 73.5 s
(c) 1744.5 s
(d) 855 s

TIME AND WORK:


46. Three small pumps and a large pump are filling a tank. Each small pump works at (2/3)rd the rate of the large pump. If
all four work at the same time, they should fill the tank in what fraction of the time it would have taken the large
pump alone?
(a) 4/7
(b) 1/3
(c) 2/3
(d) 3/4
47. Six technicians working at the same rate complete work of one server in 10 h. If they start at 11:00 am and one
additional technician per hour is being added beginning at 5:00 pm, at what time the server will be complete?
(CAT 2002)
(a) 6:40 pm
(b) 7 pm
(c) 7:20 pm
(d) 8:00 pm
48. A can complete a piece of work in 4 days. B takes double the time taken by A, C takes double that of B, and D takes
double that of C to complete the same task. They are paired in groups of two each. One pair takes two-third the
time needed by the second pair to complete the work. Which is the first pair?
(CAT 2001)
(a) A, B
(b) A, C
(c) B, C
(d) A, D
49. Shyama and Vyom walk up an escalator (moving stairway). The escalator moves at a constant speed. Shyama takes
three steps for every two of Vyoms steps. Shyama gets to the top of the escalator after having taken 25 steps, while
Vyom (because his slower pace lets the escalator do a little more of the work) takes only 20 steps to reach the top. If
the escalator was turned off, how many steps would they have to take to walk up?
(CAT 2001)
(a) 40
(b) 50
(c) 60
(d) 80

PRATHAM: BMS Special Booklet

QUANTITATIVE APTITUDE
50. At his usual rowing rate, Rahul can travel 12 miles downstream in a certain river in six hours less than it takes him to
travel the same distance upstream. But if he could double his usual rowing rate for this 24 mile round trip, the
downstream 12 miles would then take only one hour less than the upstream 12 miles. What is the speed of the
current in miles per hour?
(CAT 2001)
(a) 7/3
(b) 4/3
(c) 5/3
(d) 8/3

CLOCKS AND CALENDARS:


51. At what time between 5 and 6 O'clock are the hands of a clock 3 minutes apart ?
(a) 24 minutes past 5
(b) 22 minutes past 3
(c) 26 minutes past 4 (d) None of these
52. My watch which gains 5 seconds in 3 minutes was set right at 7 a.m. In the afternoon of the same day, when the
watch indicated quarter past 4 O 'clock, the true time is:
(a) 59 min. past 3
(b) 12 min. past 3
(c) 4 p.m.
(d) 7 min. past 4
53. At what time between 4 : 30 and 5 will the hands of a clock be in a straight line ?
(a) 50 mins. past 4
(b) 42 mins. past 4
(c) 54 mins. past 4
(d) 46 mins. past 4
54. 7 January 1992 was Tuesday. Find the day of the week on the same date after 5 years, i.e. on 7 January 1997 ?
(a) Tuesday
(b) Wednesday
(c) Saturday
(d) Friday
55. The first Republic Day of India was celebrated on 26 January 1950. What was the day of the week on that date ?
(a) Monday
(b) Wednesday
(c) Saturday
(d) Thursday

LINEAR EQUATIONS:
56. A man purchased 47 stamps of 20-paise and 15 paise. The total amount he spent was Rs. 8.80. The number of
20-paise and 15- paise stamps purchased by him (a) was 12 and 35 respectively
(b) was 10 and 37 respectively
(c) was 35 and 12 respectively
(d) None of these
57. For the given pair (x, y) of positive integers, such that 4x - I7y = 1 and x 1,000, how many integer values of y satisfy
the given conditions :
(CAT 1999)
(a) 55
(b) 56
(c) 57
(d) 58
58. If x2 + 5y2 + z2 = 2y (2x + z) then which of the following statements are necessarily true?
(CAT 2002)
A. x = 2y
B. x = 2z
C. 2x = z
(a) Only A
(b) Only B and C
(c) Only A and B
(d) None of these
59. If n is such that 36 n 72, then x =

2 + 2 +4 + 16
+4 + 4

(CAT 2003)

satisfies-

(a) 20 < x < 54


(b) 23 < x < 58
(c) 25 < x < 64
60. The total number of integer pairs (x, y) satisfying the equation x + y = xy is :
(a) 0
(b) 1
(c) 2

(d) 28 < x < 60

(CAT 2004)

(d) None of these

QUADRATIC EQUATIONS:
61. Which of the following expressions cannot be equal to zero, when x2 - 2x = 3 ?
(a) x2 - 7 x + 6
(b) x2 - 9
(c) x2 - 4x + 3
(d) x2 -6x + 9
62. If 2 + i 3 is a root of the equation x2 + px + q =0, where p and q are real, then (p, q) is
(a) (2, 3)
(b) (-2, 3)
(c) (4, 7)
(d) (- 4, 7)
63. The number of quadratic equations which are unchanged by squaring their roots is :
(a) 2
(b) 4
(c) 5
(d) 6

PRATHAM: BMS Special Booklet

64. One of the roots of the equation x2 - x + 3m = 0 is double of one of the roots of the equation x2 - x + m = 0.
If m 0, then find its value,
(a) 1
(b) -1
(c) 2
(d) -2
65. Let p and q be the roots of the quadratic equation x2 - ( - 2) x - - 1 = 0. What is the minimum possible value of p2
+ q2 ?
(CAT 2003)
(a) 0
(b) 3
(c) 4
(d) 5

PROGRESSION:
66. If logxa, ax/2 and logb x are in GP, then x is
(a) loga(logb a)
(b) loga(loge a) + loga (loge b)
(c) -loga(loga b) (d) loga (loge b) - loga(loge a)
2
67. f(a, b) is a series, of which the first three terms are (a + b) , (a2 + b2) and (a - b)2. We add the first n terms of the
series f(a, b) and call it S(a, b). If a = 7, b = 3, then find S(7, 3) for n = 20.
(a) 5980
(b) 6000
(c) 6960
(d) None of these
68. How many three digit numbers are divisible by 6 ?
(a) 102
(b) 150
(c) 151
(d) 966
69. Three distinct numbers x, y, z form a geometric progression in that order and x + y, y + z, z + x form an arithmetic
progression in that order. Find the common ratio of the geometric progression.
(a) -2
(b) 2
(c) 0.5
(d) -0.5
70. Find the sum of all natural numbers not exceeding 1000, which are divisible by 4 but not by 8.
(a) 62500
(b) 62800
(c) 64000
(d) 65600

PROBABILITY:
71. If the probability of rain on any given day in Pune city is 50 %, then what is the probability that it rains on exactly 3
days in a 5 - day period ?
8
5
8
2
(a)
(b)
(c)
(d)
125
16
25
25
72. The probability that an event happens in one trial of an experiment is 0.4. Three independent trials of the experiment
are performed. The probability that the event happens at least once is(a) 0.934
(b) 0.784
(c) 0.548
(d) 0.343
73. What is the possibility of getting at least 6 heads if eight coins are tossed simultaneously ?
1
25
1
(a) 15
(b) 57
(c) 13
(d) None of these
74. In a bag containing three balls, a white ball was placed, and then one ball was taken out at random. What is the
probability that the extracted ball would turn out to be white, if all possible hypothesis concerning the colour of the
balls that were initially in the bag were equally possible?
5
3
1
3
(a)
(b)
(c)
(d)
8
4
2
8
75. From a box containing 60 standard and 40 substandard articles, two articles are chosen at random. What is the
probability that one of them is standard and the other is substandard?
60
40
60
39
16
(a) 100 100
(b) 100 100
(c) 33
(d) 24%

PERMUTATION AND COMBINATION:


76. Boxes numbered 1, 2, 3, 4 and 5 are kept in a row and they are to be filled with either a red or a blue ball, such that
no two adjacent boxes can be filled with blue balls. Then how many different arrangements are possible, given that all
balls of a given colour are exactly identical in all respects?
(CAT 1995)
(a) 8
(b) 10
(c) 15
(d) 22

PRATHAM: BMS Special Booklet

10

QUANTITATIVE APTITUDE
77. In how many ways is it possible to choose a white square and a black square on a chess-board so that the squares
must not lie in the same row or column?
(CAT 2002)
(a) 56
(b) 896
(c) 60
(d) 768
78. A 4 digit number is formed with digits 1, 2, 3 and 5. What is the probability that the number is divisible by 25, if
repetition of digits is not allowed?
(CAT 1995)
(a) 1/12
(b) 1/4
(c) 1/6
(d) None of these
79. In how many ways can eight Directors, Vice-chairman and Chairman of a firm be seated at a round table, if the
chairman has to sit between the Vice-chairman and a Director?
(CAT 1997)
(a) 9! 2
(b) 2 8!
(c) 2 7!
(d) None of these
80. For a scholarship, at the most n candidates out of 2n + 1 can be selected. If the number of different ways of selection
of at least one candidate is 63, the maximum number of candidates that can be selected for the scholarship is:
(CAT 1999)
(a) 3
(b) 4
(c) 6
(d) 5

GEOMETRY:
81. In the given diagram, ABCD is a rectangle with AE = EF =
FB. What is the ratio of the area of the triangle CEF and that
of the rectangle?
(CAT 2000)
(a) 1 : 4
(c) 2 : 5

(b) 1 : 6
(d) 2 : 3

82. In the given figure, AB is diameter of the circle and points C


and D are on the circumference such that CAD = 30o and
CBA = 70o. What is the measure of ACD? (CAT 1995)
(a) 40o
(c) 30o

(b) 50o
(d) 90o

83. The length of the common chord of two circles of radii 15 cm and 20 cm respectively whose centers are 25 cm apart,
is (in cm):
(CAT 2002)
(a) 24
(b) 25
(c) 15
(d) 20
84. A circle with radius 2 is placed against a right angle. Another
smaller circle is also placed as shown in the adjoining figure.
What is the radius of the smaller circle?
(CAT 2004)
(a) 3 - 2 2
(c) 7 4 2

(b) 4 - 2 2
(d) 6 4 2

Directions for Question 85: Answer the question based on


the following information.
(CAT 2004)
In the adjoining figure, I and II are circles with centers P and Q
respectively. The two circles touch each other and have a
common tangent that touches them at points R and S

PRATHAM: BMS Special Booklet

S
O

11

respectively. This common tangent meets the line joining P and


Q at O. The diameters of I and II are in the ratio 4 : 3. It is also
known that the length of PO is 28 cm.
85. What is the ratio of the length of PQ to that of QO?
(a) 1 : 4
(b) 1 : 3

(c) 3 : 8

(d) 3 : 4

MENSURATION:
86. In the adjoining figure, AC + AB = 5 AD and AC AD = 8.
Then, the area of the rectangle ABCD is
(CAT 1995)
(a) 36
(c) 60

(b) 50
(d) Cannot be answered

87. PQRS is a square. SR is a tangent (at point S) to the circle


with centre O and TR = OS. Then, the ratio of area of the
circle to area of the square is
(CAT 1995)
(a) / 3
(c) 3 /

(b) 11/7
(d) 7/11

88. The adjoining figures shows a set of concentric squares. If


the diagonal of the innermost square is 2 unit and if the
distance between the corresponding corners of any two
successive squares is 1 unit, find the difference between the
areas of the eighth and the seventh squares, counting from
the innermost square.
(CAT 1997)
(a) 10 2 sq unit
(b) 30 sq unit
(c) 35 2 sq unit
(d) None of these
89. AB is the diameter of the given circle, while points C and D
lie on the circumference as shown. If AB is 15 cm, AC is 12
cm and BD is 9 cm, find the area of quadrilateral ABCD.
(CAT 1997)
(a) 54p cm2
(c) 162p cm2

(b) 216p cm2


(d) None of these

90. Consider a right circular cone of base radius 4 cm and height 10 cm. A cylinder is to be placed inside the cone with
one of the flat surfaces resting on the base of the cone. Find the largest possible total surface area (in sq cm) of the
cylinder.
(CAT 2008)
100
80
120
130
(a)
(b)
(c)
(d)
3

PRATHAM: BMS Special Booklet

12

QUANTITATIVE APTITUDE
HEIGHTS AND DISTANCE:
91. A ladder is inclined to a wall making an angle of 30 with it. A man is ascending the ladder at the rate of 2
metres/second. How fast is he approaching the wall ?
(a) 2 m/s
(b) 1.5 m/s
(c) 1 m/s
(d) None of these
92. Mr. Gidwant's Padyatra Party wanted to go from Gwalior to Bhubaneswar. The walkers travelled 150 km straight and
then took a 45 turn towards Varanasi and walked straight for another 120 km. Approximately, how far the party is
from the starting point ?
(a) 250 km
(b) 90 km
(c) 81km
(d) 30 km
93. A vertical pole PO is standing at the centre O of a square ABCD. If AC subtends an angle of 90 at the point P of the
pole, then the angle subtended by a side of the square at the point P is (a) 35
(b) 45
(c) 30
(d) 60
94. What is the height of tower if the angles of elevation of its top from two points X and Y at distances of a and b
respectively from the base and on the same straight line with the tower, are complementary ?
b

(a) a
(b) b
(c) ab
(d) None of these
95. A window on one side of a road is 12 m above ground. A ladder is placed on the road to reach the window. If the
ladder is turned to the other side of the road, keeping its feet at the same point, it can reach a window which is at a
height of 9 m from the ground. Supposing the length of the ladder to be 15 m, what is the width of the road ?
(a) 9 m
(b) 12 m
(c) 21 m
(d) None of these

COORDINATE GEOMETRY:
8

96. The points 0, 3 , (1, 3) and (82, 30) are the vertices of

(a) An obtuse-angled triangle


(b) A right-angled triangle
(c) An isosceles triangle
(d) None of these
97. The x axis is shifted to a new position such that the points H,J,K assume new coordinates as follows:
Point
Original Coordinates
New Coordinates
H
(3, 5)
(3, 2)
J
(4, 2)
(4, -1)
K
(2, 6)
(2, 3)
If the original coordinates of P were (5, 1), then which of the following are the new coordinates of P?
(a) (2, -2)
(b) (5, 4)
(c) (5, -2)
(d) (8, 1)
98. The course of an enemy submarine as plotted on a set of rectangular axes gives the equation 2x + 3y = 5. On the
same axes, the course of destroyer is indicated by the equation x y = 10. The point (x, y) at which the submarine
can be destroyed is:
(a) (7, -3)
(b) (-3, 7)
(c) (-7, 3)
(d) (3, -7)
99. A line is of length 10 and one end is (2, -3). If the abscissa of the other end is 10 then its ordinate is
(a) 3 or 9
(b) -3 or 9
(c) 3 or 9
(d) -3 or 9
100. If the points (t, 2t), (-2, 6) and (3, 1) are collinear, then t =
3
4
(a) 4
(b) 3
(c) 3
(d) 4

FUNCTION:
101. A function f(x) satisfies f(1) = 3600 and f(1) + f(2) + + f(n) = n 2f(n), for all positive integers n > 1. What is the
value of f(9)?
(CAT 2007)
(a) 240
(b) 200
(c) 100
(d) 80

PRATHAM: BMS Special Booklet

13

102. If f(x) = log

1+x
1x

(CAT 2002)

, then f(x) + f(y) is

(a) f(x + y)

(b)

x +y
1+xy

(c) (x + y)

1
1+xy

(d)

f x + f(y)
1+xy

Direction for Questions 103 104: Answer the questions independent of each other.
(CAT 2000)
X
1
2
3
4
5
6
Y
4
8
14
22
32
44
103. Which of the following equation will be best for above data?
(a) y = ax + b
(b) y = a+ bx +cx2
(c) y = eax + b
(d) None of these
104. If f(0, y) = y + 1 and f(x + 1,y) = f(x, f (x, y)). Then, what is the value of f(1, 2)?
(a) 1
(b) 2
(c) 3
(d) 4
Direction for Question 105: Answer the questions based on the following information (CAT 1994)
x3
If f(x) = 2x + 3 and g(x) =
, then
2
105. fog(x) is equal to
15x+9
1
(a) 1
(b) gof (x)
(c) 16x5
(d) x

SET THEORY:
106. In a class of 80 children, 35 % children can play only cricket, 45% children can play only table-tennis and the
remaining children can play both the games. In all, how many children can play cricket ?
(a) 55
(b) 44
(c) 36
(d) 28
107. While preparing the progress reports of the students, the class teacher found that 70% of the students passed in
Hindi, 80% passed in English and only 65% passed in both the subjects. Find out the percentage of students who
failed in both the subjects?
(a) 15%
(b) 20%
(c) 30%
(d) 35%
108. In a party, 70 guests were to be served tea or coffee after dinner. There were 52 guests who preferred tea while 37
preferred coffee. Each of the guests liked one or the other beverage. How many guests liked both tea and coffee ?
(a) 15
(b) 18
(c) 19
(d) 33
109. In a certain group of 36 people, only 18 are wearing hats and only 24 are wearing sweater. If 6 are not wearing any of
the 2, then how many people are wearing both a hat and a sweater ?
(a) 30
(b) 22
(c) 12
(d) 8
110. Consider the set Tn: {n, n + 1, n + 2, n +3, n + 4}, where n = 1, 2, 3 , 96. How many of these sets contain 6
or any integral multiple thereof (i.e., any one of the number 6, 12, 18, ....) ?
(a) 80
(b) 81
(c) 82
(d) 83

TRIGONOMETRY RATIO:
111. If sin + cos = a and sec + cosec = b, then the value of b (a2 1) is equal to
(a) 2a
(b) 3a
(c) 0
(d) 2ab
7 sin 3 cos
112. If sin A : cos A = 4 : 7, then the value of
is
3

7 sin +2 cos

(a) 14
(b) 2
(c) 3
(d) 6
113. A, B and C are three angles such that tan A + tan B + tan C = tan A tan B tan C. Which of the following statements
is always correct?
(a) ABC is a triangle, i.e., A + B + C =
(b) A = B = C, i.e. ABC is an equilateral triangle
(c) A + B = C, i.e. ABC is a right angled triangle (d) None of these
2
114. If the sine of an angle= 3 , then cosine of that angle is
2

(a) Equal to 3

(b) Greater than 3

PRATHAM: BMS Special Booklet

(c) Less than 3

(d) Infinite

14

QUANTITATIVE APTITUDE
cos 60 0 +sin 60 0

115. Simplify: cos 60 0 sin 60 0


(a) 3 2

(b) 3 + 2

(c) ( 3 + 2)

(d) 1

DIFFERENTIATION:

116. If y =sin 2 , = , =
(a)

, the value of is equal to

sec +tan
?
sec tan

(b)

cos
( 2 )

(c)

(cos
( 2 )

(d) cos

117. What is the derivative of


(a) 2 sec x (sec x + tan x)
(b) 2 sec 2 (sec + tan )2
2
(c) 2 sec x (sec x + tan x)
(d) sec x (sec x + tan x)2
118. What is the differential coefficient of (log ), where () = log x?

(b) ( log )1

(a) (log )
cos
2
2 cos
(6+sin )2

119. If y = 3x
(a)

2
2

(c)

(log )

(d)

(d)

4 sin
6+ 3

is
4 cos

(b) (6+sin )2

4 cos
6+ 3
2

2 2 + +

(c)

120. If y = a cos (log x)- b sin (log x), then the value of
(a) 4
(b) 0

is

(c) 2

(d) 3

INTEGRATION::
121. What is the value of
(a)

1
sin1

dx
2 2

?
(b) sin1

(c) log + 2 2 +

(d)

log + 2 2 +

122. If (x) dx = ()/2, then which one of the following is correct?


(a) = 2 + constant
(b) = + constant
(c) () = constant
(d) = 2
123. If () dx = g(x) and also = , then which one of the following is correct?
(a) g(x) = h(x)
(b) g(x) + h(x) = constant
(c) g(x) h(x) = constant
(d) g(x) h(x) = constant
124. What is the value of the integral e

(a) +
125. What is the value of
1

(b) e

x2+ 1

x2+ 1

sin x
sin 2 sin 2

dx

+ C

x 2+ 1

dx?

(c) e

x 2+ 1

+ C

(d) + +

dx?

(a) sin (sec a cos x) + C


(c) sinh1 (sec a cos x) + C

(b) cos 1 (sec a cos x) + C


(d) cosh1 (sec a cos x) + C

APPLICATION OF DERIVATIVE:
126. The function is increasing, when
1
1
(a) x >
(b) x <

PRATHAM: BMS Special Booklet

(c) x < 0

(d) for all real x


15

127. How many tangents are parallel to x-axis for the curve y = 2 4 + 3 ?
(a) 1
(b) 2
(c) 3
(d) No tangent is parallel to x-axis
128. What is the maximum value of the function log x x?
(a) -1
(b) 0
(c) 1
(d)

129. The second derivative of ( ) with respect to x, where is a polynomial, is:


(a) +
(b) 2 +

(c)
(d) 2 +
130. Which one of the following statements is correct?
(a) The derivative of a function () at a point will exist, if there is one tangent to the curve y = () at that point
and the tangent is parallel to y-axis.
(b) The derivative of a function () at a point will exist, if there is one tangent to the curve y = () at that point
and the tangent must be parallel to x-axis.
(c) The derivative of a function ()at a point will exist, if there is one and only one tangent to the curve y = () at
that point and the tangent is not parallel to y-axis.
(d) None of the above.

DIFFERENTIAL EQUATION:
131. An integrating factor of the differential equation (1 x2)
(a) x

dy
dx

xy = 1 is:

x
(1x 2 )

(c) (1 x 2 )

(b) 2x 2y = C

(c) 2x 2y = C

(b) -

(d) 2 log
(1 x 2 )

132. The second order differential equation of a parabola with its principal axis along the x-axis is:
(a) y + 2(y)2 = 0
(b) 3yy + 2(y)3 = 0 (c) yy + (y)2 = 0
(d) y+ 2(y)4 = 0
dy
133. The solution of dx = 2yx is:
(a) 2x + 2y = C

dy

134. The integrating factor of the differential equation cos x


+ y sin x = 1 is:
dx
(a) sec x
(b) tan x
(c) sin x
d2y
dx 2

(d) 2x + 2y = C
(d) cos x

135. If
+ sin x = 0, then the solution of differential equation is:
(a) y = sin x + Cx + D
(b) y = cos x + Cx2 + D
(c) y = tan x + C
(d) y = log sin x + Cx

VECTORS:
136. If D, E, F are respectively the mid points of AB, AC and BC in ABC, then BE + AF is equal to:
1
3
(a) DC
(b)
(c) 2BF
(d)
2
2
137. If three points A, B and C have position vectors (1, x, 3), (3, 4, 7) and (y, -2, -5) respectively and if they are collinear,
then (x, y) is equal to:
(a) (2, -3)
(b) (-2, 3)
(c) (2, 3)
(d) (-2, -3)
138. If a = i + 2j 3k and b = 3i j + k and (a + b) is perpendicular to (a b), what is the value of ?
(a) -2
(b) 2
(c) 3
(d) 3
139. If vectors, a, b and c are mutually perpendicular vectors such that a = b = 10, c = 1, then the length of vector
2a + 2b + 40c is:
(a) 20
(b) 20 6
(c) 40 6
(d) None of these

140. If a and b are two unit vectors inclined at an angle 3, then the value of a + b is:
(a) greater than 1
(b) less than 1
(c) equal to 1
(d) equal to 0

PRATHAM: BMS Special Booklet

16

QUANTITATIVE APTITUDE
3D:
141. The intercepts of the plane 5x 3y + 6z = 60 on the coordinate axes are
(a) (10, 20, -10)
(b) (10, -20, 12)
(c) (12, -20, 10)
(d) (12, 20, -10)
142. The direction cosines of the line joining the points (4, 3, -5) and (-2, 1, -8) are
6 2 3
2 3 6
6 3 2
(a) 7 , 7 , 7
(b) 7 , 7 , 7
(c) 7 , 7 , 7
(d) None of these
143. The foot of the perpendicular drawn from the origin to a plane is the point (1, -3, 1). What is the intercept cut on the
x-axis by the plane?
(a) 1
(b) 3
(c) 11
(d) 11
144. A straight line with direction cosines (0, 1, 0) is
(a) Parallel to the x-axis
(b) Parallel to the y-axis
(c) Parallel to the z-axis
(d) Equally inclined to all the axes
145. ABC is a triangle and AD is the median. The coordinates of A are (4, 7, -8) and the coordinates of centroid of the
ABC are (1, 1, 1). What are the coordinates of D?
1
1
11
(a) 2 , 2, 11
(b) 2 , 2, 2
(c) (-1, 2, 11)
(d) (-5, -11, 19)

LOGARITHMS:
146. If log a () = , then log b () is:
1

(a)
(b)

(c)

+1

(d)

147. Simplify: 2 log10 25 2 log10 3 + log10 18


(a) 18
(b) 1
(c) log103
148. If log 7 log 5 ( + 5 + ) = 0, find the value of x.
(a) 1
(b) 0
(c) 2
149. log 2 [log 7 ( 2 + 37)] = 1. Then, what could be the value of x?
(a) 3
(b) 5
(c) 4
150. What is the sum of n terms in the series: log m + log
(a) log

1
+1

/2

(b) log

/2

(c) log

(d) None of these


(d) None of these

3
+ log 2
/2
1

+ log

4
3

(d) None of these

(CAT 1997)
(CAT 2003)

+ ..
(d) log

(CAT 1994)

1+
( 1)

/2

DATA INTERPRETATION:
Directions for Questions 151 - 155: Answer the questions based on the following information.
In a Class X Board examination, ten papers are distributed over five GroupsPCB, Mathematics, Social Science,
Vernacular and English. Each of the ten papers is evaluated out of 100. The final score of a student is calculated in the
following manner. First, the Group Scores are obtained by averaging marks in the papers within the Group. The final
score is the simple average of the Group Scores. The data for the top ten students are presented below. (Dipan's score in
English Paper II has been intentionally removed in the table.)
Name of the
student

Ayesha (G)
Ram (B)
Dipan (B)

PCB Group
Phy. Chem. Bio.

98
97
98

96
99
98

97
95
98

PRATHAM: BMS Special Booklet

Mathematics
Group

98
97
95

Social Science
Group
Hist.

Geo.

95
95
96

93
96
95

Vernacular
Group

English Group

Paper Paper
Paper I Paper II
I
II

94
94
96

96
94
94

96
96
96

98
98
?

Final
Score

96.2
96.1
96.0
17

Sagnik (B)
Sanjiv (B)
Shreya (G)
Joseph (B)
Agni (B)
Pritam (B)
Tirna (G)

97
95
96
90
96
98
96

98
96
89
94
99
98
98

99
97
85
98
96
95
97

96
98
100
100
99
98
99

96
97
97
94
95
83
85

98
96
98
97
96
95
94

94
92
94
90
82
90
92

97
93
95
92
93
93
91

92
95
96
94
92
94
87

94
96
95
95
93
94
96

95.9
95.7
95.5
95.0
94.3
93.9
93.7

Note: B or G against the name of a student respectively indicates whether the student is a boy or a girl.

151. How much did Dipan get in English Paper II ?


(a) 94
(b) 96.5
(c) 97
(d) 98
152. Students who obtained Group Scores of at least 95 in every group are eligible to apply for a prize. Among those who
are eligible, the student obtaining the highest Group Score in Social Science Group is awarded this prize. The prize
was awarded to
(a) Shreya
(b) Ram
(c) Ayesha
(d) Dipan
153. Each of the ten students was allowed to improve his/her score in exactly one paper of choice with the objective of
maximizing his/her final score. Everyone scored 100 in the paper in which he or she chose to improve. After that, the
topper among the ten students was
(a) Ram
(b) Agni
(c) Pritam
(d) Ayesha
154. Among the top ten students, how many boys scored at least 95 in at least one paper from each of the groups ?
(a) 1
(b) 2
(c) 3
(d) 4
155. Had Joseph, Agni, Pritam and Tirna each obtained Group Score of 100 in the Social Science Group, then their
standing in decreasing order of final score would be
(a) Pritam, Joseph, Tirna, Agni
(b) Joseph, Tirna, Agni, Pritam
(c) Pritam, Agni, Tirna, Joseph
(d) Joseph, Tirna, Pritam, Agni
Directions for Questions 156 - 159: Refer to the following graphs and answer the questions based on them:
Foreign investment approvals and inflows (US $ Mn.)

Country wise foreign investment


approved for 1996

156. What is the contribution of EU in the foreign investment inflows for the year 1996?
(a) 1840
(b) 2000
(c) 460
(d) Data insufficient
157. The foreign investment approvals for the year 1996 in case of SAARC, ASEAN and EU would be :
(a) 2240
(b) 5000
(c) 3500
(d) 2600

PRATHAM: BMS Special Booklet

18

QUANTITATIVE APTITUDE
158. Which of the following can be concluded from the graphs?
I. Foreign investment flows have been steadily increasing
II. Foreign investment approvals have been steadily declining
III. Others contribute to the largest chunk of the foreign investment inflows in 1996.
(a) I only
(b) I and II only
(c) I and III only
(d) None of these
159. If the net foreign investment inflows for the year 1997 were to grow by 20% and if the proportion of inflows were to
remain the same as in case of approvals for the year 1996, what would be foreign investment inflows from NAFTA?
(a) 800
(b) 500
(c) 840
(d) 920
Directions for Questions 160 162: Answer the questions on the basis of the data presented in the figure below.

160. During 1996-2002, the number of commodities that exhibited a net overall increase and a net overall decrease,
respectively, were
(a) 3 and 3
(b) 2 and 4
(c) 4 and 2
(d) 5 and 1
161. The number of commodities that experienced a price decline for two or more consecutive years is
(a) 2
(b) 3
(c) 4
(d) 5
162. For which commodities did a price increase immediately follow a price decline only once in this period ?
(a) Rice, Edible oil and Dal
(b) Egg and Dal
(c) Onion only
(d) Egg and Onion
Directions for Questions 163 166: Answer the questions based on the following pie charts.

163. What fraction of Sandeep's weight consists of muscular and skin protein ?
1
2
3
(a) 50
(b) 25
(c) 50
164. Ratio of distribution of protein in muscles to the distribution of protein in eyes is:
(a) 2:5
(b) 12:50
(c) 13:11

PRATHAM: BMS Special Booklet

(d) Cannot be determined


1

(d) 52 : 25
19

165. What percentage of Ghosh Babu's body weight is made up of bones ?


(a) 0.15%
(b) 10%
(c) 1.2%
(d) Cannot be determined
166. In terms of total body weight, the portion of material other than water and protein is closest to:
1
1
85
1
(a) 5
(b) 15
(c) 100
(d) 20
Directions for Questions 167 - 168: Answer the questions on the basis of the information given below:
Each point in the graph below shows the profit and turnover data for a company. Each company belongs to one of the
three industries: textile, cement and steel.

167. For how many companies does the profit exceed 10% of turnover?
(a) 8
(b) 7
(c) 6
(d) 5
168. For how many steel companies with a turnover of more than 2000 is the profit less than 300?
(a) 0
(b) 1
(c) 2
(d) 7

PRATHAM: BMS Special Booklet

20

LOGICAL REASONING

2
LOGICAL REASONING

PRATHAM: BMS Special Booklet

21

CODING AND DECODING:


Directions for Question 1: In the question a group of letters is followed by four combinations of number/symbol
lettered (a), (b), (c) & (d). Letters are to be coded as per the scheme and conditions given below. You have to find out
the serial letter of the combination, which represents the letter group. Serial letter of that combination is your answer.
Letters
Q M S I N G D K A L P R B J E
Number/Symbol
7 @ 4 # % $ 6 1 2 5 * 9 8 3
Conditions:
(i) If the first letter is a consonant and the last a vowel, both are to be coded as the code of the vowel.
(ii) If the first letter is a vowel and the last a consonant, the codes for the first and the last are to be interchanged.
(iii) If no vowel is present in the group of letters, the second and the fifth letters are to be coded as .
1.

BKGQJN
(a) 9$7%

(b) 9$7%

(c) 91$78%

(d) %1$789

Directions for Question 2: If + means , x means -, means x, - means + then what will come in place of
question mark(?).
2. 36 + 18 12 44 30 = ?
(a) 4
(b) 20
(c) 38
(d) 23.75
Directions for Question 3: In a certain code MANISH is written as NCOKTI, MRADULA is written as
NSCEWMC, PINKI is written as QKOLK.
3. What will be the code for ANUP.
(a) COWQ
(b) TQOCM
(c) CEKUZ
(d) BOVQ
Directions for Question 4: In a language truck is train, train is tractor, tractor is ship, ship is aeroplane, aeroplane
is bulldozer, bulldozer is scooter, then in that language4. Which of the following can fly
(a) aeroplane
(b) ship
(c) bulldozer
(d) train
Directions for Question 5: In a certain code language, hope to see you is coded as re so na di, please come to
see the party is coded as fi ge na di ke zo, hope to come is coded as di so ge and see you the party is coded
as re fi zo na.
5. How is party coded in the given code language?
(a) Either re or fi
(b) Either zo or na
(c) Either zo or fi
(d) Either zo or ge

FAMILY TREE:
6.

Pointing out to a lady, a girl said, She is the daughter-in-law of the grandmother of my fathers only son. How is the
lady related to the girl?
(a) Sister-in-law
(b) Mother
(c) Aunt
(d) Cannot be determined

Directions for Questions 7 8: Read the following information and answer the question given below it:
A is the father of C. But C is not his son. E is the daughter of C. F is the spouse of A. B is the brother of C. D is the son of
B. G is the spouse of B. H is the father of G.
7. Who is the grandmother of D?
(a) A
(b) C
(c) F
(d) H

PRATHAM: BMS Special Booklet

22

LOGICAL REASONING
8.

Who is the son of F?


(a) B

(b) C

(c) D

(d) E

Directions for Questions 9 - 10: Study the following information carefully and answer the questions given below it:
P # Q means P is the father of Q; P + Q means P is the mother of Q; P Q means P is the brother of Q: P * Q
means P is the sister of Q.
9. If A + B # C D, then A is Ds
(a) Father
(b) Grandmother
(c) Sister
(d) Grandfather
10. Which of the following shows that A is the aunt of E?
(a) A + B C * D # E
(b) A # B * C + D E
(c) A * B # C * D E
(d) A B + C # D * E

ARRANGEMENTS AND PUZZLES:


Directions for Questions 11 15: Study the following information carefully and answer the questions given below:
There is a five storey building including the ground floor and each floor has only one flat. All these flats have been
occupied by the five Bank Probationary Officers. Each officer owns a different car. The cars are : Indica, Ikon, Indigo,
Elentra and Santro. The five officers A, B, X, Y and Z are employed in the different banks, viz., State Bank of India (SBI),
Punjab National Bank (PNB), UCO Bank, ICICI Bank and HDFC Bank but not necessarily in the same order. Mr. X works
in HDFC Bank and lives on the ground floor. The officer who lives on fourth (top) floor does not own Elentra or Santro.
Mr. B works in PNB and owns Elentra car. The officer who works in UCO Bank and lives on second floor owns Indica
car. Mr. Y lives on third floor and owns Indigo. Mr. Z works in ICICI bank and Mr. Y works in State Bank of India.
11. Who among the following does own Santro car ?
(a) A
(b) B
(c) X
12. Who lives on the second floor ?
(a) X
(b) Y
(c) Z
13. On which floor does B live ?
(a) First
(b) Second
(c) Fourth
14. Who among the following owns Indica car ?
(a) X
(b) A
(c) Y
15. Which of the following combinations of the officer and car is correct ?
(a) Mr. X: Elentra
(b) Mr. Y: Ikon
(c) Mr. A: Santro
(d) Mr. Z: Ikon

(d) Cannot be determined


(d) A
(d) Cannot be determined
(d) B

Direction for Questions 16 - 20: Read the following information carefully and answer the questions: Seven friends
Kamla, Manish, Rohit, Amit, Gaurav, Pritam and Priya are sitting in a circle. Kamla, Manish, Rohit, Amit, Pritam and Priya
are sitting at equal distances from each other. Rohit is sitting three places right of Pritam, who is sitting to the right of
Amit. Kamla forms an angle of 90 degrees from Gaurav and angle of 120 degrees from Manish. Manish is just opposite
Priya and is sitting on the left of Gaurav.
16. Who is the only person sitting between Rohit and Manish?
(a) Pritam
(b) Amit
(c) Gaurav
17. Gaurav is not sitting at equal distance from
(a) Rohit and Pritam
(b) Amit and kamla
(c) Manish and Pritam
18. Gaurav is sitting . of Priya.
(a) to the left
(b) to the right
(c) two places right
19. The angle between Gaurav and Manish in the clockwise direction is
(a) 1500
(b) 180
(c) 210

PRATHAM: BMS Special Booklet

(d) kamla
(d) All of the above
(d) None of them
(d) None of these

23

20. Which of the following statements is not correct?


(a) Pritam is between exactly Manish and Kamla
(c) Gaurav is sitting opposite Pritam

(b) Manish is two places away from Priya


(d) All of the above

Directions for Questions 21 - 24: Study the following information carefully and answer the questions given below:
In a building, there are thirteen flats on three floors - II, III and IV. Five flats are unoccupied. Three managers, two
teachers, two lawyers and one doctor occupy the remaining flats. There are at least three flats on any floor and not more
than six flats on any floor. No two persons of the same profession stay on any floor. On the II floor, out of four flats, one
occupant is the lawyer and he has only one neighbour. One lawyer live one floor below the other. The doctor is not the
neighbour of any of the lawyers. No flat is unoccupied on the III floor.
21. How many flats are occupied on the IV floor?
(a) Two
(b) Three
(c) Four
22. How many flats are there on the III floor?
(a) Three
(b) Four
(c) Five
23. What is the combination of occupants in the II floor?
(a) Lawyer, Teacher
(b) Manager, Teacher
(c) Manager, Doctor
24. Who among the following is the neighbour of the other lawyer?
(a) Teacher
(b) Manager
(c) Both Manager and Teacher
(d) Data inadequate

(d) Data inadequate


(d) Three or four
(d) Lawyer, Manager

Directions for Questions 25 - 29: Study the following information carefully and answer the questions given below:
A, B, C, D, E, F and G are travelling in three different vehicles. There are at least two passengers in each vehicle - I, II and
III and each vehicle has passengers of both the sexes. There are two engineers, two doctors and three teachers among
them. C is a lady doctor and she does not travel with the pair of sisters A and F. B, a male engineer, travels with only G, a
teacher in vehicle I. D is a male doctor. Two persons belonging to same profession do not travel in the same vehicle. A is
not an engineer and travels in vehicle II.
25. In which vehicle does C travel?
(a) I
(b) II
26. How many lady members are there among them?
(a) Three
(b) Four
27. What is F's profession?
(a) Doctor
(b) Engineer
28. Which of the following is not correct?
(a) A - Female - Teacher
(c) E - Male - Teacher
29. Which of the following represents the three teachers?
(a) A E G
(b) E F G

(c) III

(d) II or III

(c) Three or Four

(d) Data inadequate

(c) Teacher

(d) Data inadequate

(b) B - Male - Engineer


(d) F - Female - Teacher
(c) A E G or E F G

(d) Data inadequate

GROUPS & CONDITIONALITIES:


Directions for Questions 30 33: Read the following information given below and answer the questions that follow:
In an engineering college, there are three departments- civil, mechanical and electrical. There are, in all, 20 faculties. Of
them, five are male professors (A, H, I, O and P), seven are male lecturers (B, C, D, J, K, Q and R), three are female
professors (E, L and S), and five are female lecturers (F, G, M, N and T). Civil department has the staff members from A
to G. Mechanical department has staff members from H to N and electrical department has staff members from O to T.
Different teams are to be formed out of these staff members as per the following conditions:
I.
No male lecturer in civil department can be grouped with any female lecturer in electrical department.

PRATHAM: BMS Special Booklet

24

LOGICAL REASONING
II.

No female professor can be grouped with any male lecturer.

30. A committee of six members is to be formed with one professor and one lecturer each from each department.
There must be a minimum of two female members in this committee. Which of the following combinations is
possible?
(a) A, I, P, B, L, N
(b) H, I, P, E, L, T
(c) A, H, O, F, S, T
(d) A, F, H, M, S, T
31. A delegation of ladies is to be sent abroad to attend a seminar. Any professor in mechanical department is not ready
to join with any lecturer in civil department. The delegation should have minimum three professors. Further, every
department must be represented in the delegation. Which of the following combinations would be possible?
(a) E, L, S, F, G, N
(b) A, H, D, E, L, S
(c) F, L, N, T, S, M
(d) E, L, S, M, N, T
32. A joint committee of five members is to be formed from the faculties of civil and electrical department only. The
committee must have minimum two professors and two lecturers. There should be at least two female members in
the committee. Which of the following combinations is not possible?
(a) S, T, D, O, R
(b) A, B, O, R, T
(c) A, O, R, G, T
(d) A, O, B, G, T
33. A department committee is to be formed in mechanical department only. There should be 4 members in the
committee and one of them must be a lady. The committee must include the head of department, Professor I.
However, he cannot work with either L or N. Further, M cannot work with either N or J. Which of the following
combinations is possible?
(a) I, M, L, N
(b) I, M, H, K
(c) I, H, J, K
(d) H, J, M, N
Directions for Questions 34 38: Read the information given in the following paragraphs and then answer the
questions based on them:
There are 6 soccer teams A, B, C, D, E and F in the soccer league game. All the 6 teams play each Saturday during the
season. Each team must play against each of the other teams once and only once during the seasons.
(I) Team A plays against team D first and team F second.
(II) Team B plays against team E first and team C third.
(III) Team C plays against team F first.
34. On the first Saturday, which of the following pairs of teams play against each other?
(a) A and B, C and F, D and E
(b) A and B, C and E, D and F
(c) A and C, B and E, D and F
(d) None of these
35. Which of the following team must B play second with ?
(a) A
(b) C
(c) D
(d) Either E or A
36. What is the total number of games that each team must play during the season ?
(a) 1
(b) 2
(c) 3
(d) 5
37. If D wins 5 games, which of the following must be true ?
(a) A loses 5 games
(b) A wins 4 games
(c) A wins its first game (d) B loses at least 1 game
38. The last set of games could be between :
(a) A and B, C and F, D and E
(b) A and C, B and F, D and E
(c) A and D, B and C, E and F
(d) Cannot be determined
Directions for Questions 39 42: Study the following information to answer the given questions:
There are five identically looking boxes having different objects in it and every box has a label indicating their contents.
The following is the description of the contents and the label of each box:
Contents
Label
Two pins
PP
Two balls
BB
Two clips
CC
One pin and One clip
PC
One ball and One clip
BC

PRATHAM: BMS Special Booklet

25

Somebody has mischievously interchanged these labels in such a way that no box contains the label correctly explaining its
content.
39. If the first box opened contained label PP and second box opened contained label PC and out of the combined four
items, one item was a ball, which of the following will be definitely true?
(a) Other three items will not contain any clip
(b) Other three items will contain atleast one clip
(c) Other three items will not contain two pins
(d) None of the above
40. If the first box, containing the label BC, was opened and it was found that one item is a ball, which of the following
would definitely be true?
(a) The other item will not be a ball
(b) The other item will also be a ball
(c) The other box with BB label will contain a ball and a clip
(d) The other item may either be a ball or a clip
41. If the information is available that box PC does not contain either any pin or any clip and box PP does not contain
any pin and box CC contains one clip and one ball, which of the following will definitely be true if only one of the
remaining boxes is opened?
(a) It will have atleast one clip
(b) It will have atleast one pin
(c) It will have two pin
(d) It will have one pin and one clip
42. If the box PP contained two clips, the box CC contained two pins and the box BB contained atleast one ball, which
of the following will definitely be not true?
(a) The box PC contains two balls
(b) The box BB contains one clip
(c) The box BC contains two balls
(d) The box BC contains one pin and one clip

DIRECTIONS:
43. Radha moves towards South-east a distance of 7 m, then she moves towards West and travels a distance of 14 m.
From here, she moves towards North-west a distance of 7 m and finally she moves a distance of 4 m towards East
and stood at the point. How far is the starting point from where she stood?
(a) 3 m
(b) 4 m
(c) 10 m
(d) 11 m
44. A man is facing south. He turns 135o in the anti clockwise direction and then 180o in the clockwise direction. Which
direction is he facing now?
(a) North-east
(b) North-west
(c) South-east
(d) South west
45. A villager went to meet his uncle in another village situated 5 km away in the North-east direction of his own village.
From there he came to meet his father-in-law living in a village situated 4 km in the south of his uncles village. How
far away and in what direction is he now?
(a) 3 km in the North
(b) 3 km in the East
(c) 4 km in the East
(d) 4 km in the West
46. A man leaves for his office from his house. He walks towards East. After moving a distance of 20 m, he turns South
and walks 10 m. Then he walks 35 m towards the West and further 5 m towards the North. He then turns towards
East and walks 15 m. What is the straight distance between his initial and final positions?
(a) 0
(b) 5
(c) 10
(d) Cannot be determined

PRATHAM: BMS Special Booklet

26

LOGICAL REASONING
47. A river flows west to east and on the way turns left and goes in a semi-circle round a hillock, and then turns left at
right angles. In which direction is the river finally flowing?
(a) West
(b) East
(c) North
(d) South

VENN DIAGRAM:
48. How many triangles are there in the given figure?
(a) 18
(c) 20

(b) 28
(d) 24

49. Find the minimum number of straight lines required to make the given figure.
(a) 16
(c) 18

(b) 17
(d) 19

50. In the Venn diagram given below, circle represents sportspersons, square represents unmarried person, triangle
represents women and rectangle represents educated person. Each section is numbered. Study this diagram to
answer the following question.

Which sections is/are represented by the region numbered 10?


(a) Married educated sportswomen
(b) Unmarried uneducated women sportspersons
(c) Married educated sportsmen
(d) Unmarried educated sportswomen
51. Which one will replace the question mark?
(a) 0
(c) 11

(b) 26
(d) 12

52. Which one will replace the question mark?

(a) 262

(b) 622

PRATHAM: BMS Special Booklet

(c) 631

(d) 824

27

SYLLOGISM:
53. Statements:
Some pianos are violins.
Some violins are drums.
All drums are guitars.
No guitar is a flute.
Conclusions:
I. At least Some guitar are pianos.
II. All drums are flutes is a possibility
III. Some pianos are both violins and drums.
IV. At least some flute is a drum.
(a) Only IV follows
(b) Only either I or II follow
54. Statements:
All chairs are tables.
All tables are telephones.
All telephones are cell phones.
No cell phone is computer
Conclusions:
I. All cell phones are tables.
II. Some chairs are computers
III. No chair is computer.
(a) Only I follows
(b) Only II follows

(c) Only III follows

(d) None Follows

(c) Only III follows

(d) Only either II or III follows

Directions for Questions 55 to 57: Each question contains six statements, followed by four options of combinations of
any three of the given sentences. Choose the option in which the combinations are logically related.
55. A. No tingo is a bingo.
B. All jingoes are bingoes.
C. No jingo is a tingo.
D. Some tingoes are jingoes.
E. Some tingoes are jingoes.
F. Some bingoes are not tingoes.
(a) ABC
(b) ACB
56. A. Smoking causes pollution.
B. Smoking creates ill-will.
C. Smoking sometimes causes fire.
D. Ill-will causes pollution.
E. Smoking causes fire and pollution.
F. All smokers cause fire and pollution.
(a) ABE
(b) FEC
57. A. Lakme is a nail polish.
B. Some nail polishes contain oil.
C. All nail polishes do not contain oil.
D. All nail polishes contain chemicals.
E. Lakme has oil.
F. Lakme contain chemicals.
(a) BCE
(b) AEB

PRATHAM: BMS Special Booklet

(c) DFA

(d) BDA

(c) DBF

(d) BDA

(c) FDA

(d) AEF

28

LOGICAL REASONING
RANKING & SERIES:
58. In a row of boys, Aditya was shifted 10 places to the right of Rohan and Kewal was shifted 10 places to the left of
Vilas. If Vilas was twenty-sixth from the left and there were three boys between Kewal and Aditya after shifting,
what was the position of Rohan in the row?
(a) 10th from the right end
(b) 10th from the left end
th
(c) 39 from the right end
(d) Data inadequate
59. In a class, among the passed students, Amisha is twenty-second from the top and Anuja, who is 5 ranks below
Amisha, is thirty-fourth from the bottom. All the students from the class have appeared for the exam. If the ratio of
the students who passed in the exam to those who failed is 4:1 in the class, how many students are there in the
class?
(a) 60
(b) 75
(c) 90
(d) Data inadequate
60. In a row of girls, there are 16 girls between Priya and Natasha. Priya is thirty-second from the left end of the row. If
Priya is nearer than Natasha to the right end of the row, then how far away is Natasha from the left end of the row?
(a) 16th
(b) 14th
(c) 15th
(d) Data inadequate
Direction for Questions 61 - 62: Study the following arrangement carefully and answer the question given below.
34268754*329$16537#986@2143987243
61. How many 2s are there in the above arrangement, each of which is immediately followed by a perfect square?
(1 is also a perfect square)
(a) None
(b) One
(c) Two
(d) Three
62. If all the digits that are perfect square are dropped from the above arrangement which of the following will be 13th
(digit/symbol) from the left end of the above arrangement.
(a) $
(b) 2
(c) #
(d) 5
Directions for Questions 63 - 64: The following questions are based on five words given below:
AND FOR THE BIG SUM
(The new words formed after performing the mentioned operations may or may not necessarily be meaningful English
words)
63. If the third alphabet of each of the words is changed to the next alphabet in the English alphabetical series, which of
the following will form a meaningful English word?
(a) SUM
(b) BIG
(c) Both BIG and THE (d) FOR
64. How many letters are there in the English alphabetical series between the second letter of the word which is second
from the right and the second letter of the word which is second from the left of the given words?
(a) Two
(b) Five
(c) Six
(d) Nine
th
st
65. Radha remembers that her fathers birthday is after 16 but before 21 of March, while her brother Mahesh
remembers that his fathers birthday is before 22nd but after 19th of March. On which date is the birthday of their
father?
(a) 19th March
(b) 20th March
(c) 21th March
(d) Cannot be determined
66. Nikhil remembers that his sister Kranti had certainly been to Pune for one day after 16th February, but before 21st
February, while his mother remembers that Kranti had certainly visited Pune, before 20th February but after 17th
February. If both of the them are correct, then on which day in February had Kranti visited Pune?
(a) Either 18th or 19th
(b) 19th
th
(c) 18
(d) Data inadequate

PRATHAM: BMS Special Booklet

29

67. If it is possible to make a perfect odd square from 3rd, 5th and 10th digit of the number 2569187943, the unit digit of
the square root would be your answer. If more than one such square can be formed your answer would be @and if
no such number formed your answer is .
(a) 1
(b)
(c) 9
(d) @

DATA SUFFICIENCY:
Directions for Questions 68-69: Each of the following questions consists of a question followed by three statements
labeled I, II and III. You have to decide whether the data given in the statement(s) is/are sufficient for answering the
question. Read all the questions. A single combination of statements with least number of statements which could be
sufficient for answering the question would be your answer.
68. How is Aditya related to Mayank?
I. Pinki is the mother-in-law of Rashmi, the wife of Aditya. II. Pinkis brother is Adityas maternal uncle.
III. Pinkis husband is the only son of Mayank.
(a) Only I and II
(b) Only I and III
(c) Only I and either II or III
(d) Any two of the three
69. Who among A, B, C, D, E, F, and G each having a different height, is the second tallest?
I. B is taller than A but shorter than E.
II. Only two of them are shorter than G.
III. D is taller than only F.
(a) Only I and II
(b) Only II and III
(c) Only I and III
(d) All I, II and III are not sufficient to answer the question.
Directions for Questions 70 - 72: Each of the following questions below consists of a question and two statements
numbed I and II given below it. You have to decide whether the data provided in the statements are sufficient to answer
the question. Read both the statements and give answer.
(a) If data in statement I alone are sufficient to answer the question, while the data in statement II alone are not sufficient
to answer the question.
(b) If data in statement II alone are sufficient to answer the question, while the data in statement I alone are not sufficient
to answer the question.
(c) If the data either in statement I alone or in statement II alone are sufficient to answer the question.
(d) If the data even in both statements I and II together are not sufficient to answer the question.
(e) If the data in both statements I and II together are necessary to answer the question.
70. On which day is Vickys birthday?
I. As per Vickys friend Rahul, his birthday is on either 12th, 13th or 14th Jan.
II. As per Vickys mother Babli, his birthday is on either 14th or 15th Jan.
71. Is T grandmother of Q?
I. P is the mother of Q. Q is the son of R. R is the son of T.
II. L is father of N and N is daughter of T.
72. Is it 9 oclock now?
I.
After half an hour, the minute and the hour hands of the clock will make an angle of exactly 90o with each
other.
II. Exactly 15 minutes ago, the hour and the minute hands of the clock coincided with each other.

PRATHAM: BMS Special Booklet

30

LOGICAL REASONING
STATEMENT AND CONCLUSION:
Directions for Questions 73 and 74: In each question below a statement is given followed by two conclusions
numbered I and II. You have to take the statements to be true. Read both the conclusions and decide which of the two or
both follow from the given statement.
Give Answer
(a) If only conclusion I follows.
(b) If only conclusion II follows.
(c) If either I or II follows.
(d) If neither I nor II follows.
(e) If both conclusions I and II follow.
73. Statement:
Global ecological issues have eclipsed local environmental problems, which are being faced by the poor societies.
Conclusions:
I. Poor societies always have to suffer because of their poverty.
II. Global ecological issues are not so important. Rich societies can bear with it.
74. Statements:
70% of the worlds data is processed on XYZ companys platforms An advertisement of XYZ a computer
manufacturing company.
Conclusions:
I. There is no other company in the world which produces platforms of data processing.
II. Company XYZ has to make more efforts to market its platforms.
Directions for Questions 75 and 76: In each question below are given certain statements followed by some
conclusions. Choose the conclusion which follows from the given statement.
75. Statements:
A forest has as many Sandal trees as it has Ashoka trees. Three-fourths of the trees are old and half of the trees are
at the flowering state.
Conclusions:
(a) All ashoka trees are at the flowering stage.
(b) All Sandal trees are at the flowering stage.
(c) At least one-half of the Ashoka trees are old.
(d) One half of the Sandal trees are at the flowering stage.
(e) None of these.
76. Statements:
1. Only students can participate in the race.
2. Some participants in the race are females.
3. All female participants in the race are invited for coaching.
Conclusions:
(a) All participants in the race are invited for coaching.
(b) Al participants in the race are males.
(c) All students are invited for coaching.
(d) All the participants in the race are students.

PRATHAM: BMS Special Booklet

31

Directions for Question 77: In each question below a statement is given followed by two conclusions numbered I and
II. You have to take the statement to be true. Read both the conclusions and decide which of the two or both follow from
the given statement.
Give Answer
(a) If only conclusion I follows.
(b) If only conclusion II follows.
(c) If either I or II follows.
(d) If neither I nor II follows.
(e) If both conclusions I and II follow.
77. Statement:
The top management has asked the four managers either to resign by tomorrow or face the order of service
termination. Three of them have resigned till this very evening.
Conclusions:
I.
The manager who did not resign yesterday will resign tomorrow.
II. The management will terminate the service of one manager.

VISUAL REASONING:
Directions for Questions 78 to 82: In each of the following questions, there is a set of five figures labelled A, B, C, D
and E called the Problem Set followed by a set of five other figures labelled (a), (b), (c), (d) and (e) called the Answer Set.
Fig. (C) or (D) contains a question mark. Select a suitable figure from the Answer Set which will substitute this question
mark so that a series is formed by the figure A, B, C, D and E taken in order. The option of the selected figure is the
answer.
78.

Problem figure

Answer figure

79.

80.

81.

PRATHAM: BMS Special Booklet

32

LOGICAL REASONING
82.

CUBES & DICES:


83. Three views of the same cube are shown below -

The figure on the face opposite to the triangle is:


(a) Pentagon
(b) Circle

(c) Question mark

(d) Rectangle

Directions for Questions 84 to 86: Read the information given below to answer these questions.
(A) A cube has six sides, each of which has a different colour black, blue, brown, green, red and white.
(B) The red side is opposite to black.
(C) The green side is between the red and the black.
(D) The blue side is adjacent to the white.
(E) The brown side is adjacent to the blue.
(F) The red side is the bottom face.
84. The four colours adjacent to green are
(a) black, blue, brown, red
(b) black, blue, brown, white
(c) black, blue, red, white
(d) black, brown, red, white
85. Which of the following can be deduced from the statements (A), (B) and (F)?
(a) Black is on the top
(b) Blue is on the top
(c) Brown is on the top
(d) Brown is opposite to black
86. Which of the following statements given above adds no information that is not already given by the other
statements?
(a) (B)
(b) (C)
(c) (E)
(d) (F)
87. Four different positions of a dice are shown. Find the letter opposite to W.

(a) Y

(b) U

PRATHAM: BMS Special Booklet

(c) M

(d) S

33

INPUT - OUTPUT:
Directions for Questions 88 - 92: Read the following information carefully to answer these questions.
A word number arrangement machine when given an input as set of words-numbers, rearranges them following a
particular rule and generates stepwise output till the rearrangement is completed following that rule.
Following is an illustration of input and steps of rearrangement till the last step.
Input
: pour ask 57 dear 39 fight 17 28
Step I
: ask pour 57 dear 39 fight 17 28
Step II
: ask 57 pour dear 39 fight 17 28
Step III
: ask 57 dear pour 39 fight 17 28
Step IV
: ask 57 dear 39 pour fight 17 28
Step V
: ask 57 dear 39 fight pour 17 28
Step VI
: ask 57 dear 39 fight 28 pour 17
And step VI is the last output.
As per the rule followed in the above steps, find out the answer to each of the following questions.
88. If step II of an input is cut 97 38 end for 29 46 down, which of the following will be the last step?
(a) VI
(b) IV
(c) V
(d) VII
89. If the Step I of an input is car 17 vas tiger 92 87 like 52, which of the following will be the Step IV?
(a) car 92 like 87 tiger 17 vas 52
(b) car 92 like 87 17 vas tiger 52
(c) car 92 like 87 tiger 52 17 vas
(d) car 92 like 17 vas tiger 87 52
90. Input : zeal for 49 31 high 22 track 12. Which of the following will be the Step III?
(a) for 49 high zeal 31 22 track 12
(b) for 49 high 31 zeal 22 track 12
(c) for 49 high 31 track 22 zeal 12
(d) for 49 high 31 track zeal 22 12
91. Input : 19 feet 34 28 dog bag 43. Bag 43 dog 19 feet 34 28 would be the Step(a) I
(b) IV
(c) II
(d) None of these
92. If the step III of an input is 21 let 86 bite 16 fit 72 happy, what would be the input?
(a) 21 16 let bite happy fit 72 86
(b) happy 16 21 bite fit let 86 72
(c) fit bite happy 72 86 16 let 21
(d) cannot be determined
Directions for Questions 93 - 97: In arrangement machine when given an input of words, rearrange them following a
particular rule and solve.
Input: This chapter would give you basic concepts.
Step 1: basic this chapter would give you concepts.
Step 2: basic chapter this would give you concepts.
Step 3: basic chapter concepts this would give you.
Step 4: basic chapter concepts give this would you.
Study the logic and answer the questions that follow.
93. Input: Do not imitate others work. Which of the following will be Step 3 for the given input?
(a) Do others not imitate work.
(b) Do imitate not others work.
(c) Work do others imitate.
(d) Do not others work imitate.
(e) There will be no 3rd step
94. Input: "Adequate safety arrangements are must for kids". Which of the following will be 2nd last step for the
given input?
(a) Adequate are arrangements for kids safety must.
(b) Adequate are arrangements for kids must safety,
(c) Adequate are safety arrangements must for kids.
(d) Adequate safety are arrangements must for kids.
(e) None of these

PRATHAM: BMS Special Booklet

34

LOGICAL REASONING
95. Input: "Marcus Brauchli gives inaccurate information". How many steps are required to rearrange the above
input properly.
(a) 3
(b) 4
(c) 5
(d) 2
(e) None of these
96. Input: "Most Indian leaders cannot even visualise free India". Which of the following will be last step of above
input?
(a) Most Indian even leaders cannot visualise free India.
(b) Cannot even free India Indian most leaders visualise.
(c) Cannot even free India most Indian leaders visualise.
(d) Cannot even free India Indian leaders most visualise.
(e) None of these.
97. Step IV: "It is figure anti-establishment from media". Which of the following is the input of the above
mentioned step.
(a) It is figure from anti-establishment media.
(b) Media it is figure from anti-establishment.
(c) It is anti-establishment media from.
(d) Can't be determined.
(e) None of these

STATEMENT & ASSUMPTIONS:


Directions for Questions 98 101: In each question below is given a statement followed by two assumptions
numbered I and II. An assumption is something supposed or taken for granted. You have to consider the statement and
the following assumptions and decide which of the assumptions is implicit in the statement. Give answer(a) if only assumption I is implicit.
(b) if only assumption II is implicit.
(c) if either assumption I or II is implicit.
(d) if neither assumption I nor II is implicit.
(e) if both assumptions I and II are implicit.
98. Statement: Please check the availability of 2 tickets from Delhi to Lucknow.
Assumptions: I. Person checking knows the desired mode of travel.
II. Person checking knows the details of the person travelling.
99. Statement: An advertisement: Our shoes are for the rich.
Assumptions: I. Many people like to be labeled as rich.
II. One cant become rich unless one has that brand of shoes.
100. Statement: Two months ago, it was announced that Central Government pensioners would get dearness relief
with immediate effect but till date, banks have not credited the arrears. A statement from a Pensioners Forum.
Assumptions: I. Most of the banks normally take care of the pensioners.
II. Two months time is sufficient for the government machinery to move and give effect to
pensioners.
101. Statement: Believe me, I have read it in newspaper X.
Assumptions: I. Newspaper X gives reliable information/news.
II. I am reporting exactly as it is given in newspaper X
102. Statement: We deal in used cars. Contact us at phone no. XYZ, at the earliest possible. an advertisement.
Assumptions: I. some people want to sell or purchase old cars.
II. The advertisement will be read by the needy people.
III. Used cars may not be totally useless.
(a) Only I are implicit.
(b) Only II and III are implicit.
(c) Only I and III are implicit.
(d) All I, II, III are implicit.
(e) None of I, II, III are implicit.

PRATHAM: BMS Special Booklet

35

STATEMENT & COURSE OF ACTION:


103. Statement: Number of dropouts from the municipal school has significantly increased after withdrawal of mid-day
meal scheme.
Course of Action:
I.
The government should reconsider its decision of withdrawal of midday meal scheme.
II. The government should close down some of the municipal schools.
III. The government should carry out a detailed study to find out the reasons for school dropouts.
(a) None follows
(b) Only I follows
(c) Only I and III follow (d) Only III and III follow
(e) All follow
104. Statement: Some strains of mosquito have become resistant to chloroquine the widely used medicine for malaria
patients.
Course of Action:
I.
Selling of chloroquine should be stopped.
II. Researchers should develop a new medicine for patients affected by such mosquitoes.
III. All the patients suffering from malaria should be checked for identification of causal mosquito.
(a) None follows
(b) Only I and III follow
(c) All follow
(d) Only II and III follow
(e) None of these
105. Statement: A large number of students who have passed their XII Std. terminal examination in the country could
not get admission to colleges as the number of seats available are grossly inadequate.
Course of Action:
I.
The evaluation system of XII std. terminal examination should be made more tough so that fewer students pass
the examination.
II. The Government should encourage the private sector to open new colleges by providing them land at cheaper
rate.
III. The rich people should be asked to send their wards to foreign countries for higher studies enabling the needy
students to get admission in colleges within the country.
(a) Only I follows
(b) Only II follows
(c) Only I and II follow (d) Only II and III follow
(e) None of these
106. Statement: The Deputy Mayor of city Z has proposed to install a plant of mineral water and to supply citizens
mineral water bottles at Rs. 6 per litre as against Rs. 10 per litre being sold by local private companies.
Course of Action:
I.
The local private companies of city Z will have to close their operation.
II. The Corporation of city Z will have to provide for losses in this project in its budget.
III. The tap water schemes of city Z will have to be stopped.
(a) Only I and II follow
(b) Only II and III follow
(c) Only I and III follow (d) All follow
(e) None of these
107. Statement: The vehicular traffic has increased so much in the recent past that it takes at least two hours to travel
between the city and the airport during peak hours.
Course of Action:
I.
Non-airport bound vehicles should not be allowed to ply on the road connecting the city and the airport.
II. The load of vehicular traffic should be diverted through various link roads during peak hours.
III. The departure and arrival of flights should be regulated so as to avoid congestion during peak hours.
(a) Only I follow
(b) Only II follows
(c) Only I and II follow (d) All follow
(e) none of these

PRATHAM: BMS Special Booklet

36

LOGICAL REASONING
STATEMENT & ARGUMENT:
Directions for Questions 108 to 112: Each of the following questions consists of a statement followed by two
Arguments I and II.
Give answer:
(a) If only argument I is strong
(b) If only argument II is strong
(c) If either I or II is strong
(d) If neither I nor II is strong and
(e) If both I and II are strong
108. Statement: Should religion be banned?
Arguments:
I.
Yes. It develops fanaticism in people.
II. No. Religion binds people together.
109. Statement: Should children be legally made responsible to take care of their parents during their old age?
Arguments:
I.
Yes. Such matter can only be solved by legal means.
II. Yes. Only this will bring some relief to poor parents.
110. Statement: Should India encourage exports, when most things are insufficient for internal use itself?
Arguments:
I.
Yes. We have to earn foreign exchange to pay for our imports.
II. No. Even selective encouragement would lead to shortages.
111. Statement: Should family planning be made compulsory in India?
Arguments:
I.
Yes. Looking to the miserable conditions in India, there is no other go.
II. No. In India there are people of various religions and family planning is against the tenets of some of the
religions.
112. Statement: Should the vehicles older than 15 years be rejected in metros in India?
Arguments:
I.
Yes. This is a significant step to lower down the pollution level in metros.
II. No. It will be very difficult for vehicle owners to shift to other parts in country because they will not get suitable
job for their very existence.

ASSERTION & REASON:


Directions for Questions 113 114: Each of these questions has an assertion (A) and a reason (R). Mark answer as
(a) if both (A) and (R) are true but (R) is not the correct explanation of (A)
(b) if both (A) and (R) are true and (R) is the correct explanation of (A)
(c) if (A) is true but (R) is false
(d) if (A) is false but (R) is true
113. Assertion (A) The government must take care of the minor children of those criminals who have been sentenced
to jail and cant look after their children any more under these circumstances.
Reason (R) Yes, psychologically, the children need protection from someone, be it the government. But the
criminals must have thought about this aspect before they committed a crime for which they are in the jail now.
114. Assertion (A) The freezing of sea water at minus zero level temperature does not kill the fish.
Reason (R) It is only the top layer of the seawater that gets frozen. The lower level water remains unfrozen and fish
can remain alive in it.

PRATHAM: BMS Special Booklet

37

Directions for Question 115: For the Assertion (A) and Reason (R) below, choose the correct alternative from the
following
(a) A is true but R is false
(b) Both A and R are true but R is not the correct explanation of A
(c) Both A and R are true and R is the correct explanation of A
(d) A is false but R is true
115. Assertion (A) There appears to be a conflict between work-life and family life.
Reason (R) One is more money minded.
Directions for Questions 116 117: Each of these questions has an Assertion (A) and a Reason (R). Mark answer as
(a) if both A and R are true and R is the correct explanation of A
(b) if both A and R are true but R is not the correct explanation of A
(c) if A is true but R is false
(d) if A is false but R is true
(e)
116. Assertion (A) An uneducated person is a good stenographer.
Reason (R) A stenographer also knows typing.
117. Assertion (A) A married man is a happy person.
Reason (R) Marriage is a social obligation.

INFERENCE FROM PASSAGE:


Directions for Questions 118 126: Below is given a passage followed by some inferences. You have to examine each
inference separately in the context of the passage and decide upon its degree of truth or falsity.
Mark answer:
(a) If you think the inference is definitely true;
(b) If you think the inference is probably true though not definitely true in the light of the facts given;
(c) If the date given is inadequate i.e., from the fact given you cannot say whether the inference is likely to be true or
false;
(d) If you think the inference is probably false though not definitely false in the light of facts given; and
(e) If the inference is definitely false i.e., it contradicts the given facts.
Passage I
The interest rates in the institutional market have come down significantly and at present are in the range of 5.25 to 5.75
percent per annum. However, interest rates on small savings schemes such as the Post Office Saving Schemes, RBI Relief
Bonds, Provident Fund and PPF continue to be administered at high levels in relation to market rates. These schemes also
have a variety of tax incentives, which result in much higher post-tax returns and on the flip side more liability for the
government. It is ironic that it is predominantly the urban population (and more often people in the high tax bracket),
which is benefiting from high rates and tax benefits. The size of small savings has assumed a significant proportion of
financial savings in the country. This has also increased dramatically the debt service burden of both Central and State
Governments.
118. The interest rate in the institutional market till five years ago was more than 8 percent.
119. The interest rates on small savings have decreased considerably during the last few years.
120. Tax incentives are provided to benefit all sections of the society.
121. Returns on different Government administered savings schemes are higher for tax-paying individuals.
122. The Government administered savings schemes in the long run drain out exchequer's money.

PRATHAM: BMS Special Booklet

38

LOGICAL REASONING
Passage II
Pollution amounts to slow murder. Regular exposure to industrial and vehicular pollution leads to life threatening diseases
like asthma, heart problems, cancer and various other disorders. Therefore, nobody has the right to pollute, rich or poor.
Industrial and vehicular pollution are growing rapidly across the country. It is not just metropolitan centres that are heavily
polluted today but also small and medium towns. Pollution is growing faster than the economy. This is because the
western technological model, built on heavy use of energy and materials, is an inherently highly toxic model. It produces
huge amounts of toxic pollutants, which can be controlled if there is careful choice of technology and there is
considerable discipline in its use.
123. The industries which use higher levels of energy create more pollution.
124. The spread of pollution has gathered momentum in the recent past.
125. The pollution level in the western world is considerably more than in India.
126. Proper planning in use of modern technology leads to less pollution in the environment.

PRATHAM: BMS Special Booklet

39

3
VERBAL ABILITY

PRATHAM: BMS Special Booklet

40

VERBAL ABILITY
SYNONYMS:
Directions for Questions 1 - 10: Choose the option closest in meaning to the word given.
1.

Disparage
(a) separate
2. Intrepid
(a) middle
3. Parley
(a) fraud
4. Lassitude
(a) lethargy
5. Idiocy
(a) brilliancy
6. Stockade
(a) rampart
7. Bestial
(a) feral
8. Paddock
(a) filament
9. Anchorite
(a) eremite
10. Dither
(a) dilly-dally

(b) compare

(c) refuse

(d) belittle

(b) tolerant

(c) fearless

(d) rude

(b) paraphrase

(c) conclave

(d) spectacle

(b) puritan

(c) energy

(d) meeting

(b) privilege

(c) dogma

(d) absurdity

(b) fence

(c) paling

(d) shuttle

(b) feminine

(c) fiendish

(d) savage

(b) glebe

(c) pasture

(d) mead

(b) recluse

(c) infirm

(d) hermit

(b) falter

(c) hum and haw

(d) doughty

ANTONYMS:
Directions for Questions 11 - 20: Choose the option which is the antonym of the word mentioned.
11. Apex
(a) base
12. Haughtiness
(a) unskilled
13. Awry
(a) Austere
14. Befoul
(a) flounder
15. Blasphemous
(a) irreligious
16. Paradoxical
(a) rational
17. Callow
(a) mature
18. Candour
(a) frankness
19. Diffidence
(a) ascent
20. Cleave
(a) disjunction

(b) zenith

(c) meridian

(d) median

(b) affability

(c) adduce

(d) abject

(b) offend

(c) asthenia

(d) straight

(b) cleanse

(c) buoyant

(d) flamb

(b) inferior

(c) reverent

(d) blarney

(b) crazy

(c) daft

(d) zany

(b) oppose

(c) bustle

(d) burly

(b) cunning

(c) ingenuous

(d) sincere

(b) confidence

(c) penchant

(d) prescription

(b) separate

(c) adjunct

(d) revoke

PRATHAM: BMS Special Booklet

41

ANALOGIES:
Directions for Questions 21- 30: Answer the questions based on the following information.
In each of the following questions, a pair of capitalized words is followed by four pairs of words. You are required to
mark as the answer the pair of words which have a relationship between them most similar to the relationship between
the capitalized pair.
21. DULCET : RAUCOUS
(a) Sweet : Song
(c) Palliative : Exacerbating
22. ACTION : REACTION
(a) Introvert : Extrovert
(c) Diseased : Treatment
23. MALAPROPISM : WORDS
(a) Anachronism : Time
(c) Jinjanthropism : Apes
24. FOOD : GOURMET
(a) Book : Critic
(b) Art : Connoisseur
25. LIMPID : MURKY
(a) Dazed : Clouded
(b) Obscure : Vague
26. SAUNTER : STROLL ::
(a) perambulate : walk
(b) gyrate : twist
27. SLANDER : REPUTATION ::
(a) deceit : hope
(b) indiscretion : secrecy
28. ARTIST : CANVAS ::
(a) drive : car
(c) composer : symphony
29. SCABBARD : SWORD ::
(a) gamble : cards
(b) cup : beverage
30. UNCLE : AVUNCULAR ::
(a) mother : maternity
(b) father : patricide

(b) Crazy :Insane


(d) Theory : Practical

(CAT 1996)

(b) Assail : Defend


(d) Death : Rebirth

(CAT 1996)

(b) Ellipsis : Sentence


(d) Catechism : religion

(CAT 1996)
(CAT 1993)

(c) Sports : Fan

(d) Craft : Skill

(c) Bright : Gloomy

(d) Nebulous : Dim

(c) amble : path

(d) baby carriage : walk

(c) hypocrite : praise

(d) vandalism : property

(CAT 1995)

(b) pedestrian : road


(d) surgeon : operation
(c) wish : intuition

(d) foot : shoe

(c) brother : fraternal

(d) sister : sorority

IDIOMS:
Directions for Questions 31- 40: In each of the following questions, an idiom or a phrase has been given, followed by
four alternatives. Choose the one which best expresses the meaning of the given idiom / phrase.
31. To save ones face
(a) to hide oneself
(b) to oppose
32. Hush money
(a) Money overdue
(c) Money earned by wrong means
33. To be lost in the cloud
(a) To meet with ones death
(c) To be concealed from view
34. A ladys man
(a) A woman tailor
(c) A man working as per a ladys directions

PRATHAM: BMS Special Booklet

(c) To evade disgrace

(d) To say plainly

(b) Easy money


(d) Bribe paid to secure silence
(b) To be perplexed
(d) to find oneself in a very uncomfortable position
(b) A lover of woman company
(d) A timid husband
42

VERBAL ABILITY
35. To worship the rising sun
(a) To honour a man who is coming home
(c) To indulge in flattery
36. To disabuse ones mind
(a) To conceal something
(b) To remove a misapprehension
(c) To banish from ones mind a thought
(d) To proceed cautiously so as to avoid risks and dangers
37. To tempt providence
(a) To invite punishment
(c) To take reckless risks
38. To cross swords
(a) To fight
(b) To defend
39. All agog
(a) Everybody
(b) All ready
40. To bite ones lips
(a) To be angry
(b) To feel sorry

(b) To honour the promising people


(d) To welcome the coming events

(b) To achieve a fortune


(d) To have Gods favour
(c) To kill

(d) To rob

(c) Restless

(d) Almighty

(c) To have doubt

(d) To laugh at others

SENTENCE COMPLETION:
Directions for Questions 41 - 49: Answer the questions based on the following information.
There are two gaps of the following sentences. From the pairs of words given, choose the one that fill the gap most
appropriately. The first word in the pair should fill the first gap.
41. From the time she had put her hair up, every man she had met had groveled before her and she had acquired a
mental attitude toward the other sex which was a blend of ____ and ___.
(a) admiration, tolerance
(b) indifference, contempt
(c) impertinence, temperance
(d) arrogance, fidelity
(CAT 2003)
42. Physicians may soon have ____ to help paralyzed people move their limbs by bypassing the ____ nerves that once
controlled their muscles.
(a) instruments, detrimental
(b) ways, damaged
(c) reason, involuntary
(d) impediments, complex
(CAT 2003)
43. The argument that the need for a looser fiscal policy to _____ demand outweighs the need to ___ budget deficits is
persuasive.
(CAT 2003)
(a) assess, minimize
(b) outstrip, eliminate
(c) stimulate, control (d) restrain, conceal
44. But____ are now regularly written not just for tools but well-established practices, organisations and institutions
not all of which seem to be ____ away.
(a) reports, withering
(b) stories, trading
(c) book, dying
(d) obituaries, fading
(CAT 2001)
45. The Darwin, who____ as the most remarkable for the way in which he ____ the attributes of the word class
thinker and head of the household.
(a) comes, figures
(b) arises, adds
(c) emerges, combines (d) appeared, combines
46. Indian intellectuals may boast if they are so inclined of being ____ to the most elitist among the intellectual ____ of
the world.
(a) subordinate, traditions
(b) heirs, cliques
(c) ancestors, societies
(d) heir, traditions

PRATHAM: BMS Special Booklet

43

47. The law prohibits a person from felling a sandal wood tree, even if it grows on ones own land without prior
permission from the government. As poor people cannot deal with the government this legal provision leads to a
rip-roaring business for____ who care neither for the____ nor for the trees.
(a) middlemen ... rich
(b) the government ... poor
(c) touts ... rich
(d) touts ... poor
(CAT 2000)
48. In these bleak and depressing time of____ prices, a non-performing government and____ crime rate, Sourav
Ganguly has given us Indians a lot to cheer about.
(a) escalating ... increasing
(b) spiraling ... booming
(c) spiraling ... soaring
(d) ascending ... debilitating
(CAT 2000)
49. The manners and ____ of the nouveau riche is a recurrent____ in the literature.
(CAT 2000)
(a) style ... motif
(b) morals ... story
(c) wealth ... theme
(d) morals ... theme
Directions for Question 50: Answer the question based on the following information.
Fill in the blanks of the following sentences using the most appropriate word from the given options.
50. When children become more experienced with words as visual symbols, they find that they can gain meaning
without making ____ sounds.
(a) aural
(b) audible
(c) vocal
(d) intelligible
(CAT 1998)

SENTENCE IMPROVEMENT:
Directions for Questions 51 - 55: Answer the questions based on the following information.
A part of each sentence given below has been underlined. You have to select the option that best replaces the underlined
part.
51. British Airspace has been focusing on building European links.
(a) concentrating on creating European links
(b) pursuing ways of building European connectivity
(c) stressing on building European links
(d) focusing on forging European links
(CAT 1998)
52. It must be noticed that under no circumstance should the company go in for diversification.
(a) It must be noticed
(b) it must be noted
(c) It must be pointed out
(d) It should be noticed
(CAT 1997)
53. Contemplating whether to exist with an insatiable romantic temperament, he was the author and largely the
subject of number of memorable novels.
(a) Contemplating whether to exist
(b) Combining realistic details
(c) Miscegenation a brilliant mind
(d) Aware that he had been born
(CAT 1996)
54. Victory is everything in the Indian universe and Tendulkar will be expected to translate his genius to that effect. To
contemplate any other option is to contemplate the risk of failure.
(a) To contemplate any other option is to contemplate the risk of failure.
(b) Failure is not an action that can be contemplated.
(c) Any other action has the potential of failure.
(d) Failure is not an option.
55. The MP rose up to say that in her option, she thought the Womens Reservation Bill should be passed on
unanimously.
(a) rose to say that she thought the Womens Reservation should be passed
(b) rose up to say that, the Womens Reservation Bill should be passed on
(c) rose to say that, in her option, she thought that the Womens Reservation Bill should be passed
(d) rose to say that, in her opinion, the Womens Reservation Bill be passed on
(CAT 1999)

PRATHAM: BMS Special Booklet

44

VERBAL ABILITY

SENTENCE CORRECTION:
Directions for Questions 56 - 68: In each of the following sentences, four/five options are given. You are required to
identify the best way of writing the sentence in the context of the correct usage of standard written English. While doing
so, you have to ensure that the message being conveyed remains the same in all the cases.
56. Poor product quality angers Mr. Garbole, who wonders if it is part of a strategy by marketers.
(a) Poor product quality angers Mr. Garbole, who wonders if it is part of a strategy by marketers.
(b) Poor product quality angers Mr. Garbole, who wonders if marketers are part of the strategy.
(c) Poor product quality angers Mr. Garbole, that wonders if it is part of a strategy by marketers.
(d) Poor product quality angers Mr. Garbole, who wonders if they are part of a strategy by marketers.
(e) Poor product quality angers Mr. Garbole, who wonders if it are part of a strategy.
57. The shopkeeper hadn't hardly any of those kind of goods.
(a) The shopkeeper hadn't hardly any of those kind of goods.
(b) The shopkeeper hadn't hardly any of those kinds of goods.
(c) The shopkeeper had hardly any of those kind of goods.
(d) The shopkeeper had hardly any of those kinds of goods.
(e) The shopkeeper had not hardly any of those kinds of goods.
58. Entertainment being recognised as an important factor in improving mental and physical health and thereby
reducing human misery and poverty.
(a) Entertainment being recognised as an important factor in improving mental and physical health and thereby
reducing human misery and poverty.
(b) Recognising entertainment as an important factor in improving mental and physical health and thereby reducing
human misery and poverty.
(c) Recognition of it being an important factor in improving mental and physical health entertainment reduces
human misery and poverty.
(d) Entertainment is recognised as an important factor in improving mental and physical health and thereby
reducing human misery and poverty.
(e) Entertainment while being recognised as an important factor in improving mental and physical health and
thereby reducing human misery and poverty.
59. The child is neither encouraged to be critical or to examine all the aspects of his opinion.
(a) The child is neither encouraged to be critical or to examine all the aspects of his opinion.
(b) The child is neither encouraged to be critical nor to examine all the aspects of his opinion.
(c) The child is either encouraged to be critical or to examine all the evidence for his opinion.
(d) The child is either encouraged to be critical not to examine all the aspects of his opinion.
(e) None of these
60. The panel interviewed several candidates who they thought had the experience and qualifications the position at
IIM Bangalore required.
(a) The panel interviewed several candidates who they thought had the experience and qualifications the position
at IIM Bangalore required.
(b) The panel interviewed several candidates whom they thought had the experience and qualifications the position
at IIM Bangalore required.
(c) The panel interviewed several candidates from whom they thought had the experience and qualifications the
position at IIM Bangalore required.
(d) The panel interviewed several candidates which he thought had the experience and qualifications the position at
IIM Bangalore required.

PRATHAM: BMS Special Booklet

45

61.

62.

63.

64.

65.

66.

(e) The panel interviews several candidates who they thought has the experience and qualifications the position at
IIM Bangalore required.
The trend toward a decrease in the working hours is already evident in the longer weekend given to employees in
many multinational organisations.
(a) The trend toward a decrease in the working hours is already evident in the longer weekend given to employees
in many multinational organisations.
(b) The trend toward a decrease in the working hours is all ready evident in the longer weekend given to
employees in many multinational organisations.
(c) The trend toward a decrease in the working hours is all ready evident in the longer weekend given to
employees in many multinational organisations.
(d) The trend toward a decrease in the working hours is all in already evident in the longer weekend given to
employees in many multinational organisations.
(e) The trend toward a decrease in working hours is already evidently in the longer weekend given to employers in
many multinational organisations.
Most students like to read these kind of books for using their spare time.
(a) Most students like to read these kind of books for using their spare time.
(b) Most students like to read these kind of book for using their spare time.
(c) Most students like to read this kind of book for using their spare time.
(d) Most students like to read this kind a book for using their spare time.
(e) Most students like to read this kind of book for using of their spare time.
Not only was he efficient but also welcoming in nature.
(a) He not only was competent but also friendly in nature.
(b) Not only was he competent but also friendly in nature.
(c) He not only was competent but friendly too in nature.
(d) He was not only competent but also friendly in nature.
(e) He, not only was competent but friendly also in nature.
Liberalisation has gone hand in hand and has offered incentives for such things as personal initiative, ambition,
loyalty, hard work, and resourcefulness.
(a) Liberalisation has gone hand in hand and has offered incentives for such things as personal initiative, ambition,
loyalty, hard work, and resourcefulness.
(b) Liberalisation has gone hand in hand with and has offered incentives for such things as personal initiative and
ambition, loyalty, hard work, and resourcefulness.
(c) Liberalisation has gone hand in hand with and has offered incentives for such as things as personal initiative and
ambition, loyalty, hard work, and resourcefulness.
(d) Liberalisation has gone hand in hand and is offering incentives for such things as personal initiative ambition,
loyalty, hard work, and resourcefulness.
(e) Liberalisation has gone hand in hand while and providing incentives for such things as personal initiative and
ambition, loyalty, hard work, and resourcefulness.
May I venture to say that 1 think this batting performance is the most superior I have ever seen.
(a) May I venture to say that I think this batting performance is the most superior 1 have ever seen?
(b) May I venture to say that this batting performance is the most superior I have ever seen?
(c) May I say that this batting performance is the most superior I have ever seen?
(d) I think this performance is superior to any I have ever seen.
(e) May I venture to say that this batting performance is more superior to anyone I have ever seen.
The matter was referred back to the expert committee since the solution to the problem was different from the
one proposed earlier.
(a) The matter referred back to the expert committee since the solution to the problem was different from the one
proposed earlier.

PRATHAM: BMS Special Booklet

46

VERBAL ABILITY
(b) The matter referred to the expert committee since the solution to the problem was different from the one
proposed earlier.
(c) The matter referred back to the expert committee since the solution to the problem was different than the
one proposed earlier.
(d) The matter referred to committee since the solution to the problem was different than the one proposed
earlier.
(e) The matter referred back to the committee since the solution for problem was different than one proposed
earlier.
67. The smaller firms in any industry sell either on a price or quality of workmanship basis.
(a) The smaller firms in any industry sell either on a price or quality-of-workmanship basis.
(b) The smaller firms in any industry either sell on a price or quality-of-workmanship basis.
(c) The smaller firms in any industry sell on either a price or a quality-of-workmanship basis.
(d) The smaller firms in any industry sell on either a price or on a quality-of-workmanship basis.
(e) The smaller firms of any industry sell off either on a price or quality-of-workmanship basis.
68. Dr. Pam Saxena, an Indian specialist in drug rehabilitation medicine, advises against going easy on rehabilitated drug
addicts for the fear of their relapsing into addiction.
(a) Dr. Pam Saxena, an Indian specialist in drug rehabilitation medicine, advises against going easy on rehabilitated
drug addicts for the fear of their relapsing into addiction.
(b) An Indian specialist in drug rehabilitation medicine, Dr. Pam Saxena, advises against going easy on rehabilitated
drug addicts for the fear of their relapsing into addiction.
(c) An Indian specialist in drug rehabilitation medicine, Pam Saxena, is of the opinion that one should not go easy on
rehabilitated drug addicts for the fear of their relapsing into addiction.
(d) Dr. Pam Saxena, an Indian specialist in drug rehabilitation medicine, advice against going easy on rehabilitated
drug addicts for the fear of their relapsing into addiction.
(e) Dr. Pam Saxena, an Indian specialist in drug rehabilitation medicines advises against easy going of rehabilitated
drug addicts for the fear of their relapsing into addiction.
Directions for Questions 69 and 70: Answer the questions based on the following information.
A part of each sentence given below has been underlined. You have to select the option that best replaces the underlined
part.
69. Many middle-class South Asians find that they cannot obtain good medical attention, despite the fact they need it
badly.
(a) they need it badly.
(b) they badly need it.
(c) of they need it badly.
(d) that they need it badly.
(e) they need it that badly.
70. Panchayati Raj institutions are now entrusted upon the execution of all rural upliftment schemes and programs in
India.
(a) entrusted upon the execution of all rural upliftment schemes and programs
(b) entrusted with the execution of all rural uplift schemes and programs
(c) entrusted with the execution of all rural upliftment schemes and programs
(d) entrusted within the execution for all rural uplift programs and schemes
(e) entrusted of the execution of all rural upliftment schemes and programs

PRATHAM: BMS Special Booklet

47

JUMBLED SENTENCES:
Directions for Questions 71 - 73: The sentences given in each question, when properly sequenced, form a coherent
paragraph. Each sentence is labelled with a letter. Choose the most logical order of sentences from among the given
choices to construct a coherent paragraph.
71. A. He felt justified in bypassing Congress altogether on a variety of moves.
B. At time he was fighting the entire Congress.
C. Bush felt he had a mission to restore power to the presidency.
D. Bush was not fighting just the democrats.
E. Representative democracy is a messy business, and a CEO of the White House does not like a legislature of
second guessers and the wasters.
(a) CAEDB
(b) DBAEC
(c) CEADB
(d) ECDBA
72. A. Call it the third wave sweeping the Indian media.
(CAT 2003)
B. Now, they are starring in a new role, as suave dealmakers who are in a hurry to strike alliances and agreements.
C. Look around and you will find a host of deals that have been inked or are ready to be finalized.
D. Then the media barons wrested back control for their editors and turned marketing warriors with the brand as
their missile.
E. The first came with those magnificent men in their mahogany chambers who took on the world with their
mighty fountain pens.
(a) ACBED
(b) CEBDA
(c) CAEBD
(d) AEDBC
73. A. Passivity is not, of course, universal.
(CAT 2001)
B. In areas where there are no lords or laws, or in frontier zones where all men go armed, the attitude of the
peasantry may well be different.
C. So indeed it may be on the fringe of the unsubmissive.
D. However, for most of the soil-bound peasants, the problem is not whether to be normally passive or active, but
when to pass from one state to another.
E. This depends on an assessment of the political situation.
(a) BEDAC
(b) CDABE
(c) EDBAC
(d) ABCDE
Directions for Questions 74 and 75: Sentences given in each question, when properly sequenced, form a coherent
paragraph. The first and last sentences are 1 and 6, and the four in between are labelled A, B, C and D. Choose the most
logical order of these four sentences from among the four given choices to construct a coherent paragraph from
sentences 1 to 6.
74. 1. Security inks exploit the same principle that causes the vivid and constantly changing colours of a film of oil on
water.
A. When two rays of light meet each other after being reflected from these different surfaces, they have each
travelled slightly different distances.
B. The key is that the light is bouncing of two surfaces, that of the oil and that of the water layer below it.
C. The distance the two rays travel determines which wavelengths, and hence colours, interfere constructively and
look bright.
D. Because lights, an electromagnetic wave, the peaks and troughs of each ray then interfere either constructively,
to appear bright, or destructively, to appear dim.
6. Since the distance the rays travel changes with the angle as you look at the surface, different colours look bright
from different viewing angles.
(a) ABCD
(b) BADC
(c) BDAC
(d) DCAB
(CAT 2000)

PRATHAM: BMS Special Booklet

48

VERBAL ABILITY
75. 1. Buddhism is a way to salvation.
A. But Buddhism is more severely analytical.
B. In the Christian tradition, there is also a concern for the fate of human society conceived as a whole, rather than
merely as a sum or network of individuals.
C. Salvation is a property, or achievement of individuals.
D. Not only does it dissolve society into individuals, the individual in turn is dissolved into component parts and
instants a stream of events.
6. In modern terminology, Buddhist doctrine is reductionist.
(a) ABDC
(b) CBAD
(c) BDAC
(d) ABCD
(CAT 1998)

READING COMPREHENSION:
Passage I:

(CAT 2004)

Throughout human history the leading causes of death have been infection and trauma. Modern medicine has scored
significant victories against both, and the major causes of ill health and death are now the chronic degenerative diseases,
such as coronary artery disease, arthritis, osteoporosis, Alzheimer's, macular degeneration, cataract and cancer. These
have a long latency period before symptoms appear and a diagnosis is made. It follows that the majority of apparently
healthy people are pre-ill.
But are these conditions inevitably degenerative? A truly preventive medicine that focused on the pre-ill, analysing the
metabolic errors which lead to clinical illness, might be able to correct them before the first symptom. Genetic risk
factors are known for all the chronic degenerative diseases, and are important to the individuals who possess them. At
the population level, however, migration studies confirm that these illnesses are linked for the most part to lifestyle
factorsexercise, smoking and nutrition. Nutrition is the easiest of these to change, and the most versatile tool for
affecting the metabolic changes needed to tilt the balance away from disease.
Many national surveys reveal the malnutrition is common in developed countries. This is not the calorie and/or
micronutrient deficiency associated with developing nations (Type A-malnutrition); but multiple micronutrient depletion,
usually combined with calorific balance or excess (Type B-malnutrition). The incidence and severity of Type Bmalnutrition will be shown to be worse if newer micronutrient groups such as the essential fatty acids, xanthophylls and
flavonoids are included in the surveys. Commonly ingested levels of these micronutrients seem to be far too low in many
developed countries.
There is now considerable evidence that Type B-malnutrition is a major cause of chronic degenerative disease. If this is
the case, then it is logical to treat such disease not with drugs but with multiple micronutrient repletion, or 'pharmacynutrition'. This can take the form of pills and capsules'nutraceuticals', or food formats known as 'functional foods'. This
approach has been neglected hitherto because it is relatively unprofitable for drug companiesthe products are hard to
patentand it is strategy which does not sit easily with modern medical interventionism. Over the last 100 years, the
drug industry has invested huge sums in developing a range of subtle and powerful drugs to treat the many diseases we
are subject to. Medical training is couched in pharmaceutical terms and this approach has provided us with an exceptional
range of therapecutic tools in the treatment of disease and in acute medical emergencies. However, the pharmaceutical
model has also created an unhealthy dependency culture, in which relatively few of us accept responsibility for
maintaining our own health. Instead, we have handed over this responsibility to health professionals who know very little
about health maintenance, or disease prevention.
One problem for supporters of this arguments lack of the right kind of hard evidence. We have a wealth of
epidemiological data linking dietary factors to health profiles/disease risks, and a great deal of information on mechanism:
how food factors interact with our biochemistry. But almost all intervention studies with micronutrients, with the notable
exception of the omega 3 fatty acids, have so far produced conflicting or negative results. In other words, our science
appears to have no predictive value. Does that invalidate the science? Or are we simply asking the wrong questions?

PRATHAM: BMS Special Booklet

49

Based on pharmaceutical thinking, most intervention studies have attempted to measure the impact of a single
micronutrient on the incidence of disease. The classical approach says that if you give a compound formula to test
subjects and obtain positive results, you cannot know which ingredient is exerting the benefit, so you must test each
ingredient individually. But in the field of nutrition, this does not work. Each intervention on its own will hardly make
enough difference to be measured. The best therapeutic response must therefore combine micronutrients to normalize
our internal physiology. So do we need to analyse each individual's nutritional status and then tailor a formula specifically
for him or her? While we do not have the resources to analyse millions of individual cases, there is no need to do so. The
vast majority of people are consuming suboptimal amounts of most micronutrients, and most of the micronutrients
concerned are very safe. Accordingly, a comprehensive and universal program of micronutrient support is probably the
most cost- effective and safest way of improving the general health of the nation.
76. Tailoring micronutrient-based treatment plans to suit individual deficiency profiles is not necessary because :
(a) it is very likely to give inconsistent or negative results.
(b) it is a classic pharmaceutical approach not suited to micronutrients.
(c) most people are consuming suboptimal amounts of safe-to-consume micronutrients.
(d) it is not cost effective to do so.
77. The author recommends micronutrient-repletion for large-scale treatment of chronic degenerative diseases
because:
(a) it is relatively easy to manage.
(b) micronutrient deficiency is the cause of these diseases.
(c) it can overcome genetic risk factors.
(d) it can compensate for other lifestyle factors.
78. Why is a large number of apparently healthy people deemed pre-ill ?
(a) They may have chronic degenerative diseases.
(b) They do not know their own genetic risk factors which predispose them to diseases.
(c) They suffer from Type B-malnutrition.
(d) There is lengthy latency period associated with chronically degenerative diseases.
79. Type B-malnutrition is a serious concern in developed countries because :
(a) developing countries mainly suffer from Type A-malnutrition.
(b) it is a major contributor to illness and death.
(c) pharmaceutical companies are not producing drugs to treat this condition.
(d) national surveys on malnutrition do not include newer micronutrient groups.

Passage II:

(CAT 2002)
The conceptions of life and the world which we call 'philosophical' are a product of two factors: one inherited, religious
and ethical conceptions; the other, the sort of investigation which may be called 'scientific', using this word in this
broadest sense. Individual philosophers have differed widely in regard to the proportions in which these two factors
entered into their systems, but it is the presence of both in some degree that characterizes philosophy.
'Philosophy is a word which has been used in many ways, some wider, some narrower. I propose to use it in a very wide
sense, which I will now try to explain.
Philosophy, as I shall understand the word, is something intermediate between theology and science. Like theology it
consists of speculations on matters as to which definite knowledge has, so far, been unascertainable; but like science, it
appeals to human reason rather than to authority, whether that of tradition or that of revelation. All definite knowledge
so I should contendbelongs to science; all dogma as to what surpass definite knowledge to theology. But between
theology and science there is a 'No man's Land' exposed to attack from both sides; this 'No Man's Land' is philosophy.
Almost all the questions of most interest to speculative minds are such as science cannot answer, and the confident
answers of theologians no longer seem so convincing as they did in former centuries. Is the world divided into mind and
matter, and if so, what is mind and what is matter ? Is mind subject matter, or is it possessed of independent powers ?

PRATHAM: BMS Special Booklet

50

VERBAL ABILITY
Has the universe any unity or purpose ? Is it evolving towards some goal ? Are there really laws of nature, or do we
believe in them only because of our innate love of order ? Is man what he seems to the astronomer, a tiny lump of carbon
and water impotently crawling on a small and unimportant planet ? Or is he what he appears to Hamlet ? Is he perhaps
both at once ? Is there a way of living that is noble and another that is base, or are all ways of living merely futile ? If there
is a way of living that is noble, in what does it consist, and how shall we achieve it ? Must the good be eternal in order to
deserve to be valued, or is it worth seeking even if the universe is inexorably moving towards death ? Is there such a thing
as wisdom or is what seems such merely the ultimate refinement of folly ? To such questions no answer can be found in
the laboratory.
Theologies have professed to given answers, all to definite; but their definiteness causes modern minds to view them with
suspicion. The studying of these questions, if not the answering of them, is the business of philosophy.
Why, then you may ask, waste time on such insoluble problems ? To this one may answer as a historian, or as an
individual facing the terror of cosmic loneliness.
The answer of the historian, in so far as I am capable of giving it, will appear in the course of this work. Ever since men
became capable of free speculation, their actions in innumerable important respects, have depended upon their theories
as to the world and human life, as to what is good and what is evil. This is as true in the present day at any former time.
To understand an age or a nation we must understand its philosophy, and to understand its philosophy we must ourselves
be in some degree philosophers. There is here a reciprocal causation: the circumstances of men's lives do much to
determine their philosophy, but, conversely, their philosophy does much to determine their circumstances.
There is also, however, a more personal answer. Science tells us what we can know, but what we can know is little, and
if we forget how much we cannot know we may become insensitive to many things of very great importance. Theology,
on the other hand, induces a dogmatic belief that we have knowledge, where in fact we have ignorance, and by doing so
generated a kind of impertinent insolence towards the universe. Uncertainty, in the presence of vivid hopes and fears, is
painful, but must be endured if we wish to live without the support of comforting fair tales. It is not good either to forget
the questions that philosophy asks, or to persuade ourselves that we have found indubitable answers to them. To teach
how to live without certainty, and yet without being paralyzed by hesitation, is perhaps the chief things that philosophy, in
our age, can still do for those who study it.
80. The purpose of philosophy is to :
(a) reduce uncertainty and chaos.
(b) help us to cope with uncertainty and ambiguity.
(c) help us to find explanations for uncertainty.
(d) reduce the terror of cosmic loneliness.
81. Based on this passage what can be concluded about the relation between philosophy and science ?
(a) The two are antagonistic.
(b) The two are complimentary.
(c) There is no relation between the two.
(d) Philosophy derives from science.
82. From reading the passage, what can be concluded about the profession of the author ? He is most likely NOT to be
a:
(a) Historian
(b) Philosopher
(c) Scientist
(d) Theologian
83. According to the author, which of the following statements about the nature of the universe must be definitely true
?
(a) The universe has unity.
(b) The universe has a purpose.
(c) The universe is evolving towards a goal.
(d) None of these.

Passage III:

(CAT 2000)

The teaching and transmission of North Indian classical music is, and long has been achieved by largely oral means. The
raga and its structure, the often breathtaking intricacies of tala or rhythm, and the incarnation of rage and tala as bandish
or composition, are passed thus, between guru and shishya by word of mouth and direct demonstration, with no printed
sheet of notated music, as it were acting as a go-between. Saussure's conception of language as a communication
between addresser and addressee is given, in this model, a further instance, and a new, exotic complexity and glamour.

PRATHAM: BMS Special Booklet

51

These days, especially with the middle-class having entered the domain of classical music and playing not a small part in
ensuring the continuation of this ancient tradition, the tape recorder serves as a handy technological slave and preserves,
from oblivion, the vanishing, elusive moment of oral transmission. Hoary gurus, too, have seen the advantage of this
device, and increasingly use it as an aid to instruct their pupils; in place of the shawls and other traditional objects that
used to pass from shishya to guru in the past, as a token of the regard of the former for the latter, it is not unusual, today,
to see cassettes changing hands.
Part of my education in North Indian classical music was conducted via this rather ugly but beneficial rectangle of plastic,
which I carried with me to England when I was an undergraduate. One cassette had stored in it various talas played upon
the tabla, at various tempos, by my music teacher's brother in-law, Hazarilalji, who was a teacher of Kathak dance, as well
as a singer and a tabla player. This was a work of great patience and prescience, a one and half hours performance
without any immediate point or purpose, but intended for some delayed future moment when I'd practise the talas
solitarily.
This repeated playing out of the rhythmic cycles on the tabla was inflected by the noisesan irate auto driver blowing a
horn; the sound of overbearing pigeons that were such a nuisance on the banister; even the cry of a kulfi seller in
summerentering from the balcony of the third floor flat we occupied in those days, in a lane in a Mumbai suburb,
before we left the city for good. These sounds, in turn, would invade, hesitantly, the ebb and flow of silence inside the
artificially heated room, in a borough of West London in which I used to live as an undergraduate. There, in the trapped
dust, silence and heat, the theka of the tabla, qualified by the imminent but intermittent presence of the Mumbai suburb,
would come to life again. A few years later, the tabla and, in the background, the pigeons and the itinerant kulfi seller,
would inhabit a small graduate room in Oxford.
The tape recorder though remains an extension of the oral transmission of music, rather than a replacement of it. And
the oral transmission of North Indian classical music remains, almost uniquely, a testament to the fact that the human
brain can absorb, remember and reproduce structures of great complexity and sophistication without the help of the
hieroglyph or written mark or a system of notation. I remember my surprise on discovering that Hazarilaljiwho had
mastered Kathak dance, tala and North Indian classical music, and who used to narrate to me, occasionally, composition
meant for dance that were grand and intricate in their verbal prosody, architecture and rhythmic complexitywas near
illiterate and had barely learnt to write his name in large and clumsy letters.
Of course, attempts have been made, throughout the 20th century, to formally codify and even notate this music, and
institutions set up and degrees created, specifically to educate students in this 'scientific' and codified manner.
Paradoxically, however, this style of teaching has produced no noteworthy student or performer, the most creative
musicians still emerge from the guru-shishya relationship, their understanding of music developed by oral communication.
The fact that North Indian classical music emanates from, and has evolved through, oral culture, means that this music has
a significantly different aesthetic, and that this aesthetic has a different politics, from that of Western classical music. A
piece of music in the Western tradition, at least in its most characteristic and popular conception, originates in its
composer, and the connection between the two, between composer and the piece of music, is relatively unambiguous
precisely because the composer writes down, in notation, his composition, as a poet might write down and publish his
poem. However far the printed sheet of notated music might travel thus from the composer, it still remains his property;
and the notion of property remains at the heart of the Western conception of 'genius', which derives from the Latin
gignere or 'to beget'.
The genius in Western classical music is, then, the originator, begetter and owner of his workthe printed, notated sheet
testifying to his authority over his product and his power, not only for expression or imagination, but of origination. The
conductor is a custodian and guardian of this property. Is it an accident that Mandelstam, in his note-books, compares the
conductor's baton to a policeman's, saying all the music of the orchestra lies mute within it, waiting for its first movement
to release it into the auditorium ?
The ragatransmitted through oral means is, in a sense, no one's property; it is not easy to pin down its source, or to
know exactly where its provenance or origin lies. Unlike the Western classical tradition, where the composer begets his
piece, notates it and stamps it with his ownership and remains in effect larger than or the father of his work in the North
Indian classical tradition, the raga unconfined to a single incarnation, composer or performerremains necessarily greater
than the artists who invokes it.

PRATHAM: BMS Special Booklet

52

VERBAL ABILITY
This leads to a very different politics of interpretation and valuation to an aesthetic that privileges the evanescent moment
of performance and invocation over the controlling authority of genius and the permanent record. It is a tradition thus
that would appear to value the performer as medium, more highly than the composer who presumes to originate what
effectively, cannot be originated in a single personbecause the raga is the inheritance of a culture.
84. The author's contention that the notion of property lies at the heart of the Western conception of genius is best
indicated by which one of the following ?
(a) The creative output of a genius is invariably written down and recorded.
(b) The link between the creator and his output is unambiguous.
(c) The word 'genius' is derived from a Latin word which means 'to beget'.
(d) The music composer notates his music and thus becomes the 'father' of a particular piece of music.
85. Saussure's conception of language as a communication between addresser and addressee according to the author is
exemplified by the :
(a) teaching of North Indian classical music by word of mouth and direct demonstration.
(b) use of the recorded cassette as a transmission medium between the music teacher and the trainee.
(c) written down notation sheets of musical compositions.
(d) conductor's baton and the orchestra.
86. The author holds that the 'rather ugly but a beneficial rectangle of plastic, has proved to be a 'handy technological
slave' in :
(a) storing the talas played upon the table at various tempos.
(b) ensuring the continuance of an ancient tradition.
(c) transporting North Indian classical music across geographical borders.
(d) capturing the transient moment of oral transmission.
87. The oral transmission of North Indian classified music is an almost unique testament of the :
(a) efficacy of the guru-shishya tradition.
(b) learning impact of direct demonstration.
(c) brain's ability to reproduce complex structures without the help of written marks.
(d) the ability of an illiterate person to narrate grand and intricate musical compositions.
88. According to the passage in the North Indian classical tradition the raga remains greater than the artist who invokes
it. This implies an aesthetic which :
(a) emphasises performance and invocation over the authority of genius and permanent record.
(b) makes the music no one's property.
(c) values the composer more highly than the performer.
(d) supports oral transmission of traditional music.
89. Which one of the following cannot be inferred ?
(a) It is easy to transfer a piece of Western classical music to a distant place.
(b) The conductor in the Western tradition as a custodian can modify the music since it 'lies mute' in his baton.
(c) The authority of the Western classical music composer over his music product is unambiguous.
(d) The power of the Western classical music composer extends to the expression of his music.
90. According to the author the inadequacy of teaching North Indian classical music through a codified, notation based
system is best illustrated by :
(a) a loss of the structural beauty of the ragas.
(b) a fusion of two opposing approaches creating mundance music.
(c) the conversion of free-flowing ragas into stilled set pieces.
(d) its failure to produce any noteworthy student or performer.
91. Which of the following statements best conveys the overall idea of the passage ?
(a) North Indian and Western classical music are structurally different.
(b) Western music is the intellectual property of the genius while the North Indian raga is the inheritance of the
culture.

PRATHAM: BMS Special Booklet

53

(c) Creation as well as performance are important in the North Indian classical tradition.
(d) North Indian classical music is orally transmitted while Western classical music depends on written down
notations.

Passage IV:
(CAT 2000)
The story begins as the European pioneers crossed the Alleghenies and started to settle in the Midwest. The land they
found was covered with forests. With incredible effort they felled the trees, pulled the stumps and planted their crops in
the rich, loamy soil. When they finally reached the western edge of the place, we now call Indiana, the forest stopped and
ahead lay thousand miles of the great grass prairie. The Europeans were puzzled by this new environment. Some even
called it the 'Great Desert'. It seemed untellable. The earth was often very wet and it was covered with centuries of
tangled and matted grasses. With their cast iron plows, the settlers found that the prairie sod could not be cut and the
wet earth stuck to their plowshares. Even a term of the best oxen bogged down after a few years of tugging. The iron
plow was a useless tool to farm the prairie soil. The pioneers were stymied for nearly two decades. Their western march
was halted and they filled in the eastern regions of the Midwest.
In 1837, a blacksmith in the town of Grand Detour, Illinois, invented a new tool. His name was John Deere and tool was a
plow made of steel. It was sharp enough to cut through matted grasses and smooth enough to cast off the mud. It was a
simple tool, the 'sod buster' that opened the great prairies to agricultural development.
Sauk County, Wisconsin is the part of the prairie where 1 have a home. It is named after the Sauk Indians. In 1673, Father
Marquette was the first European to lay his eyes upon their land. He found a village laid out in regular patterns on a plain
beside the Wisconsin River. He called the place Prairie du Sac. The village was surrounded by fields that had provided
maize, beans and squash for the Sauk people for generations reaching back into the unrecorded time.
When the European settlers arrived at the Sauk prairie in 1837, the government forced the native Sauk people, West of
the Mississippi River. The settlers came with John Deere's new invention and used the tool to open the area to a new
kind of agriculture. They ignored the traditional ways of the Sauk Indians and used their sod-busting tool for planting
wheat. Initially, the soil was generous and the farmers thrived. However, each year the soil lost more of its nurturing
power. It was only 30 years after the Europeans arrived with their new technology that the land was depleted. Wheat
farming became uneconomic and tens of the thousands of farmers left Wisconsin seeking new land with sod to bust.
It took the Europeans and their new technology just one generation to make their homeland into a desert. The Sauk
Indians who knew how to sustain themselves on the Sauk prairie land were banished to another kind of desert called a
reservation. And they even forgot about the techniques and tools that had sustained them on the prairie for generations
unrecorded. And that is how it was that three deserts were createdWisconsin, the reservation and the memories of
people. A century later, the land of the Sauks in now populated by the children of a second wave of European farmers
who learned to replenish the soil through the regenerative powers of dairying, ground cover crops and animal manures.
These third and fourth generation farmers and townspeople do not realise, however, that a new settler is coming soon
with an invention as powerful as John Deer's plow.
The new technology is called 'bereavement counselling'. It is a tool forged at the great state university, an innovative
technique to meet the needs of those experiencing the death of a loved one, a tool that can 'process' the grief of the
people who now live on the Prairie of the Sauk. As one can imagine the final days of the village of the Sauk Indians before
the arrival of the settlers with John Deere's plow, one can also imagine these final days before the arrival of the settlers
with John Deere's plow, one can also imagine these final days before the arrival the first bereavement counsellor at Prairie
du Sac. In these final days, the farmers and the townspeople mourn at the death of a mother, brother, son, or friend. The
bereaved is joined by neighbours and kin. They meet grief together in lamentaion, prayer and song. They call upon the
words of the clergy and surround themselves in community.
It is in these ways that they grieve and then go on with life. Through their mourning they are assured of the bonds
between them and renewed in the knowledge that this death is a part of the Prairie of the Sauk. Their grief is common
property, an anguish from which the community draws strength and gives the bereaved the courage to move ahead.
It is into this parairie community that the bereavement counsellor arrives with the new grief technology. The counsellor
calls the invention a service and assures the prairie folk of its effectiveness and superiority by invoking the name of the

PRATHAM: BMS Special Booklet

54

VERBAL ABILITY
great university while displaying a diploma and certificate. At first, we can imagine that the local people will be puzzled by
the bereavement counsellor's claim. However, the counsellor will tell a few of them that the new technique is merely to
assist the bereaved's community at the time of death. To some other prairie folk who are isolated or forgotten, the
counsellor will approach the Country Board and advocate the right to treatment for these unfortunate souls. This right
will be guaranteed by the Board's decision to reimburse those too poor to pay for counselling services. There will be
others, schooled to believe in the innovative new tools certified by universities and medical centres, who will seek out the
bereavement counsellor by force of habit. And one of these people will tell a bereaved neighbour who is unschooled that
unless his grief is processed by a counsellor, he will probably have major psychological problems in later life. Several
people will begin to use the bereavement counsellor because, since the Country Board now taxes them to insure access
to the technology, they will feel that to fail to be counselled is to waste their money, and to be denied a benefit, or even a
right. Finally, one day, the aged father of a Sauk woman will die. And the next door neighbour will not drop by because he
doesn't want to interrupt the bereavement counsellor. The woman's kin will stay home because they will have learned
that only the bereavement counsellor knows how to process grief the proper way. The local clergy will seek technical
assistance from the bereavement counsellor to learn the correct form of service to deal with guilt and grief. And the
grieving daughter will know that it is the bereavement counsellor who really cares for her because only the bereavement
counsellor comes when death visits this family on the Prairie of the Sauk.
It will be only one generation between the bereavement counsellor arrives and the community of mourners disappears.
The ounsellor's new tool will cut through the social fabric throwing aside kinship care neighborly obligations and
community ways of coming together and going on. Like John Deere's plow the tools of bereavement counselling will
create a desert where a community once flourished. And finally even the bereavement counsellor will see the
impossibility of restoring hope in clients once they are genuinely alone with nothing but a service for consolation. In the
inevitable failure of the service the bereavement counsellor will find the deserts even in herself.
92. Which one of the following best describes the approach of the author ?
(a) Comparing experiences with two innovations tried in order to illustrate the failure of both.
(b) Presenting community perspectives on two technologies which have had negative effects on people.
(c) Using the negative outcomes of one innovation to illustrate the likely outcomes of another innovation.
(d) Contrasting two contexts separated in time to illustrate how 'deserts' have arisen.
93. According to the passage bereavement handling traditionally involves :
(a) the community bereavement counsellors working the bereaved to help her overcome grief.
(b) the neighbours and kin joining the bereaved and meeting grief together in mourning and prayer.
(c) using techniques developed systematically in formal institutions of learning, a trained counsellor helping the
bereaved cope with grief.
(d) the Sauk Indian Chief leading the community with rituals and rites to help lessen the grief of the bereaved.
94. Due to which of the following reasons, according to the author, will the bereavement counsellor find the deserts
even in herself ?
(a) Over a period of time working with Sauk Indians who have lost their kinship and relationships she becomes one
of them.
(b) She is working in an environment where the disappearance of community mourners makes her work place a
social desert.
(c) Her efforts at grief processing with the bereaved will fail as no amount of professional service can make up for
the loss due to the disappearance of community mourners.
(d) She has been working with people who have settled for a long time in the Great Desert.
95. According to the author, the bereavement counsellor is :
(a) a friend of the bereaved helping him or her handle grief.
(b) an advocate of the right treatment for the community.
(c) a kin of the bereaved helping him/her handle grief.
(d) a formally trained person helping the bereaved handle grief.

PRATHAM: BMS Special Booklet

55

96. The prairie was a great puzzlement for the European pioneers because :
(a) it was covered with thick, untellable layers of grass over a vast stretch.
(b) it was a large desert immediately next to lush forests.
(c) it was rich cultivable land left fallow for centuries.
(d) it could be easily tilled with iron plows.
97. Which of the following does the 'desert' in the passage refer to ?
(a) Prairie soil depleted by cultivation wheat.
(b) Reservations in which native Indians were resettled.
(c) Absence of, and emptiness in, community kinship and relationships.
(d) All of the above.
98. According to the author, people will begin to utilise the service of the bereavement counsellor because :
(a) new Country regulations will make them feel it is a right, and if they don't use it, it would be a loss.
(b) the bereaved in the community would find her a helpful friend.
(c) she will fight for subsistence allowance from the Country Board for the poor among the bereaved.
(d) grief processing needs tools certified by universities and medical centres.
99. Which of the following parallels between the plow and bereavement counselling is not claimed by the author ?
(a) Both are innovative technologies.
(b) Both result in migration of the communities into which the innovations are introduced.
(c) Both lead to 'deserts' in the space of only one generation.
(d) Both are tools introduced by outsiders entering existing communities.

Passage V:

(CAT 1999)
Have you ever come across a painting, by Picasso, Mondrian, Miro, or any other modern abstract painter of this century,
and found yourself engulfed in a brightly coloured canvas which your senses cannot interpret ? Many people would tend
to denounce abstractionism as senseless trash. These people are disoriented by Miro's bright fanciful creatures and twodimensional canvases. They click their tongues and shake their heads at Mondrian's grid works, declaring that the poor
guy played too many scrabble games. They silently shake their heads in sympathy for Picasso, whose gruesome, distorted
figures must be a reflection of his mental health. Then, standing in front of a work by Charlie Russel, the famous Western
artist, they'll declare it a work of God. People feel more comfortable with something they can relate to and understand
immediately without too much thought. This is the case with the work of Charlie Russell. Being able to recognise the
elements in his paintingstrees, horses and cowboysgives people a safety line to their world of 'reality'. There are
some who would disagree when I say abstract art requires more creativity and artistic talent to produce a good piece
than does representational art, but there are many weaknesses in their arguments.
People who look down on abstract art have several major arguments to support their beliefs. They feel that artists turn
abstract because they are not capable of the technical drafting skills that appear in a Russell: therefore, such artists create
an art form that anyone is capable of and that is less time consuming, and then parade it as artistic progress. Secondly,
they feel that the purpose of art is to create something of beauty in an orderly, logical composition. Russell's compositions
are balanced and rational: everything sits calmly on the canvas, leaving the viewer satisfied that he has seen all there is to
see. The modern abstractionists, on the other hand, seem to compose their pieces irrationally. For example, upon seeing
Picasso's Guernica, a friend of mine asked me, "What's the point ?" Finally, many people feel that art should portray the
idea and real. The exactness of detail in Charlie's Russell's work is an example of this. He has been called a great historian
because his pieces depict the life style, dress, and events of the times. His subject matter is derived from his own
experiences on the trial, and reproduced to the smallest detail.
I agree in part with many of these arguments, and at one time even endorsed them. But now, I believe differently. Firstly,
I object to the argument that abstract artists are not capable of drafting.
Many abstract artists, such as Picasso, are excellent draftsmen. As his work matured, Picasso became more
abstract in order to increase the expressive quality of his work. Guernica was meant as a protest against the
bombings of that city by the Germans. To express the terror and suffering of the victims more vividly, he

PRATHAM: BMS Special Booklet

56

VERBAL ABILITY
distorted the figures and presented them in a black and white journalistic manner. If he had used representational
images and colour, much of the emotional content would have been lost and the piece would not have caused
the demand for justice that it did. Secondly, I do not think that a piece must be logical and aesthetically pleasing to
be art. The message it conveys to its viewers is more important. It should reflect the ideals and issues of its time
and be true to itself, not just a flowery, glossy surface. For example, through his work, Mondrian was trying to
present a system of simplicity, logic, and rational order. As a result, his pieces did end up looking like a scrabble
board.
Miro created powerful surrealistic images from his dream and subconscious. These artists were trying to evoke a
response from society through an expressionistic manner. Finally, abstract artists and representational artists maintain
different ideas about reality.' To the representational artist, reality is what he sees with his eyes. This is the reality he
reproduced on canvas. To the abstract artist, reality is what he feel about what his eyes see. This is the reality he
interprets on canvas. This can be illustrated by Mondrian's Trees series. You can actually see the progression from the
early recognizable, though abstracted, Trees, to his final solution, the grid system.
A cycle of abstract and representational art began with the first scratching of prehistoric man. From the abstractions of
ancient Egypt to representational, classical Rome, returning to abstractionism in early Christian art and so on up to the
present day, the cycle has been going on. But this day and age may witness its death through the camera. With film there
is no need to produce finely detail, historical records manually; the camera does this for us more efficiently. May be ,
representational art would cease to exist. With abstractionism as the victor of the first battle may be a different kind of
cycle will be touched off. Possibly, sometime in the distant future, thousands of years from now, art itself will physically
non-existent. Some artists today believe that once they have planned and constructed a piece in their mind, there is no
sense in finishing it with their hands; it has already been done and can never be duplicated.
100. The author argues that many people look down upon abstract art because they feel that :
(a) modern abstract art does not portray what is ideal and real.
(b) abstract artists are unskilled in matters of technical drafting.
(c) abstractionist compose irrationally.
(d) All of the above.
101. The author believes that people feel comfortable with representational art because :
(a) they are not engulfed in brightly coloured canvases.
(b) they do not have to click their tongues and shake their heads in sympathy.
(c) they understand the art without putting too much strain on their minds.
(d) painting like Guernica do not have a point.
102. In the author's opinion, Picasso's Guernica created a strong demand for justice since :
(a) it was a protest against the German bombing Guernica.
(b) Picasso managed to express the emotional content well with his abstract depiction.
(c) it depicts the terror and suffering of the victims in a distorted manner.
(d) it was a mature work of Picasso, painted when the artist's drafting skills were excellent.
103. The author acknowledges that Mondrian's pieces may have ended up looking like a scrabble board because :
(a) many people declared the poor guy played too many scrabble games.
(b) Mondrian believed in the 'grid-works' approach to abstractionist painting.
(c) Mondrian was trying to convey the message of simplicity and rational order.
(d) Mondrian learned from his Tree series to evolve a grid system.
104. The main difference between the abstract artist and the representational artist in matter of the 'ideal' and the 'real',
according to the author is :
(a) how each chooses to deal with 'reality' on his or her canvas.
(b) the superiority of interpretation of reality over production of reality.
(c) the different values attached by each to being a historian.
(d) the varying levels of drafting skills and logical thinking abilities.

PRATHAM: BMS Special Booklet

57

Passage VI:

(CAT 1998)
The persistent patterns in the way nations fight reflect their cultural and historical traditions and deeply rooted attitudes
that collectively make up their strange culture. These patterns provide insights that go beyond what can be learnt just by
comparing armaments and divisions. In the Vietnam War, the strategic tradition of the United States called for forcing the
enemy to fight a massed battle in an open area, where superior American weapons would prevail. The United States was
trying to re-fight Second World War in the jungles of South-East Asia, against an enemy with no intention of doing so.
Some British historians describe the Asian way of war as one of indirect attacks, avoiding frontal attacks meant to
overpower an opponent. This traces back to Asian history and geography : the great distances and harsh terrain have
often made it difficult to execute the sort of open field clashes allowed by the flat terrain and relatively compact size of
Europe. A very different strategic tradition arose in Asia.
The bow and arrow were metaphors for an Eastern way of war. By its nature, the arrow is an indirect weapon. Fired
from a distance of hundreds of yards, it does not necessitate immediate physical contact with the enemy. Thus, it can be
fired from hidden positions. When fired from behind a ridge, the barrage seems to come out of nowhere, taking the
enemy by surprise. The tradition of this kind of fighting is captured in the classical strategic writing of the East. The 2,000
years worth of Chinese writings on war constitutes the most subtle writing on the subject in any language. Now until
Clausewitz, did the West produce a strategic theorist to match the sophistication of Sun-tzu, whose Arc of War was
written 2,300 years earlier.
In Sun-tzu and other Chinese writings, the highest achievement of arms is to defeat an adversary without fighting. He
wrote: "To win one hundred victories in one hundred battles is not the acme of skill. To subdue the enemy without
fighting is the supreme excellence." Actual combat is just one among many means towards the goal of subduing an
adversary. War contains too many surprise to be a first resort. It can lead to ruinous losses, as had been seen time and
again, it can have the unwanted effect of inspiring heroic efforts in an enemy, as the United States learned in Vietnam, and
as the Japanese found out after Pearl Harbour.
Aware of the uncertainties of a military campaign, Sun-tzu advocate war only after the most thorough preparations. Even
then it should be quick and clean. Ideally, the army is just an instrument to deal the final blow to an enemy already
weakened by isolation, poor morale, and disunity. Ever since Sun-tzu, the Chinese have been seen as masters of subtlety
who take measured actions to manipulate an adversary without his knowledge. The dividing line between war and peace
can be obscure. Low level violence often is the backdrop to a larger strategic campaign. The unwitting victim, focused on
the day-to-day events, never realizes what's happening to him until it's too late. History holds many examples. The Viet
Cong lured French and US infantry deep into the jungle, weakening their morale over several years. The mobile army of
the United States was designed to fight on the plains of Europe, where it could quickly move unhindered from one spot
to the next. The jungle did more than make quick movement impossible; broken down into smaller units and scattered in
isolated bases. US forces were deprived of the feeling of support and protection that ordinarily comes from being part of
big army.
The isolation US troops in Vietnam was not just a logistical detail, something that could be overcome by, for instance,
bringing in reinforcements by helicopter. In a big army reinforcements are from being part of a larger formation. Just the
knowledge of its lowers the soldier's fear and increase his aggresiveness. In the jungle and on isolated bases, this feeling
was removed. The thick vegetation slowed down the reinforcements and made it difficult to find stranded units. Soldiers
felt they were on their own.
More important, by altering the way the war was fought, the Viet Cong stripped the United States of its belief in the
inevitabilility of victory, as it had done to the French before them. Morale was high when these armies first went to
Vietnam, only after many years of debilitating and demoralising fighting did Hanoi launch its decisive attacks, at Dien Bien
Phu in 1954 and against Saigon in 1975. It should be recalled that in the final push to victory the North Vietnamese
abandoned their jungle guerrilla tactics completely, committing their entire army of twenty divisions to pushing the South
Vietnamese into collapse. This final battle, with the enemy's army all in one place, was the one that the United States had
desperately wanted to fight in 1965. When it did come out into the open in 1975, Washington had already withdrawn its
forces and there was no possibility of re-intervention.

PRATHAM: BMS Special Booklet

58

VERBAL ABILITY
The Japanese early in Second World War used a modern form of the indirect attack, one that relied on stealth and
surprise for its effects. At Pearl Harbour, in the Philippines, and in South-East Asia, stealth and surprise were attained by
sailing under radio silence so that the navy's movements could not be tracked. Moving troops aboard ships into SouthEast Asia made it appear that the Japanese army was also 'invisible'. Attacks against Hawaii and Singapore seemed, to the
American and British defenders, to come from nowhere. In Indonesia and the Philippines, the Japanese attack was even
faster than the German blitz against France in the West.
The greatest military surprises in American history have all been in Asia. Surely there is something going on here beyond
the purely technical difficulties of detecting enemy movements. Pearl Harbour, the Chinese intervention in Korea, and the
Tet offensive in Vietnam all came out of a tradition of surprise and stealth. Us technical intelligence the location of enemy
units and their movementswas greatly improved after each surprise, but with no noticeable improvement in the
American ability to foresee or prepare what would happen next. There is a cultural divide here, not just a technical one.
Even when it was possible to track an army with intelligence satellites, as when Iraq invaded Kuwait or when Syria and
Egypt attacked Israel, surprise was achieved. The United States was stunned by Iraq's attack on Kuwait even though it had
satellite pictures of Iraqi troops massing at the border.
The exception that proves the point that cultural differences obscure the West's understanding of Asian behaviour was
the Soviet Union's 1979 invasion of Afghanistan. This was fully anticipated and understood in advance. There was no
surprise because the United States understood Moscow's world view and thinking it could anticipate Soviet action almost
as well as the Soviets themselves, because the Soviet Union was really a Western country.
The difference between the Eastern and the Western way of war is striking. The West's great strategic writer,
Clausewitz, linked war to politics, as did Sun-tzu. Both were opponents of militarism, of turning war over to the generals.
But there, all similarity ends. Clausewitz wrote that the way to achieve a larger political purpose is through destruction of
the enemy's army. After observing Napoleon conquer Europe by smashing enemy armies to bits, Clausewitz made his
famous remark in On War (1932) that combat is the continuation of politics by violent means. Morale and unity are
important, but they should be harnessed for the ultimate battle. If the Eastern way of war is embodied by the stealthy
archer, the metaphorical Western counterpart is the swordsman charging forward, seeking a decisive showdown, eager
to administer the blow that will obliterate the enemy once and for all with this view, war proceeds along a fixed course
and occupies a finite extent of time, like a play in three acts with a beginning, a middle, and an end. The end, the final
scene, decides the issue for good.
When things don't work out quite this way, the Western military men feels tremendous frustration. Sun-tzu's great
disciples, Mao Zedong and Ho Chi Minh, are respected in Asia for their clever use of indirection and deception to achieve
and advantage over stronger adversaries. But in the West, their approach is seen as underhanded and devious. To the
American strategic mind, the Viet Cong guerilla did not fight fairly. They should have come out into the open and fought
like men, instead of hiding in the jungle and sneaking around like a cat in the night.
105. According to the author, the main reason of the US losing the Vietnam war was :
(a) the Vietnamese understood the local terrain better.
(b) the lack of support for the war from the American people.
(c) the failure of the US to mobilise its military strength.
(d) their inability to fight a war on terms other than those they understood well.
106. Which of the following statements does not describe the Asian' way of war ?
(a) Indirect attacks without frontal attacks.
(b) The swordsman charging forward to obliterate the enemy once and for all.
(c) Manipulation of an adversary without his knowledge.
(d) Subduing an enemy without fighting.
107. Which of the following is not one of Sun-tzu's ideas ?
(a) Actual combat is the principal means of subduing an adversary.
(b) War should be undertaken only after through preparation.
(c) War is linked to politics.
(d) War should not be left to the generals alone.

PRATHAM: BMS Special Booklet

59

108. The difference in the concepts of war of Clausewitz and Sun-tzu is best characterized by :
(a) Clausewitz's support for militarism as against Sun-tzu's opposition to it.
(b) their relative degrees of sophistication.
(c) their attitude to guerilla warfare.
(d) their differing conceptions of the structure, time and sequence of a war.
109. To the Americans, the approach of the Viet Cong seemed devious because :
(a) the Viet Cong did not fight like men out in the open.
(b) the Viet Cong allied with America's enemies.
(c) the Viet Cong took strategic advice from Mao Zedong.
(d) the Viet Cong used bows and arrows rather than conventional weapons.
110. According to the author, the greatest military surprise in American history have been in Asia because :
(a) the Americans failed to implement their military strategies many miles away from their own country.
(b) the Americans were unable to use their technologies like intelligence satellites effectively to detect enemy
movements.
(c) the Americans failed to understand the Asian culture of war that was based on stealth and surprise.
(d) Clausewitz is inferior to Sun-tzu.

Passage VII:

(CAT 1998)

Each one has his reasons: for one art is a flight; for another, a means of conquering. But one can flee into a hermitage, into
madness, into death. One can conquer by arms. Why does it have to be writing? Because, behind the various aims of
authors, there is a deeper and more immediate choice which is common to all of us. We shall try to elucidate this choice,
and we shall see whether it is not in the name of this very choice of writing that the engagement of writers must be
required.
Each of our perceptions is accompanied by consciousness that human reality is a 'revealer'. That is, it is through human
reality that 'there is' being, or, to put it differently, that man is the means by which things are manifested. It is our
presence in the world which multiplies relations. It is we who set up a relationship between this tree and that bit of sky.
Thanks to us, that star which has been dead for millenia, that quarter moon, and that dark river are disclosed in the unity
of a landscape. It is the speed of our auto and our airplane which organizes the great masses of the earth. With each of
our acts, the world reveals to us a new face. But, if we know that we are directors of being, we also know that we are
not its producers. If we turn away from this landscape, it will sink back into its dark permanence. At least, it will sink back
: there is no one mad enough to think that it is going to be annihilated. It is we who shall be annihilated, and the earth will
remain in its lethargy until another consciousness comes along to awaken it. Thus, to our inner certainty of being
'revealers' is added that of being inessential in relation to the thing revealed.
One of the chief motives of artistic creation is certainly the need of feeling that we are essential in relationship to the
world. If I fix on canvas or in writing a certain aspect of the fields or the sea or a look on someone's face which I have
disclosed, I am conscious of having produced them by condensing relationships, by introducing order where there was
none, by imposing the unit of mind on the diversity of things. That is , I think myself essential in relation to my creation.
But this time it is the created object which escapes me; I cannot reveal the produce at the same time. The creation
becomes inessential in relation to the creative activity. First of all, even if it appears to others as definitive, the created
object always seems to us in a state of suspension; we can always change this line, that shade, that word. Thus, it never
forces itself. A novice painter asked his teacher, 'When should I consider my painting finished'? And the teacher answered,
"When you can look at it in amazement and say to yourself Im the one who did that!".
Which amounts to saying 'never'. For it is virtually considering one's work with someone else's eyes and revealing what
has been created. But it is self-evident that we are proportionally less conscious of the thing produced and more
conscious of our productive activity. When it is a matter of poetry or carpentry, we work according to traditional norms,
with tools whose usage is codified; it is Heidegger's famous 'they' who are working without our hands. In this case, the
result can seem to us sufficiently strange to preserve its objectivity in our eyes. But if we ourselves produce the rules of
production, the measures, the criteria, and if our creative drive comes from the very depths of our heart, then we never

PRATHAM: BMS Special Booklet

60

VERBAL ABILITY
find anything but ourselves in our work. It is we who have invented the laws by which we judge it, it is our history, our
love, our gaiety that we recognise in it. Even if we should regard it without touching it any further, we never receive from
it that gaiety or love we put them into it. The results which we have obtained on canvas or paper never seem to us
objective. We are too familiar with the processes of which they are the effects. These processes remain a subjective
discovery: they are ourselves, our inspiration, our ruse, and when we seek to perceive our work, we create it again, we
repeat mentally the operations which produced it; each of its aspects appears as a result. Thus, in the perception, the
object is given as the essential thing and the subject as the inessential. The latter seeks essentially in the creation and
creation and obtains it, but then it is the object becomes the inessential.
The dialectic is nowhere more apparent than in the art of writing, for the literary object is a peculiar top which exists only
in movement. To make it come into view a concrete act called reading is necessary, and it last only as long as this act can
last. Beyond that, there are only black marks on paper. Now, the writer cannot read what he writes, whereas the
shoemaker can put on the shoes he has just made if they are to his size, and the architect can live in the house he has
built. In reading, one foresees: one waits. He foresees the end of the sentence, the following sentence, the next page. He
waits for them to confirm or disappoint his foresights. The reading is composed of a host of hypotheses, followed by
awakenings, of hopes and deceptions. Readers are always ahead of the sentence they are reading in a merely probable
future which partly collapses and partly comes together in proportion as they progress, which withdraws from one page
to the next and forms the moving horizon of the literary object. Without waiting, without a future, without ignorance,
there is no objectivity.
111. The author holds that :
(a) there is an objective reality and a subjective reality.
(b) nature is the sum total of disparate elements.
(c) It is human action that reveals the various facets of nature.
(d) apparently disconnected elements in nature are unified in fundamental sense.
112. It is author's contention that :
(a) artistic creations are results of human consciousness.
(b) the very act of artistic creation leads to the escape of the created object.
(c) man can produce and reveal at the same time.
(d) an act of creation forces itself on our consciousness leaving us full of amazement.
113. The passage makes a distinction between perception and creation in terms of :
(a) objectivity and subjectivity.
(b) revelation and action.
(c) objective reality and perceived reality.
(d) essentiality and non-essentiality of objects and subjects.
114. The art of writing manifests the dialectic of perception and creation because :
(a) seading reveals the writing till the act of reading lasts.
(b) writing to be meaningful needs the concrete act of reading.
(c) this art is anticipated and progresses on a series of hypotheses.
(d) this literary object has a moving horizon brought about by the very act of creation.
115. A writer as an artist :
(a) reveals the essentiality of revelation. (b) makes us feel essential vis-a-vis nature.
(c) creates reality.
(d) reveals nature in its permanence.

Passage VIII:

(CAT 1997)
When talks come to how India has done for itself in 50 years of independence, the world has nothing but praise for our
success in remaining a democracy. On other fronts, the applause is less loud. In absolute terms, India hasn't done too
badly, of course, life expectancy has increased. So has literacy. Industry, which was barely a fledging, has grown

PRATHAM: BMS Special Booklet

61

tremendously. And as far as agriculture is concerned, India has been transformed from a country perpetually on the edge
of starvation into a success story held up for others to emulate.
But these are competitive times when change is rapid, and to walk slowly when the rest of the world is running is almost
as bad as standing still on walking backwards. Compared with large chunks of what was then the developing world South
Korea, Singapore, Malaysia, Thailand, Indonesia, China and what was till lately a separate Hong Kong-India has fared
abysmally.
It began with a far better infrastructure than most of these countries had. It suffered hardly or not at all during the Second
World War. It had advantages like an English speaking elite, quality scientific manpower (including a Nobel laureate and
others who could be ranked among the world's best) and excellent business acumen. Yet, today, when countries are
ranked according to their global competitiveness, it is tiny Singapore that figures at the top. Hong Kong is an export
powerhouse. So is Taiwan. If a symbol were needed of how far we have fallen back, note that while Korean Cellos are
sold in India, no one in South Korea is rushing to buy an Indian car.
The reasons list themselves. Topmost is economic isolationism. The government discouraged imports and encouraged
self-sufficiency. Whatever the aim was, the result was the creation of a totally inefficient industry that failed to keep pace
with global trends and, therefore, became absolutely uncompetitive. Only when the trade gates were opened a little did
this become apparent. The years since then have been spent in merely trying to catch up.
That the government actually sheltered its industrialists from foreign competition is a little strange. For in all other
respects, it operated under the conviction that businessmen were little more than crooks who were to be prevented
from entering the most important areas of the economy, who were to be hamstrung in as many ways as possible, who
were to be tolerated in the same way as an inexcusable wart. The high, expropriator rates of taxation, the licensing laws,
the reservation of whole swathes of industry for the public sector, and the granting of monopolies to the public sector
firms were the principal manifestations of this attitude. The government forgot that before wealth could be distributed, it
had to be created. The government forgot that it itself could not create, but only squander wealth.
Some of the manifestations of the old attitude have changed. Tax rates have fallen. Licensing has been all but abolished.
And the gates of global trade have been opened wide. But most of these changes were forced by circumstances partly by
the foreign exchange bankruptcy of 1991 and the recognition that the government could no longer muster the funds to
support the public sector, leave along expand it. Whether the attitude of the government itself, or that of more than
handful of ministers, has changed is open to question.
In many other ways, however, the government has not changed one with. Business still has to negotiate a welter of
negotiations. Transparency is still a longer way off. And there is no exit policy. In defending the existing policy, politicians
betray an inability to see beyond their noses. A no-exit policy for labour is equivalent to a no-entry policy for new
business. If one industry do not allow to retrench labour, other industries will think a hundred times before employing
new labour.
In other ways too, the government hurts industries. Public sector monopolies like the department of telecommunications
and Videsh Sanchar Nigam Ltd. make it possible for Indian businesses to operate only at a cost several times that of their
counterparts abroad. The infrastructure is in a shambles partly because it is unable to formulate a sufficiently
remunerative policy for private business, and partly because it does not have the stomach to change market rates for
services.
After a burst of activity in the early nineties, the government is dragging its feet. At the rate it is going, it will be another
50 years before the government realizes that a pro-business policy is the best pro-people policy. By then of course, the
world would have moved even farther ahead.
116. The writer's attitude towards the government is :
(a) critical
(b) ironical
(c) sarcastic
(d) derisive
117. The writer is surprised at the government's attitude towards its industrialists because :
(a) the government did not need to protect its industrialists
(b) the issue of competition was non-existent
(c) the government looked upon its industrialists as crooks
(d) the attitude was a conundrum

PRATHAM: BMS Special Booklet

62

VERBAL ABILITY
118. The government was compelled to open the economy due to :
(a) pressure from international markets
(b) pressure from domestic market
(c) foreign exchange bankruptcy and paucity of funds with the government
(d) All of the above
119. The writer ends the passage on a note of :
(a) cautious optimism
(b) pessimism
(c) optimism
(d) pragmatism.
120. According to the writer, India should have performed better than the other Asian nations because :
(a) it had adequate infrastructure
(b) it had better infrastructure
(c) it had better politicians who could take the required decisions
(d) All of the above
121. India was in better condition than the other Asian nations because :
(a) it did not face the ravages of the Second World War
(b) it had an English speaking populace and good business sense
(c) it had enough wealth through its exports
(d) Both (a) and (b) above
122. The major reason for India's poor performance is :
(a) economic isolationism. (b) economic mismanagement (c) inefficient industry (d) All of these
123. One of the features of the government's projectionist policy was :
(a) encouragement of imports
(b) discouragement of exports
(c) encouragement of exports
(d) discouragement of imports
124. The example of the Korean Cielo has been presented to highlight :
(a) India's lack of stature in the international market
(b) India's poor performance in the international market
(c) India's lack of creditability in the international market
(d) India's disrepute in the international market
125. According to the writer:
(a) Indias politicians are myopic in their vision of the countrys requirements
(b) Indias politicians are busy lining their pockets
(c) Indias politicians are not conversant with the needs of the present scenario
(d) All of the above

Passage IX:

(CAT 1996)
I want to stress this personal helplessness we are all stricken with in the face of a system that has passed beyond our
knowledge and control. To bring it nearer home, I propose that we switch off from the big things like empires and their
wars to more familiar little things. Take pins for example! I do not know why it is that I so seldom use a pin when my wife
cannot get on without boxes to them at hand; but it is so; and I will therefore take pins as being for some reason specially
important to women.
There was a time when pinmakers would by the material; shape it; make the head and the point; ornament it; and take it
to the market, and sell it and the making required skill in several operations. They not only knew how the thing was done
from beginning to end, but could do it all by themselves. But they could not afford to sell you a paper or pins for the
farthing. Pins cost so much that a woman's dress allowance was calling pin money.
By the end of the 18th century Adam Smith boasted that it took 18 men to make a pin, each man doing a little bit of the
job and passing the pin on the next, and none of the them being able to make a whole pin or to buy the materials or to
sell it when it was made. The most you could say for them was that at least they had some idea of how it was made,
though they could not make it. Now as this meant that they were clearly less capable and knowledgeable men than the
old pin-makers, you may ask why Adam Smith boasted of it as a triumph of civilisation when its effect had so clearly a

PRATHAM: BMS Special Booklet

63

degrading effect. The reason was that by setting each man to do just one little bit of the work and nothing but that, over
and over again, he became very quick at it. The men, it is said, could turn out nearly 5000 pins a day each; and thus pins
became plentiful and cheap. The country was supposed to be richer because it had more pins, though it had turned
capable men into mere machines doing their work without intelligence and being fed by the spare food of the capitalist
just as an engine is fed with coal and oil. That was why the poet Goldsmith, who was a farsighted economist as well as a
poet, complained that 'wealth accumulates, and men decay'.
Now-a-days Adam Smith's 18 men are as extinct as the diplodocus. The 18 flesh-and-blood men have been replaced by
machines of steel which spout out pins by the hundred million. Even sticking them into pink papers is done by machinery.
The result is that with the exception of a few people who design the machines, nobody knows how to make a pin or how
a pin is made : that is to say, the modern worker in pin manufacture need not be one-tenth so intelligent, skilful and
accomplished as the old pinmaker; and the only compensation we have for this deterioration is that pins are so cheap that
a single pin has no expressible value at all. Even with a big profit stuck on the cost-price you can buy dozens for a farthing;
and pins are so recklessly thrown away and wasted that verses have to be written to persuade children (without success
that it is a sin to steal, if even it's a pin.
Many serious thinkers, like John Ruskin and William Morris, have been greatly troubled by this, just as Goldsmith was, and
have asked whether we really believe that it is an advance in wealth to lose our skill and degrade our workers for the sake
of being able to waste pins by the ton. We shall see later on, when we come to consider the Distribution Leisure, that the
cure for this is not to jo back to the old free for higher work than pin-making or the like. But in the meantime the fact
remains that the workers are now not able to make anything themselves even in little bits. They are ignorant and helpless,
and cannot lift their finger to begin their days work until it has all been arranged for them by their employers who
themselves do not understand the machines they buy, and imply pay other people to set them going by carrying out the
machine maker's directions.
The same is true for clothes. Earlier the whole work of making clothes, from the shearing of the sheep to the turning out
of the finished and washed garment ready to put on, had to be done in the country by the men and women of the
household, especially the women; so that to this day an unmarried woman is called a spinster. Now-a-days nothing is left
of all these but the sheep shearing; and even that, like the milking of cows, is being done by machinery as the sewing is.
Give a woman a sheep today and ask her to produce a woollen dress for you; and not only will she be quite unable to do
it, but you are likely to find that she is not even aware of any connection between sheep and clothes. When she gets her
clothes, which she does by buying them at the shop, she knows that here is a difference between wool and cotton and
silk, between flannel and merino, perhaps even between stockinet and other wefts; but as to how they are made, or what
they are made of, or how they came to be in the shop ready for her to buy, she knows hardly anything. And the shop
assistant from whom she buys is no wiser. The people engaged in the making of them know even less; for many of them
are too poor to have much choice of materials when they buy their own clothes.
Thus the capitalist system has produced an almost universal ignorance of how things are made and done whilst at the
same time it has caused them to be made and done on a gigantic scale. We have to buy books and encyclopedias to find
out what it is we are doing all day; and as the books are written by people who are not doing it, and who get their
information from other books, what they tell us is twenty to fifty years out of date knowledge and almost impractical
today. And of course most of us are too tired of our work when we come home we do not want to read about it; what
we need is cinema to take our minds off it and feel our imagination.
It is a funny place, this word of capitalism, with its astonishing spread of education and enlightenment. There stand the
thousands of property owners and the millions of wage workers, none of them able to make anything, none of them
knowing what to do until somebody tells them, none of them having the least notion of how it is made that they find
people paying them money, and things in the shops to buy with it. And when they travel they are surprised to find that
savages and Esquimaxand villages who have to make everything for themselves are more intelligent and resourceful! The
wonder would be of they were anything else, we should die of idiocy through disuse of our mental faculties if we did not
fill our heads with romantic nonsense out of illustrated newspapers in novels and play and films. Such stuff keeps us alive,
but it falsifies everything for us so absurdly that it leaves us more or less dangerous lunatics in the real world.
Excuse my going on like this; but as I am a writer of books and play myself, I know the folly and peril of it better than you
do. And when I see that this moment of our utmost ignorance and helplessness, delusion and folly, has been stumbled on

PRATHAM: BMS Special Booklet

64

VERBAL ABILITY
by the blind forces of capitalism as the moment for giving votes to everybody, so that the few wise women are hopelessly
overruled by the thousands whose political minds, as far as they can be said to have any political minds at all, have been
formed in the cinema, 1 realize that I had better stop writing plays for a while to discuss political and social realities in this
book with who are intelligent enough to listen to me.
126. A suitable title to the passage would be :
(a) You can't hear a pin-drop now-a-days
(b) Capitalism and labour Disintegration : Pinning the Blame
(c) The Saga of the Non Safety Pins
(d) Reaching the Pinnacle of Capitalistic Success
127. Why do you think that the author gives the example of Adam Smith ?
(a) Because he thinks that Adam Smith was a boaster without any facts to back his utterance
(b) Because he wants to give us an example of something undesirable that Adam Smith was proud of
(c) Because he is proud to be a believer in a tenet of production that even a great man like Adam Smith boasted
about
(d) Because he feels that Adam Smith was right when he said that it took 18 men to make a pin
128. Which of the following is true as far as pins are concerned ?
(a) The cost of pins is more now-a-days to produce
(b) Earlier, workmen made pin with a lot of love and care
(c) Pinball machines are the standard pin producing gadgets now-a-days
(d) It took much longer to make a pin earlier
129. The reason that children have to be taught that stealing a pin is wrong is that :
(a) They have an amazing proclivity to steal them right from childhood
(b) Pins are so common and cheap that taking one would not even be considered stealing by them
(c) Stealing a pin would lead to stealing bigger and bigger things in the future
(d) Stealing an insignificant thing like a pin smacks of kleptomania
130. It may be inferred from the passage that the author :
(a) is a supporter of the craftsmanship over bulk mechanished production
(b) is a supporter of assembly line production
(c) is a defender of the faith in capitalistic production
(d) None of the above.
131. Which of the following is not against the modern capitalistic system of mass production ?
(a) John Ruskin
(b) Goldsmith
(c) Adam Smith
(d) William Morris
132. Goldsmith's dictum, "wealth accumulates, and men decay, "in the context of the passage, probably means :
(a) The more wealthy people get, the become more and more corrupt
(b) The more rich people get, they forget the nuances of individual ability
(c) People may have a lot of money, but they have to die and decay someday
(d) The more a company gets wealthy, the less they take care of people
133. When the author says that a woman now is likely to know about any connection between sheep and clothes, he is
probably being :
(a) vindictive
(b) chauvinistic
(c) satirical
(d) demeaning
134. Which of the following can be a suitable first line to introduce the hypothetical next paragraph at the end of the
passage ?
(a) The distribution of leisure is not a term that can be explained in a few words
(b) If people wear clothes they hardly seem to think about the mode of production
(c) Machines are the gods of our age and there seems to be no atheists
(d) None of the above

PRATHAM: BMS Special Booklet

65

Passage X:

(CAT 1996)
That the doctrines connected with the name of Mr. Darwin are altering our principles has become a sort of common
place thing to say. And moral principles are said to share in this general transformation. Now, to pass by other subjects, I
do not see why Darwinism need change our ultimate moral ideas. It was not to modify our conception of the end, either
for the community, or the individual, unless we have been holding views, which long before Darwin were out of date. As
to the principles of ethics I perceive, in short, no sign of revolution. Darwinism has indeed helped many to truer
conception to the end, but I cannot admit that it has either originated or modified that conception.
And yet in ethics Darwinism after all perhaps be revolutionary, it may lead not to another view about the end, but to a
different way of regarding the relative importance of the means. For in the ordinary moral creed those means seem
estimated on no rational principle. Our creed appears rather to be an irrational mixture of jarring elements. We have the
moral code of Christianity, accepted in part; rejected practically by all save a few fanatics. But we do not realize how in its
very principle the Christian ideals is false. And when we reject this code for another and in part a sounder morality, we
are in the same condition of blindness and of practical confusion. It is here that Darwinism, with all the tendencies we
may group under that name, seems destined to intervene. It will make itself felt, I believe more and more effectually. It
may force us in some points a correction of our moral view and a return to non-Christian and perhaps a Hellenic ideal. I
propose to illustrate here these general statements by some remarks on punishment.
Darwinism, I have said, has not even modified our ideas of the Chief Good. We may take that asthe welfare of the
community realised in its members. There is, of course, a question as to meaning to be given to welfare. We may identify
that with mere pleasure, or gain with mere system, or may rather view both as inseparable aspects of perfection and
individuality. And the extent and nature of the community would once more be a subject for some discussion. But we are
forced to enter on these controversies here. We may leave welfare undefined, and present purpose need not distinguish
the community from the state. The welfare of this whole exists, of course, nowhere outside the individuals again have
rights and duties only as members in the whole. This is the revived Hellenism-or we may call it in the organic view of
thing-urged by German Idealism early in the present century.
135. What is most probably the author's opinion of the existing moral principles of the people ?
(a) He thinks they have to be revamped in the light of Darwinism
(b) He thinks that they are okay as they are and do not need any major change
(c) He thinks that it may be a good idea to have a modicum of the immortal Darwinism in us
(d) Cannot be determined from the passage
136. According to the author, the doctrines of Mr. Darwin :
(a) have changed our physical and moral principles
(b) have to be re-evaluated to correct the faults endemic in them
(c) do not have to change our moral ideas
(d) are actually new versions of old moral rules
137. What, according to the passage, is the Chief Good ?
(a) Being good and kind to all fellow human beings
(b) The greatest good of the greatest number
(c) The welfare of the community realised in its members
(d) Cannot be determined from the passage
138. It is implied in the passage that :
(a) A Hellenic ideal is not proper substitute of the Christian ideal
(b) What mankind needs is a Hellenic ideal rather than a Christian one
(c) Darwinism is more Christian than Hellenic
(d) Fanatics do not understand what Darwinism really is

PRATHAM: BMS Special Booklet

66

ANSWERS & EXPLANATIONS

ANSWERS & EXPLANATIONS

PRATHAM: BMS Special Booklet

67

UNIT 1: QUANTITATIVE APTITUDE


NUMBER SYSTEM:
1. (b) Let the number of currency of 1 miso, 10 misos and 50 misos be x, y and z respectively.
Then, x + 10y + 50z =107
Now, possible values of z = 0, 1, 2
If z = 0, then x + 10y = 107. Now, number of pairs of values of x and y that satisfy the above equation are 11. These
pairs are: (7, 10), (17, 9), . (107, 0). If z = 1, then x + 10y = 57. For this number of pairs of value of x and y is 6.
(7, 5), (17, 4), (27, 3), .., (75, 0)
If z = 2, then x + 10y = 7. There is only one such pair of x and y, (7, 0) which satisfy the equation.
Therefore, total number of ways = 11 + 6 + 1 = 18
2. (c) Using options, we find that four consecutive odd numbers are 37, 39, 41 and 43. The sum of these 4 numbers is
160, divided by 10, we get 16 which is a perfect square.
41 is one of the odd numbers.
3. (a) a, a, + 2, a + 4 are prime numbers.
Put value of a starting from 3
We will have 3, 5, 7 as the only set of prime numbers satisfying the given relationship.
4. (b) At last thief is left with one diamond. Hence, the number of diamonds before he gave some diamonds to the third

watchman = x 2 + 2 = 1
4

= 1 x = 6.
2
Hence, he had 6 diamonds before he gave 5 to the third watchman. Similarly, number of diamonds before giving to
second watchman
4
4
= 2 = 6 = 16 and number of diamonds before giving to the first watchman = 2 = 16 = 36.
Therefore, the thief stole 36 diamonds originally.
5. (c) Given, 100A + 10B + C = A! + B! + C!
We can take the help of options. Value of B will be less than 7 because 7! = 5040 which is a four digit number.
Hence, B can be either 4 or 2. Again value of A and C has to be any of 6! = 720 or 5! = 144 because any other
combination will not produce a three digit number. Hence, required number will be 145 as 1! + 4! + 5! = 1 + 24 +
120 = 145.

SIMPLIFICATION:
6. (d) Given expression
+2
+3
+4
+5
+5
=

=
+1
+2
+3
+4
+1
7. (d) r 4 + 2r 3 + 3r 2 + r
= 16 + 2 (-8) + 3 4 - 2 = 10
2 2

2+ 2
x y = + =
2+ 2
2+ 2
x = + =

2+ 2
1 2 2 =

8. (d)

= =

x=

2+ 2

+ 2 2

( 2 2 )

2 2

PRATHAM: BMS Special Booklet

68

ANSWERS & EXPLANATIONS


9. (a) Cost of Cave
= 5 rocks + 2 stones + 3 pebbles
= 35 stones + 14 pebbles + 3 pebbles
= 245 pebbles + 14 pebbles + 3 pebbles
= 262 pebbles
1 rock = 49 pebbles
To use 6 rocks, it requires 294 pebbles
Change required = 294 262 = 32 pebbles
= 4 stones and 4 pebbles
10. (c) r = 9, s = 4
1

852 = + 2 = 9 + 18 = 9 9 = 9.22

PERCENTAGE:
11. (a) Let the number of questions in A, B and C be a, b and c respectively.
We have, a+b+c = 100
Total marks would be a + 2b + 3c
Given b = 23, Total marks from section B = 46
Different possible values for c are 1, 2, 3, ..
Corresponding values for a are 76, 75, 74, (since total questions are 100).
When c = 1, a = 76, b = 23
Total marks from sections A, B, C are 76, 46, 3 respectively.
76
76
Percentage marks from A = 76+46+3 = 125 > 60%
When c = 2, a = 75, b = 23
75
75
Percentage marks from A = 75+46+6 = 127 < 60%
For all other values, when c increases, a decreases and contribution of marks from A keeps decreasing.
There is only 1 possible value for questions from group C.
12. (c) Marks from section C = 24
Since, B contributes at least 20% and A contributes at least 60% to the total contribution, C is maximum of the 20%.
Total marks 120.
Total questions are 100. So a + b = 92
Minimum number questions in A = 72 and in B = 12.
Total score
Condition
c
b
a
% contribution of A % contribution of B
(a+2b+3c)
satisfied
80
24
8
12
80
128

> 60%
< 20%
128
128
8
13
79
129
>60%
>20%

8
14
78
130
60%
>20%

8
15
77
131
<60%
>20%

Hence, only 2 possible values of B exist.


1+p
1q
13. (c) If p = q, then 9 doz. 100 100 = 9 doz.

Hence, for the final value to be equal to the original value, p should be greater than q. To understand the concept let
p = q = 20 and original number be 100, then
100 12 08 = 96.
Hence, it is very clear that for final value to be equal to 100, p should be greater than q.

PRATHAM: BMS Special Booklet

69

14. (a) Let number of bacteria in the first generation be x.


Number of bacteria in the second, third, fourth .. generation would be
x
4x
16x
8 2 , 8 2 , 8 2 and so on.

x, 4x, 16x, 64x, it is a GP with common difference 4. Hence, seventh term of GP


= x(4)6 = 4096 x(2)12 = 4096
x = 1 or 1 million.
15. (c) Let x voters voted against the party in the Assembly Poll. Then, votes in favour = (y x)
{For calculations simplicity suppose 2,60,000 = y}
Majority of votes by which party won in previous poll = (y x) x = (y 2x)
Now, voters polled against the party in general election = 1.25x and votes polled in favour of the party = (y 1.25x)
Majority of votes by which party lost in general election = (1.25x) (y 1.25x) = (2.5x y)
It is given that,
(2.5x y) = 2(y - 2x) 2.5x y = 2y 4x
3 2,60,000
or 6.5x = 3y x =
= 1,20,000
6.5
Votes polled by the voters for the party in Assembly Polls = (2,60,000 1,20,000) = 1,40,000

PROFIT AND LOSS:


16. (c) Let x g of sucrose be added with 1 g of saccharin to obtain a mixture 100 times as sweet as glucose.
Then, 1.00x + 1 675.00 = 0.74(x ) 100
or x + 675 = 74(x )
x = 9.26g or 9 g (approximately).
17. (a)

0.74 + 1.00 2 + 1.7 3


6

= 1.3

18. (a) 78 = 50 + 10 + 10 + 2 + 2 + 2 + 2 = 7
69 = 50 + 10 + 5 + 2 + 2 = 5
1.01 = 50 + 25 + 10 + 10 + 2 + 2 + 2 = 7
Hence, 7 + 5 + 7 = 19 coins
y
19. (d) Given, (x + y) x 2 + y 2 =
2

+
x + 2 = x2 + y2
x
Now, using options we find that option (d) satisfies the
above relationship 3 + 2 = 9 + 16 5 = 5
y
1
x
20. (b) Mayank paid of the sum paid by other three. Let the other three paid Rs x jointly, then Mayank paid

So, x + 2 = 60000 x = 40000. Hence, Mayank paid $20,000.


Similarly, Mirza paid $15000 and little paid $12000
Thus, Jaspal had to pay $13000.

RATIO & PROPORTION:


21. (a) Let the numbers be 6K and 13K
78K = 312 K = 4
Larger number = 52
22. (d) Ratio of the amount collected from the 1st class and the 2nd class passengers = 3:50.

PRATHAM: BMS Special Booklet

70

ANSWERS & EXPLANATIONS


Amount collected from the 2nd class passengers
50
= 53 1325 = Rs. 1250
23. (d) y

1
3

= 3

when x = 2, y =
1

1
6

6 = 82 = 1
1

y = 3

x=1y=0=

24. (d) Let y p + q. where p x and q


y = K (p + q),

N
p = Mx, q = K Mx + x
y=

10
3

, when x = 3 K 3M +

N
3

MK = 2 and NK = - 8
2
8
8
p = , = and so y = 2x - .

25. (b) Let the number be x

5
Required ratio = 1 = 6 =
+
5

10
3

, also y = 6, when x = 4 24 = K(16M+N)

AVERAGES:
26. (b) If each box is to be weighed (in pair) with every other box, then there would be ten such combinations of weight.
The best way to solve the question is through option.
Max. weight cannot be 60 kg because to make the total weight of two boxes 121 kg other should be 61 kg. Again if
the max. weight is 64 kg then to make a total weight of 121 kg other box weight should be 57 kg and to make up to a
total of 120 kg the next box weight should be 63 kg. Now if we add up weight of 63 kg and 64 kg boxes, the total
becomes 127 and this combination is not given in the question. However, if we consider the max. weight to be 62 kg,
then other boxes would be of 59 kg, 54 kg, 58 kg, 56 kg respectively. Hence, the maximum weight would be 62 kg.
n

27. (a) Sum of odd integers in the set S = 2 3 + 1 2 = 2 + 4 = ( + 2)


2
2
Therefore, the average of the odd integers in the set S = n + 2
n
n
Sum of even integers in the set S = 2 2 2 + n 1 2 = 2 2n + 2 = n(n + 1)
Therefore, the average of the even integers in the set S = n + 1, Therefore, X Y = (n + 2) (n + 1) = 1
28. (d) Number of students in group D is maximum. But no information about the weight of the students is given, hence
no comparison of weight of group D can be made with that of other group.
29. (c) If one student from group A is shifted to group B, the number of students in the class still remains the same and it
does not affect the average weight of the class.
150
200
30. (c) Suppose the weight of each student be 10 kg, then average weight of class A, B, C, D would be 15 = 10, 20 =
250

400

10, 25 = 10 and 40 = 10.


Hence, (a) is true.
Total weight of A and C = (150 + 250) = 400 kg.
which is 2 200. Hence, (b) is true.
In option (a), we have seen that average weight of all the four groups is same, hence average weight of D cannot be
greater than the average weight of A. Hence option (c) is the correct answer.

PRATHAM: BMS Special Booklet

71

SIMPLE INTEREST:
31. (d) Suppose the person had deposited Rs. x at the time of opening the account.
After one year, he had
101
11
Rs. + 100
= . 10
After two years, he had
11
11
101
121
Rs. 10 + 10 100 = 100
. (i)
After withdrawing Rs. 5000 from Rs.
121500000
100

121
100

The balance = Rs.


After 3 years, he had
121500000
121500000
10 1
+
100
100
100
11(121500000 )

=
1000
After withdrawing Rs. 6000 from amount (ii), the balance
1331
= Rs. 1000 11500
After 4 years, he had
1331 x11500000
Rs.
+ 10% of Rs.
1000
= Rs.

11
10

1331
1000

. (ii)

1331 x11500000
1000

.. (iii)

11500

After withdrawing Rs. 10000 from amount (iii), the balance = 0


11 1331
10 1000 11500 10000 = 0 x = 15470

32. (b) Let the annual installment be Rs. x


1005 4
Amount of Rs. 100 after 4 years = Rs. 100 + 100
= Rs. 120
Present Value (P. V.) of Rs. 120 due after 4 years = Rs. 100
Present Value (P.V.) of Rs. x due after 4 years
100
5
= 120 = 6
Similarly, P. V. of Rs. x due after 3 year =
10
P. V. of Rs. x due after 2 years = 11
20
P. V. of Rs. x due after 1 year = 21
5
20
10
20
Hence, 6 + 23 + 11 + 21 = 6450

20
23

x 1810
33. (d) Let the amount of money lent at 12% interest be Rs. x
Amount of money lent at 12.5% interest = Rs. (2540 x)
121
(2540 ) 12.51

+
= 311.60

100
0.5
100

31750
100

100

= 311.60

0.5x = 31750 31160 = 590 x = 1180


PR12
100
1
34. (d) 100 = P R = 12 = 8 3 = 8.33%
35. (a) Value of flat after two years
= 1 x 1.1 x 1.1= 1.21 lakh
Value of plot after two years

PRATHAM: BMS Special Booklet

72

ANSWERS & EXPLANATIONS


=1.1 x 1.05 x 1.05 = 1.21275 lakh
Hence, difference in price after two years (1.21275 1.21) x 100000 = Rs. 275.

COMPOUND INTEREST:
10 4

36. (a) Rs. 10000 after 4 years become 10000 1 + 100


11

11

11

11

= 10000 10 10 10 10 = 14641
Rs. 10000 after 3 years become
11
11
11
= 10000 10 10 10 = 13310
Rs. 10000 after 2 years become
11
11
= 10000 10 10 = 12100
Rs. 10000 after 1 year become Rs. 11000
Total amount after 4 years = Rs. 51051
37. (b) Let P be the sum
21
Amount after one year = P + 5% of P = 20
Amount after two years
21
21
231
= 20 + 10% of 20 = 200
231
231
1386
+ 20% of 200 = 1000
200
1386 P
1000 = 16632 P = 12000

38. (c) Suppose value of the asset two years ago = Rs. x
Value of the asset one year ago
9
= x 10% of x =
10

9
10

x - = 12000 = 120000
Value of the asset at present
9
9
= 10 10% 10
81

= 100 81 1200 = Rs. 97200


39. (b) S. I. for one year = Rs. 1100
Amount at the end of 2nd year
1100 100
= 101 = Rs. 11000
40. (a) Let Amount of A = P
Amount of B = 1301 P
P 1+

4 7
100

= 1301

P = 1301

26
25

26

1+

4 9
100

25

625P = 1301 676 676P P = 676


A gets Rs. 676 and B gets Rs. 625

PRATHAM: BMS Special Booklet

73

TIME, SPEED AND DISTANCE:


41. (b) Fuel consumption is given in litre per hour. It is, therefore, clear from the graph that in travelling 60 km fuel
4
consumption is 4 L. Hence in travelling 200 km fuel consumption will be 60 200 = 13.33 L.
42. (b) At a speed of 40 km/h, 60 km/h and 80 km/h distance travelled in 1 L of petrol is
40
60
80
= 2.5 = 16 km, 4 = 15 km, 7.9 = 10.1 km respectively.
Hence, at lower speed fuel consumption is less. Hence, in order to minimize the fuel consumption, the speed should
be decreased.
43. (b) Let the distance of race be x metres which is covered by A in t seconds. Then in the same time B covers (x-12)m
and C covers (x 18) m.
x
Speed of A = t m/s,
(x12)
m/s
t
(x18)
= t m/s

Speed of B =
and Speed of C

Time taken by B to finish the race =

x
(x 12)
t

Now, distance travelled by C in this time =

x(x18)
(x12)

tx

= (x12) s
x

(x12)

(x18)
t

=x8

= x 8 x = 48 m

44. (b) Let the length of the tunnel and speed of cat be 8 km and 8 km/h respectively.
Time taken by cat to reach entrance of tunnel = 3 h and time taken by cat to reach the exits of tunnel = 5 h.
Time taken by train to cover tunnel = 2 h
Hence, ratio of speeds of train and cat = 4 : 1
45. (b) Since, each word is put off after a second, hence the required least time
5
17
41
= LCM of 2 + 1, 4 + 1, 8 + 1
7 21 49
, ,
2 4 8
LCM of (7,21,49)
49 3
= 2
HCF of (2,4,8)

= LCM of
=

= 73.5 seconds

TIME AND WORK:


2

46. (b) Small pump = 3 the rate of large pump


3 small pumps = 2 large pumps
Hence, 3 small pumps + 1 large pumps = 3 large pumps
1
Hence, required fraction of time = 3
47. (d) Since, six technicians working at the same rate complete work of one server in 10 h.
Hence, total work = 10 6 = 60 man hours.
Now, from 11.00am to 5 pm total man hours = 6 6 = 36
From 5 pm to 6 pm total man hours = 7
From 6 pm to 7 pm total man hours = 8
From 7 pm to 8 pm total man hours = 9
60
Hence, the work will be completed at 8 pm.

PRATHAM: BMS Special Booklet

74

ANSWERS & EXPLANATIONS


1 1

48. (d) Working efficiency per day of A, B, C and D = 4 , 8 , 16 and


3
16

9
32

1
32

respectively. Using options, we find that B and C

does of work per day, A and D does work per day. Hence, A and D take
the first pair must comprise of A and D.
1
49. (b) If Shyama takes 1 min for every 3 steps, then he takes 3 min for every step.
For every 25 steps, he will take =

1
2

25
3

32
9

days, B and C take

16
3

days. Hence,

min.

Similarly, Vyom takes min for every step, hence for 20 steps, he will take 10 min.
25

Difference between their time = 10 3 = 1.66 min


Escalator takes 5 steps in 1.66 min. Therefore, Speed of escalator is 1 step for 0.33 min = 3 steps/min. If escalator is
moving, then Shyama takes 25 steps and escalator also takes 25 steps.
Hence, total number of steps = 50.
50. (d) Let the speed of man in still water be x miles/h and speed of the current be y miles/h.
12
12
Then, xy x + y = 6
.(i)
and

12
2xy

12
2x+y

.(ii)

=1

Solving Eqs. (i) and (ii),

y=

8
3

CLOCKS AND CALENDARS:


51. (a) Here, H = 5 and M = 3
12
12
6
(5H M) = (5 5 3) = 30 and 24.
11

11

11

The hands will be 3 minutes apart at 3011 minutes past 5 and 24 minutes past 5 O'clock.
52. (c) From 7 a. m. to 4 : 15, the time is 9 hours 15 min, i.e., 555 min.
37
Now, 12 min of this watch = 3 minutes of correct watch.
312

555 min. of this watch = 37 555 on a correct watch


= 540 min. or 9 hours of correct watch.
correct time after 7 a.m. is 4 p.m.
53. (c) At 4 O'clock hands are 20 min. spaces apart. At time between 4 : 30 and 5 the hands will be in a straight line when
they point in opposite directions and there is a space of 30 mins. between them. So, to be in this position min. hand
has to gain 30 + 20 = 50 min. spaces. Min. hand gains 50 min. in
60
600
6
50 = 11 = 54 11 min.
55
6

Required time = 54 mins. past 4


11
54. (a) During the interval we have two leap years as 1992 and 1996 and it contains February of both these years
The interval has (5 + 2) = 7 odd days or 0 odd day.
Hence, 7 January 1997 was also Tuesday.
55. (d) Total number of odd days = 1600 years have 0 odd day + 300 years have 1 odd day + 49 years (12 leap + 37
ordinary) have 5 odd days + 26 days of Jan. have 5 odd days
= 0 + 1 + 5 + 5 = 4 odd days
So, the day was Thursday.

PRATHAM: BMS Special Booklet

75

LINEAR EQUATIONS:
56. (c) Let the number of 20 paise stamps purchased be x and number of 15 paise stamps purchased be y.
x + y = 47
.(i)
.20x + 0.15y = 8.80 x + 0.75y = 44
...... (ii)
Solving (i) and (ii), we get
y = 12, x = 35
57. (d) 4x- 17y= 1. And given that x 1000
Hence, we can say that 17y + l 4000
i.e., y 235
Further also note that every 4th value of y e.g., 3,7,11, ...will give an integer value of x.
235
So, number of values of y = 4 = 58.
58. (c) x2 + 5y2 + z2 = 2y (2x + z)
Put x = 2y
4y2 + 5y2 + z2 = 2y (4y + z) or 9y2 + z2 = 8 y2 + 2 yz
..(i)
This is not necessarily true. Put y = z in (i), we get
9z2 + z2 = 8 z2 + 2z2 or 10z2 = 10z2
(i) is true for y = z(x = 2y & x = 2z y = z). Therefore only A & B satisfy the given result.
59. (d) x =

2 + 2 +4 + 16
+4 +4

Let n = t
x=

4 + 2 2 + 4 + 16
2 + 4+4

+2 ( 3 + 8)
+2 2

3+ 8
+2

= 2 2 + 4

For t = 6 to t = 6 2 (putting in above equation)


(40 - 12) < x < (72 + 4 - 12 2) 28 < x < 76 - 12 2 x or 28 < x < 60
60. (c) Given, x y - x - y = 0
Adding 1 to both sides of the equation, we get
xy x y + 1 = + 1
y (x - l) - l(x - l) = 1
(y - l)(x - l) = l
...(i)
As x and y are integers, x - 1 and y - 1 are integers.
Hence, x - 1 and y - 1 must both be 1 or - 1 to satisfy equation (i) i . e . , x = 2, y = 2 or x = 0, y = 0.
Hence only two integer pairs satisfy the condition x + y = x y .

QUADRATIC EQUATIONS:
61. (a) x2 2x - 3 = (x - 3) (x + 1)
(a) = x 2 - 7x + 6 = (x - 6) (x - 1)
(b) = x2 - 9 = (x + 3) (x - 3)
(c) = x2 4x + 3 = (x - 3) (x - 1)
(d) = x2 6x + 9 = (x - 3)2
62. (d) If 2 + i 3 is a root of the equation
2 + + = 0
Then its other root will be 2- 3.
x 2 + px + q = x 2 i 3 (x 2 + i 3)
= 2

PRATHAM: BMS Special Booklet

76

ANSWERS & EXPLANATIONS


= 2 + 4 4 + 3 = 2 4 + 7
p = -4, q = 7
63. (a) Let the equation be 2 + + = 0
Suppose its roots are and .

Then, + =
(i)

(ii)

also + =
2 2

... (iii)

=
From (ii) and (iv), we get
= 1 or c = a
From (i), we get
b2

(iv)

2 + 2 + 2 = 2 + 2 = 2
b2 + ab 2a2 = 0
2 + 8 2

Or b =
= , 2.
2
64. (d) Let , be the roots of the equation
2 + = 0
+ = 1, = m
Let 2 , be the roots of the equation
2 + 3 = 0
2 + = 1, 2 = 3m

(i) + = 1
2 + m = 0
3
(ii) 2 + 2 = 1

(i)
... (ii)
... (iii)

42 2 + 3m = 0
.. (iv)
Solving (iii) and (iv)
m = 2
2 = 3m = 3
2 + = 1 = - 1
+ = 1 = 2,
=-1
= 2, = 3 m = -2.
65. (d) Sum of roots, p + q = - 2
Product of roots, pq = - - 1
Now p2 + q2 = (p + q)2 - 2pq
= ( - 2)2 + 2 ( + 1)
= 2 + 4 - 4 + 2 + 2
= ( - l)2 + 5
Hence, the minimum value of this will be 5.

PRATHAM: BMS Special Booklet

77

PROGRESSION:
66. (a) Given statement
(ax/2)2 = (logb x) x (logx a)
ax = logb a
x log a = log [logb a]
x = loga [logb a]
67. (a) (a + b)2,a2 + b2 (a - b)2
This is a series in A.P. with common difference (- 2ab).
Given n = 20
20
S(a, b) = 2 [2(a + b)2 + (20 - 1) ( - 2ab)]
= 10[2a2 + 2b2 + 4ab 38ab]
= 20[a2 + b2 - 17ab]
S (7, 3) = 20 [49 + 9 - 357]
= 20 (- 299) = -5980
68. (b) 102, 108, 114, .. ,996
There are 150 such numbers divisible by 6.
69. (a) x, y, z are in GP.
x + y, y + z, z + x are in A.P.
...... (i)

z
Common ratio of the G.P. = x = y = r,
Say, Also (i)
y+z=

r= =
r=

2
1+

+ +(+)
2x
2

2
2

( +) = + =
1+
2

=y+z

2 + 2 = 0
(r - 1) (r + 2) = 0
r = -2 [r = 1 x = y = z].
70. (a) 4+ 12 + 20 + 28 + .. + 996
Tn = a + (n - 1)d
Here, a = 4, d = 8,
Tn = 996
996 = 4 + (n - l) 8
8n -8 + 4 = 996
n = 125
125
125 = 2 2 4 + 125 1 8
=

125
2

[8 125] = 62500

PRATHAM: BMS Special Booklet

78

ANSWERS & EXPLANATIONS

PROBABILITY:
1

71. (b) Prob. that it rains on the 1st day = 2


1

Prob. that it rains on the 2nd day = 2


1

Prob. that it rains on the 3rd day = 2


Prob. that it rains on the 4th day =
Prob. that it rains on the 5th day =

1
2
1
2

Prob. that it rains on the any day in a 5 day period = 32


Prob. that it rains on exactly 3 days in a 5 day period
1
1
10
5
= 5C3 = 5C2 = =
32
32
32
16
72. (b) Required Probability
= 3C1 (0.4)(0.6)2 + 3C2 (0.4)2(0.6) + 3C3 (0.4)3
= 3(0.144) + 3(0.096) +1(0.064) = 0.784
73. (d) Probability of getting 2 tails + Prob. of getting 1 tail + Prob. of getting no tail
= 8C2

1
256

+ 8C1

1
256

+ 8C0

1
256

37
256

74. (a) Since all possible hypothesis regarding the colour of the balls are equally likely, therefore there could be 3 white
balls, 2 white balls or 1 white ball, initially in the bag.
Required Probability
1
3
1
1
1 4+3+2+1
5
=4 1+4+2+4 =4
=8
4
75. (c) Required probability =

60
100

40
99

40
100

60
99

4800
10099

16
33

PERMUTATION & COMBINATIONS:


76. (d) Total number of ways of filling 5 boxes numbered as (1, 2, 3, 4 and 5) with either blue or red balls = 25 = 32.
Two adjacent boxes with blue balls can be got in 4 ways ie, (12), (23), (34) and (45). Three adjacent boxes with blue
can be got in 3 ways ie, (123), (234) and (345). Four adjacent boxes with blue can be got in 2 ways ie, (1234) and
(2345) and five boxes with blue can be got in 1 way. Hence, the total number of ways of filling the boxes such that
adjacent boxes have blue = (4 + 3 + 2 + 1) = 10.Hence, required answer = 32 10 = 22.
77. (d) There are 32 black and 32 white squares on a chess-board. Number of ways in choosing one white and one black
square on the chess
= 32C1 x 32C1 = 32 x 32 = 1024
Number of ways in which square lies in the same row (white square = 4, black square = 4, Number of rows = 8) =
4
C1 x 4C1 x 8 = 128
Number of ways in which square lie on the same column = 4C1 x 4C1 x 8 = 128
Total number in which square lie on the same row or same column = 128 + 128 = 256.
Required answer is 1024-256 = 768
78. (a) The total number of 4 digit numbers that can be formed = 4!. If the number is divisible by 25, then the last two
digits are 25. So the first two digits can be arranged in 2! ways.
2!
1
Hence, required probability = =
4!
12
79. (b) We consider Vice-Chairman and the Chairman as 1 unit. Now, 9 persons can be arranged along a circular table in
8 ! ways. And Vice-Chairman and Chairman can be arranged in 2 different ways.
Hence, required number of ways = 2 8 !.

PRATHAM: BMS Special Booklet

79

80. (a) At least one candidate out of (2n + 1) candidates can be selected in (2n+1 1) ways.
22n+1 1 = 63 22n+1 = 64 = (2)6 n = 2.5.
Since, n cannot be a fraction. Hence, n = 3.

GEOMETRY:
81. (b) Let BC = x and FB = y = EF = AE
CD = 3y
1
Now, area of CBF = 2 xy
1
2

and area of CBE = x 2y = xy


1

Area of CEF = xy - 2 xy = 2 xy
and area of ABCD = 3xy
Area (CEF )
1 xy

= .
=16
Area (ABCD )

2 3xy

82. (a) Drawing lines DB and AC


Now, DAC = DBC
( A chord of a circle subtends equal angle on all its circumference)
CBD = 30o
DBA = 40o
Now, DBA = ACD = 40o
( A chord of a circle subtends equal angle on all its circumference)
83. (a) Given, AP = 15 cm, AQ = 20 cm and PQ = 25 cm
Let OQ be y cm, Hence, OP = (25 y) cm.
A
In APO, AO2 = (15)2 (25 y)2
.(i)
2
2
2
In AQO, AO = (20) y
.(ii)
15
20
From Eqs. (i) and (ii),
P
x O 25 Q
225 625 y2 + 50y = 400 y2 y = 16
Hence, from Eq. (i),
AO2 = (15)2 (25 16)2 = 225 81
D
Or AO2 = 144 AO = 12 cm
Hence, length of common chord
AD = 12 2 = 24 cm
84. (d) Let the radii of the bigger and smaller circles be R and r respectively.
In the figure AB = AD = R
As ADC = 90o; ABC = 90o and DCB = 90o
ABCD is a square.
BC = R and AC = 2R
and AC = AP + PQ + QC
= R + r + 2r (QC = 2r can be proved in the same way we proved AC = 2R)

r=

21 R
2+1

Rationalizing the denominator, we get r = (3 - 2 2)R


Given R = 2, we get
R = 2(3 - 2 2) = 6 - 4 2

PRATHAM: BMS Special Booklet

80

ANSWERS & EXPLANATIONS


85. (b) In OSQ and ORP
O is the common angle.
OSQ = ORP = 90o (angle between the radius
and the tangent = 90o)
As the two angles are equal, the third angle should
also be equal ie, RPO = SQO
OSQ ~ ORP
SQ
OQ
3
RP = OP = 4

OQ
OQ +PQ

3
4

R
4x
P 4x
I

S
3x
3x Q

II
28 cm

= 4 OQ = 3 OQ + 3 PQ
PQ

OQ = 3PQ OQ = 3

MENSURATION:
86. (c) AC + AB = 5AD
or AC + AB = 5BC

(given)
(i)
(ABCD being a rectangle)
(given)
(ii)

and AC BC = 8
or AC = BC + 8
From Eq. (i), AC + AB = 5BC
BC + 8 + AB = 5BC
[using Eq. (ii)]
AB = 4(BC 2)
(iii)
By the Pythagoras theorem,
AB2 + BC2 = AC2
Expressing AB and AC in terms of BC, we get
BC = 5
AB = 12 and AC = 13
So, area of the rectangle = 5 12 = 60 sq unit
87. (a) Let the radius of the circle be r.
Then, area of the circle is r2.
Now, OR = OT + TR
=r+r
[ TR = OS = r (given)]
= 2r
Now, by Pythagoras theorem, SR = 3 r
So, area of PQRS = 3r2
r 2

Required ratio = 3r 2 = 3
88. (b) The diagonal of the innermost square is 2 unit.
Diagonal of the 7th square = 14
And Diagonal of the 8th square = 16
1
Area of 7th square = 2 142 = 98
1

And area of the 8th square = 2 162 = 128


Their difference = 128 98 = 30

PRATHAM: BMS Special Booklet

81

89. (d) ACB = 90o


( Diameter of a circle subtends 900 at all points of its circumference)
Now,
AB = 15 and AC = 12
By Pythagoras theorem, we find BC = 9
Area of ABC = Area of BAD (all the 3 sides of the triangle are equal)
1
Area of ACBD = 2 AC CB = 108 cm2
2
90. (a) Let the radius of cylinder DEFG be x cm.
OG = x cm
APD and AOB are similar
AP
AO
5
PD = BO = 2
( height of case = 10 cm and radius = 4cm)

AP = 2 x
Height of cylinder = PO = AO AP
5
= 10 x
2
Total surface area of cylinder = 2r2 + 2r h
5

= 2 2 + 10 2 x

= 2 x 2 + 10x

5x 2
2

= 2 10 2 x 2
Now, maximum value of a quadratic equation
4ac b 2
4a
50
.
3

ax2 + bx + c, where a 0, is

3
2

Max value of 10 x 2 is

Hence maximum total surface area of cylinder =

100
3

sq cm

HEIGHTS AND DISTANCE:


91. (c) Let AB be the ladder inclined at an angle of 300 with the wall BC.
After one second the man will be at D such that AD = 2m.
1
AE = AD cos 600 = 2 = 1m
2
Similarly after 2 seconds, the man will be at F such that AF = 4 m
AG = 2m
Thus after every second, the man is approaching the wall by a
distance equal to 1 m, i.e., @ 1m/sec.
92. (a) Since AC 2 = AB 2 + BC 2 + 2AB BC cos(1350 )
AC =
150 2 + 120 2 + 150 120 2 sin 450
250 km.

93. (d) Given APO + CPO = 900


APO = CPO = 450 = BPO = DPO
cos APB = cos =

2 + 2 2
2 .

PRATHAM: BMS Special Booklet

82

ANSWERS & EXPLANATIONS


=
=

2 + 2 + 2 + 2 2
2 .
2

= .
.
1
= sin 450 sin 450 = .
2

Thus, = cos 1
94. (c) Let h be the height of the tower AB.
h
a = tan p
h
b

1
2

= 600

= tan 90 = cot
h

a b = tan p cot p = 1

h2 = ab h = ab
95. (c) Width of the road = 9 + 12 = 21 cm.

COORDINATE GEOMETRY:
8

96. (d) Let A (0, 3) , B(1, 3) and C(82, 30) be the three points.
AB =

10

+ 3

8 2
3

1
9

1+ =

10
3

BC = (82 1)2 + 30 3 2
= 6561 + 729 = 7290 = 27 10
CA =
=

82 0

10 82 2
9

82
3

8 3

+ 30 3
10

Hence AB + BC = CA ABC is a straight line.


97. (c) Subtract 3 from the y-coordinate.
98. (a) 2x + 3y = 5, x y = 10 x = 7, y = - 3
99. (c)
10 2 2 + + 3 2 = 10
64 + 2 + 9 + 6 100 = 0
2 + 6 27 = 0 = 9, 3
26
61
4
100. (b)
=
15 = 10 =
+2

23

FUNCTION:
101. (d) Given function
= 1 + 2 + 3 + . . + = 2 ()
Given, 1 = 3600
For n = 2, 1 + 2 = 22 (2)
22 2 2 = (1)
(1)
2 = (22 1)

PRATHAM: BMS Special Booklet

83

For n = 3, 1 + 2 + 3 = 32 (3)
(1)
1 + (22 1) = 32 3 (3)
1

1 + (22 1) = 3 (32 1)
1 1+

1
22 1

= (3)(32 1)
22

3 = 1 22 1 32 1
Similarly,

22 32 42 . 82
22 1 32 1 4 2 1 (92 1)

9 = 1

Therefore, f(9) = 80.


1+x
102. (b) f(x) = log
and f(y) = log
1x

f(x) + f(y) = log

= log
= log
= log

1+x
1x
1+xy
1+xy
x +y
1+xy
x +y
1
1+xy

1+

1+y
1y

x +y
1+xy
x +y
1
1+xy

1+y
1y
1+x
1+y
+ log 1y
1x
1+x+y+xy
= log 1xy+xy

1+

=f

[Divide the Nr. And Dr. by (1 + xy)]

x+y
1+xy

103. (b) It is not linear in x and y, thats why option (a) is neglected. It also cant be exponential. By substituting x and y in y
= a + bx + cx2 we see that it gets satisfied.
104. (d) f(x + 1, y) = f[x, f(x, y)]
Put x = 0, f(1, y) = f[0, f(0, y)] = f[0, y + 1]
=y+1+1=y+2
Put y = 2, f(1, 2) = 4
x3
x3
105. (b) fog (x) = f{g (x)} = 2 = 2 2 + 3 = x
gof(x) = g{f(x)} = g(2x + 3) =
fog(x) = gof(x)

2x+33
2

=x

SET THEORY:
106. (b)

107. (a)

15

Thus, percentage failed = 100 = 15%.

PRATHAM: BMS Special Booklet

84

ANSWERS & EXPLANATIONS

108. (c)

19 Guests like Tea and Coffee both.


109. (c) No. of people wearing a hat or a sweater or both = 36 - 6 = 30

No. of people wearing both the hat and the sweater = 12 or


n(H) + n(S) - n(H S) = n (H S)
18 + 24 n(H S) = 30
n(H S) = 12
110. (a) Consider 1 = {1, 2, 3, 4, 5}. This does not contain a multiple.
T2 = 2, 3, 4, 5, 6
T3 = 3, 4, 5, 6, 7
T4 = 4, 5, 6, 7, 8
T5 = 5, 6, 7, 8, 9
T6 = {6, 7, 8, 9, 10}
All these do contain multiples of 6. T7 once again does not contain a multiple of 6. Thus, one part out of every 6 taken
in a sequence will not contain a multiple of 6.
96
Therefore 6 = 16 sets will not contain multiples of 6.
96 - 16 = 80 will contain.

TRIGONOMETRIC RATIO:
111. (a) b (a2 1)
= (sec + cosec ) [(sin + cos )2 1]
1
1
= cos + sin (2 sin cos )
=

sin +cos
sin

2 sin cos

= 2(sin + cos ) = 2a.


sin
4
112. (d)
= . Therefore,
cos

113. (a) tan [(A + B) +

+

tan + +tan
C] = 1 tan + tan

+
+

+

.
.

+ +

=0

PRATHAM: BMS Special Booklet

85

[ tan A + tan B + tan C = tan A tan B tan C]


A + B + C = , i.e. A, B, C is a triangle.
2

114. (b) sin = 3 cos =


1

cos 60 0 +sin 60 0

115. (c) cos 60 0 sin 60 0 = 21


=

1+ 3
1 3

4+2 3
13

1+ 3
1+ 3

= 2+

3
2
3
2

5
3

= 0.745

DIFFERENTIATION:
2

116. (b) Given that, y = sin x 2 , z = ey , t = z


2
dy
dz
Now, = cos x 2 .2x, = ey .2y = 2yz
dx
dy
dt
1
1
And dz = 2 z = 2t
dt
dt dz dy
dx = dz . dy . dx
1
2xyz
= 2t .2yz .2x cos x 2 = t cos x 2
sec +tan
(sec +tan )2
117. (c) Let y =
=
sec tan
(sec 2 tan 2 )
2

= (sec + tan )
On differentiating wrt x, we get

= 2 (sec x + tan x)(sec x tan x + sec2x)

= 2 sec (sec + tan )2


118. (b) Given that
= log
(log ) = log log
On differentiating wrt x, we get
1
(log ) = log = ( log )1
cos

119. (c) We have, y = 3x 2


On differentiating wrt y, we get

sin
1=3 +
.

........ (i)

1
sin
2

3+

Again, Differentiating wrt y, we get

3 2
cos 2
sin 2
+
+
. 2
2

2
2
sin
cos
1
3 + 2 2 + 2 . sin 2
3+

0=
=

sin 2
2 cos
+ (6+sin )2
2
2
sin 2
2 cos
3+
=
2
2
(6+sin )2

= 3+

120. (b) y = a cos (log x) b sin (log x)

PRATHAM: BMS Special Booklet

2 =

4 cos
(6+sin )3

86

ANSWERS & EXPLANATIONS

( sin (log ))

cos (log )

=-

( sin (log )+ cos (log ))

= ( sin(log ) + cos(log ))
2

cos (log )

2 + =
2

sin (log )

2 2 + + = 0

INTEGRATION:
dx

121. (b) Let I =


=
=
=

2 2
dx

2 2 + 2 + 2
dx
2 2
1
sin
+

122. (d) Let =

2
2
2

2 =

= 2
Thus, = 2
123. (a) If () =
124. (b) Let I = e
=
Put

x2+ 1

1
2

dx

= 1

= , = ()

x 2+ 1
e

dx

2+ 1

x2+ 1

1
2

=
2+ 1

= = + =
+
sin x
125. (b) We have, I =

2
2
=

sin x
cos 2 cos 2

sin sin

Put cos x = t - sin x dx = dt


dt

I== cos1 cos +


2
2
cos

I = cos 1 (cos sec ) +

APPLICATION OF DERIVATIVE:
126. (a) Let y =

On differentiating wrt x, we get = (1 + log x)

For increasing function, > 0

1 + > 0 1 + log > 0


1
1
log e > log e >
1

Function is increasing when x >


127. (a) Given, curve = 2 4 + 3

PRATHAM: BMS Special Booklet

87

Now, differentiating wrt x, we get


dy
= 2x 4 = 2 x 2
dx

...... (i)

dy

Here at x = 2,
=0
dx
i.e., for the given curve only one tangent is possible because slope of tangent parallel to x-axis is zero.
128. (a) Let y = log x x

1
= 1
2

And 2 = 2
For maximum and minimum values of y,

1
= 1=0

=1
=1

For x = 1,
2
2

=
Thus, the value of the given function is maximum for x = 1.
So, the maximum value of the function
=log (1) 1 = -1
129. (d) We have,
=
. (i)
On differentiating Eq. i wrt x, we get

Again, differentiating, we get


d2y
dx 2

= f ex ex . ex + f ex . ex
2

2 = 2 +

3
2 4

130. (d) For option (a) and (b), by graph. Hence, sin x is differentiable at x = ,

For option (c), 2 = 4


Which is differentiable at origin.

PRATHAM: BMS Special Booklet

88

ANSWERS & EXPLANATIONS

DIFFERENTIAL EQUATION:
dy

131. (c) Given, (1 x2) dx xy = 1


dy

IF = e

p dx

dx 1x 2 y = 1 2
This is a linear equation, comparing with the equation
dy
x
1
+ Py = Q P = 1x 2 and Q = 1x 2
dx
1

=e

x
dx
1x 2

IF = e2log (1x ) = 1 x 2
132. (c) Equation of parabola is y2 = 4ax
On differentiating wrt x, we get 2 yy = 4a
Again differentiating, we get
2yy + 2(y)2 = 0 yy + (y)2 = 0
133. (c) Given equation,
dy
= 2y 2x 2y dy = 2x dx
dx
On integrating both sides, we get
2y
2
1
=
1
log 2
log 2
y
x
2
2
log 2 = log 2 + C1
y
x

+
1

2 = 2 + C1 log 2
1
1
x y = C1 log 2 =
2

dy

sin x

134. (a) The given differential is dx + y cos x = cos x

IF = e

sin x
dx
cos x

= elog sec x = sec x

135. (a) The given equation can be rewritten as,


On integrating the given equation,
dy
dx

d2 y
dx
dx2

d2 y
dx2

= sin

sin x dx + C

= ( cos x) + C = cos x + C
Again integrating, we get
dy = cos x dx + C dx + D
y = sin x + Cx + D

VECTORS:
136. (a) Let O be the origin
BE + AF = OE OB + OF OA
+

=
+
2

+ 2 + 2
+ 2
2

= OC

OA+ OB
2

=

=
A

PRATHAM: BMS Special Booklet

B
89

137. (a) Given that , OA = +


+ 3
OB = 3+ 4+ 7
And
OC = y 2 5
Since A, B, C are collinear, Then, AB = BC
2+ 4 + 4= (y 3) 6 12
On comparing the coefficient of , ,
, we get

2=(y 3)
. . . . . .(i)
4 x = - 6
. . . . . (ii)
1
And 4 = - 12 = - 3
. . . . . (iii)
On putting the value of in Eqs. (i) and (ii), we get
y = - 3 and x = 2
138. (b) a = i + 2j 3k and b = 3i j + k
a + b = i + 2j 3k + 3i j + k
= 4i + j + ( 3) k
And a b = i + 2j 3k 3i + j k
= -2i + 3j (3 + ) k
(a + b) is perpendicular to (a b).
(a + b) (a b) = 0
{4i + j + ( 3) k} {-2i + 3j (3 + ) k} = 0
-8 + 3 + (32 2) = 0
4 2 = 0
= 2
139. (b) Since, a, b and c are mutually perpendicular vectors.
Let a = 10i, b = 10j, c = k
2a + 2b + 40c = 20i + 20j + 40k
= 20(i + j + 2k) = 20 6

140. (a) Given that, a and b are two unit vectors inclined at an angle 3
i.e.,

a = b = 1 and =

a+b =

a 2 + b 2 + 2 a b cos

= 1+1+211

1
2

= 3
a+b = 3 > 1
a+b >1

3D:
141. (c) The given equation can be rewritten as
5x 3y 6z
x
y
z
+
=1
+
=1
60 60 60
12 20 10
Hence, the intercepts are (12, -20, 10).
142. (a) Let the points be P = (4, 3, -5) and Q = (-2, 1, -8)
Now, PQ = (2 4)2 + (1 3)2 + (8 + 5)2
= 36 + 4 + 9 = 7

PRATHAM: BMS Special Booklet

90

ANSWERS & EXPLANATIONS


DCs of line PQ is
y y
x x
z z
l = 2PQ 1 , m = 2PQ 1 , n = 2PQ 1
6

l = 7,m = 7,n = 7
143. (d) Equation of plane passing through (1, -3, 1) and the direction cosines of its normal (1, -3, 1) is
1(x 1) -3(y + 3) +1(z 1) = 0
x 3y + z - 11 = 0
x
y
z
11 11/3 + 11 = 0
(intercept form)

The above plane intercept the x-axis at a distance of 11.


144. (b) As we know, direction cosines of y-axis are (0, 1, 0). If the line is parallel to y-axis, then the direction cosines of
A (4, 7, -8)
the line are (0, 1, 0).
145. (b) Let the coordinates of D be (x, y, z).
For x-coordinate,
2
2x+14
1 = 1+2
G (1, 1, 1)
1
x = 2
1
For y-coordinate,
2 y+1 7
C
B
D (x, y, z)
1=
y = 2
1+2

And for z-coordinate,


2 z+1 (8)
1=
z=
1+2

Coordinates of D are

11
2
1
11
2 , 2, 2

LOGARITHMS:
146. (c) loga
=

log
=
log

log
+log
=x
log
log
log
log + 1 = log =
log
1
log = 1
log
log
logb
=
=1+
log
log
1

= 1 + 1 = 1

147. (b) Given expression


1
= 2 log10 52 2 log10 3 + log10 (2 3 3)
= log10 5 2 log103 + log102 + log1032
= log105 2 log103 + log102 + 2 log103
= log105 + log102 = log10 (5 2)
= log1010 = 1
148. (b) log7 log5 ( + 5 +
)=0

log5 ( + 5 +
) = 70 = 1
x + 5 + x = 51 = 5 2 = 0 = 0
149. (c) log2 [log7 (2 + 37)] = 1
Use logp = =

PRATHAM: BMS Special Booklet

91

2 = log7 (2 + 37)
49 = 2 + 37 2 12 = 0
4 + 3 = 0 = 4
2

150. (d) S = log m + log + log 2 + log 3 + .


2

= log
2 1

+1

= log

+1
1

= log

/2

DATA INTERPRETATION:
151. (c) Let marks obtained by Dipan in English group paper II be x, then group average of Dipans
Physics, Chemistry and Biology group = 98
Maths = 95
Social Science = 95.5
Vernacular group = 95
96+x
English group =
2

Now, final score =


767+(96+x)
10

98+95+95.5+

96+x
+ 95
2

5
863+x
or
10

96 =
=
960 = 863 + x or x = 97
152. (a) Dipan wins the prize for the highest average in social science group, i.e., 96.
153. (d)
Paper selected for
Name of the students
Revised group average
improvement
Vernacular group I or II
Ram
97
94
Agni

Vernacular group I 82

96.5

Pritam

Social Science History-83

97.5

Ayesha

Social science Geo.-93

97.5

Revised final score


97+97+95.5+97+97
= 96.7
5
97+99+95.5+96.5+92.5
= 96.1
5
97+98+97.5+91.5+94
= 95.6
5
97+98+97.5+95+97
= 96.9
5

Clearly, final score of Ayesha is maximum.


154. (a) The only boy getting 95 in at least one of the subjects of the group among all the groups is Dipan.
155. (a) Revised final scores of all the four students after revision in group scores of Social Science.
97+98+100+91.5+94
94+100+100+91+94.5
Pritam :
= 96.1
Joseph :
= 95.9
5
97+99+100+87.5+92.5

156.
157.
158.
159.

5
97+99+100+91.5+91.5

Agni :
= 95.2
Tirna :
= 95.8
5
5
Their score in decreasing order is : Pritam, Joseph, Tirna, Agni.
(c) 23% of 2000 = 460
(a) Contribution of foreign investment approval for the year 1996 in case of SAARC, ASEAN and EU
= (2 + 3 + 23)% = 28%
28% of 8000
= 2240
(d) Only III is correct.
(c) From the bar chart, in 1996 foreign investment approved is 2000 approximately. At 20% growth, the foreign
investment inflows in 1997 = 2000 1.2 = 2400. Now, if the proportion of inflows for the year 1997 was as per
the pattern followed by the proportion of the foreign investment inflows for the year 1996, then the contribution of
NAFTA in the foreign investment inflows = 35%. The foreign investment inflows from NAFTA

PRATHAM: BMS Special Booklet

92

ANSWERS & EXPLANATIONS

160.
161.
162.
163.
164.
165.
166.
167.
168.

= 35% of 2400 = 840.


(c) The commodities that exhibited net overall increase were onion, rice, egg and chillies. Therefore, the
commodities that exhibited net overall decrease were dal and edible oil.
(d) The commodities that experienced a price decline for and two or more consecutive years were rice, egg, onion,
dal and chillies.
(c) The given condition is satisfied only for the commodity onion.
(c) Total protein = 20% of total body weight and skin and muscular protein = 30% of total proteins. Therefore,
3
the percentage of skin and muscular protein as a fraction of the total body weight = 20% and 30% = 6% = 50.
(a) Required ratio = 20:50 = 2 : 5
(d) Data are not adequate to answer the question.
20
1
(a) Required proportion = 100 = 5
(b) Draw a diagonal line from bottom left point to top right point. All companies lying above this line have profit in
exceeds of 10% of turnover. From the graph there are 7 companies which have the profit exceeding 10% of
turnover.
(c) From the graph there are 2 steel companies with a turnover of more than 2000 and profit less than 300.

UNIT 2: LOGICAL REASONING


CODING AND DECODING:
1.
2.

(a) 9$7% (condition III follows).


(c) 36 18 12 + 44 30 = 38

Solution for Question - 3: Each constant in series changes to next letter while each vowel changes to two letter next.
3. (a) COWQ
4. (c) bulldozer
Solution for Question - 5
See = na,
to = di, hope = so,
you = re,
come = ge,
the/party/ = fi/zo,
please = le,
5. (c)
6. (d) Girls fathers only son-Girls brother. Daughter in law of girls grandmother can be their mother, or maternal
uncles wife, i.e. aunt. So relation cannot be determined.
7. (c) D is the son of B, B is the brother of C and A is the father of C. This means that B is the father of D and A is the
father of B. So, A is the grandfather of D. Since F is the spouse of A, so F is the grandmother of D.
8. (a) As explained above, B is the son of A and F is the spouse of A. So, B is the son of F.
9. (b) A + B # C D means A is the mother of B, who is the father of C, who is the brother of D, i.e. A is the mother
of Ds father. So, A is Ds grandmother.
10. (c) A is aunt of E means A is the sister of the father of E i.e. A * B # E. Clearly, the code in (c) indicates the same.

ARRANGEMENTS AND PUZZLES:


A
B
X
Y
Z

Floor
2nd
1st
Ground
3rd
4th

PRATHAM: BMS Special Booklet

Bank
UCO
PNB
HDFC
SBI
ICICI

Car
Indica
Elentra
Santro
Indigo
Ikon

93

11.
12.
13.
14.
15.

(c) X owns Santro car.


(d) A lives on the second floor.
(a) B lives on first floor.
(b) A owns indica car.
(d) Mr. Z has Ikon car with him.

Solution for Questions 16 20: Clearly in a circle the arrangement is as shown:


Priya
Amit
y

kamla
o

60 60o
o
o
60
60
o

60
Pritam

16.
17.
18.
19.
20.

30o

30o

Rohit
Gaurav
Manish

(c) Gaurav is sitting between Rohit and Manish.


(d) Gaurav is not at equal distances from Rohit and Pritam or Amit and Kamla or Manish and Pritam.
(d) Gaurav is three places left or four places right of Priya.
(d) The angle between Gaurav and Manish in clockwise direction is 300.
(d) Clearly, all the statements do not follow from the diagram above.

Solution for Questions 21 24: On the basis of the information given in the question we can derive this table:
Floor
II
III
IV
21.
22.
23.
24.

Flats
4
3
6

Occupied
2
3
3

Unoccupied
2
0
3

Occupants
Lawyer, Manager
Teacher, Manager, Lawyer
Teacher, Manager, Doctor

(b) Three flats are occupied on the IV floor.


(a) There are three flats on the III floor.
(d) Lawyer and Manager live on the II floor.
(c) Teacher and manager are neighbours of the other lawyer living on the III floor.

Solution for Questions 25 29: We are given many clues. On summarizing all these clues we get the following
information.
Person
Vehicle
Profession
A
II
Teacher
B
I
Engineer
C
III
Doctor
D
II
Doctor
E
III
Teacher
F
II
Engineer
G
I
Teacher
25. (c) C travels in vehicle III.
26. (b) There are four ladies in the group A, C, F and G.
27. (b) F is an engineer by profession.

PRATHAM: BMS Special Booklet

Sex
F
M
F
M
M
F
F

94

ANSWERS & EXPLANATIONS


28. (d) Since F is an engineer, so, F-female Teacher is the incorrect combination.
29. (a) A, E and G are the three teachers.

GROUPS &CONDITIONALITIES:
CONDITIONALITYS:

30. (d) Option (a) is not valid as no lecturer from electrical department is there. Option (b) is not valid as no lecturer
from civil department is there. Option (c) is not valid as no lecturer from mechanical department is there. Option (d)
complies with all the conditions and hence is a valid option.
31. (d) According to the question, following three conditions must be complied withL cannot go with FG (Considering only female delegates)
... (1)
The delegation must have minimum three professors
...(2)
Representation from every department must also be there
...(3)
Now option (a) is not valid as it violates condition (1)
Option (b) violates condition (1), as all the members have to be females.
Option (c) also violates condition (1).
Option (d) complies with all the conditions.
32. (b) Conditions given in the question can be summarised as below :
Only civil and electrical departments must be used
(1)
Committee must have two professors and two lecturers
....(2)
There should be at least two female members in it
...(3)
Option (a) complies with all the conditions hence follows.
Option (b) is not valid as it has only one female member.
Option (c) and (d) comply with all the conditions and hence follow.
Therefore, option (b) is our answer.
33. (b) Conditions given in the question can be listed as below : only mechanical department people are used in forming
the committee ... (1)
One must be a lady out of four members
... (2)
I must be one of the members
... (3)
I cannot work with L or N
... (4)
M cannot work with N or J
(5)
Option (a) is not valid as it violates condition (5)
Option (b) is valid as two ladies J and K are there.
Option (c) is not valid as it violates condition (2)
Option (d) is not valid as it violates condition (5)
Hence option (b) is our answer as it complies with all the conditions.
34. (d) From condition (I) A play, first game with D. Team B will play first game with E because its possible combination
of first game is ruled out with teams A, D, C, F as per conditions (I) and (III). And team C will play its first game with
team F. None of the options is the right combination of first game, hence options (d) is the right answer.
35. (c) Team B cannot play second game with E and C as per condition II and also its possibility with F is ruled out as per
condition I. However, its combination with either team A or D can be formed for second game. But A is playing with
F. therefore B and D play.
36. (d) Since each team must play against all other teams, therefore, each has to play 5 games.
37. (d) If D wins 5 games, it means all the teams have lost at least 1 game. The only alternative definitely stating this is
option (d).
38. (d) No information in the condition is given about the order of play for the last set of games.
39. (d) None of these

PRATHAM: BMS Special Booklet

95

40. (b) Apart from box BC, box with label BB contains ball. It means that container BB has been labeled BC. From which
we conclude that another item will also be a ball.
41. (b) On the basis of information given in the question, we conclude that
P C is labeled as B B
P P is labeled as C C
C C is labeled as B C.
If one of the remaining boxes is opened, we shall have at least one of the objects as pin.
42. (c) On the basis of the information given in the question, we conclude that
P P is labeled as C C
C C is labeled as P P
P C is labeled as B B
B C is labeled as P C
Since B C will be labeled as P C, it will not contain two balls.

DIRECTIONS:
43. (c) The movements of Radha are as shown in figure. Clearly, Radhas distance from the starting point O = OD =
(OC CD) = (AB CD) = (14 4) m = 10 m.
C 4m

7m

O
7m

A
B
14 m
44. (d) As shown in figure, the man initially faces in the direction OA. On moving 135o anti-clockwise, he faces in the
direction OB. On further moving 180o clockwise, he faces the direction OC, which is South-west.

45. (b) The villager moves from his village at O to his uncles village at A and thereon to his father-in-laws village to B.
Clearly, OBA is right-angled at B. So, 2
= 2
+ 2
, 2
= 2
2
= (25 16
= 9 km = 3 km. Thus, B is 3 km to
the east of his initial position O.

46. (b) Clearly, DC = AB + FE. So, F is in the line with A. Also AF = (BC DE) = 5 m. So, the man is 5 metres away
from his initial position.

PRATHAM: BMS Special Booklet

96

ANSWERS & EXPLANATIONS


47. (b) As shown in figure, the river flows eastwards from A towards B, turns left and follows a semi-circular path to
reach C where it turns left and flows eastwards towards D.

VENN DIAGRAM:
48. (b) The triangles are:

EAB; FBC; GCD; HAD; ADC; BCD; ABC; ABD; MAB; MBC; MCD; MAD; EAI; EBI; BFJ;
FCJ; GCK; GDK; HDL; HAL; AHM; DHM; BFM; CFM; ALM; MLD; BMJ; MCJ;
So, there are 28 triangles.
49. (b) The figure may be labeled as shown.

The horizontal lines are IK, AB, HG and DC i.e. 4 in number.


The vertical lines are AD, EH, JM, FG and BC i.e. 5 in number.
The slanting lines are IE, JE, JF, KF, DE, DH, FC and GC i.e. 8 is number.
Thus, there are 4 + 5 + 8 = 17 straight lines in the figure.
50. (d) Unmarried educated sports woman.
51. (b) (2)2 + (4)2 = 20, (3)2 + (9)2 = 90, Therefore, (1)2 + (5)2 = 26.
52. (b) 915 364 = 551 and 789 543 = 246, Similarly, 863 241 = 622.

SYLLOGISM:
53. (d)

None Follows.

PRATHAM: BMS Special Booklet

97

54. (c)

Only III follow


55. (a) No tingo is a bingo and all jingos are bingos, therefore No jingo is tingo.
56. (d) Smoking creates ill will + ill will causes pollution
Smoking cause pollution. Hence, BDA are logically related.
57. (b) Clearly, AEB are logically related.

RANKING & SERIES:


58. (d) Vilas is 26th from left and Kewal is 10th places to the left of Vilas. So, kewal is 16th from left. Now, there are three
boys between kewal and Aditya. So, Aditya may be 12th or 20th from left. Since the exact position of Aditya can not
be ascertained, so the given data is inadequate.
59. (b) Amisha is 22nd from the top and Anuja is 5 ranks below Amisha. So, Anuja is 27th from the top. Also, Anuja is 34th
from the bottom. So, number of students passed = (26 + 1 + 33) = 60; Let the number of students passed and the
number failed be 4x and x respectively. Then, 4x = 60 or x = 15. Hence, number of students in the class = (60 +
15) = 75.
60. (c) There are two possible arrangements:
31

61.
62.
63.
64.
65.
66.
67.

16

N
N

16
31

I
II
But Priya is nearer than Natasha to the right end of the row, so only arrangement II follows.
Number of girls to the left of Natasha in II = 31 (1 + 16) = 14. Clearly, Natasha is 15th from the left end of the
row.
(c) Two = 29, 21
(d) 5
(a) SUM
(b) B I G
FOR
I J K L M N O = Five
(b) 20th March.
(a) Either 18th or 19th of February.
(c) Digits are = 6, 1, 3 Therefore Perfect square = 361 = 192 = 9

DATA SUFFICIENCY:
68. (b) From statement I: Aditya is the son of Pinki.
Statement III: Pinki is daughter-in-law of Mayank.
Therefore, Aditya is grandson of Mayank.

PRATHAM: BMS Special Booklet

98

ANSWERS & EXPLANATIONS


69. (d) From I, II and III we cannot find the answer.
70. (e) Both the statements are required. Vickys birthday would be on 14th Jan
71. (d) From I

From II

We cant find gender of T

Now T is female
So from I and II we can find the answer.
72. (c) using statement I If the time is 9 oclock now, then after 30 minute i.e. at 9:30 the angle between the minute hand and hour hand
cannot be 90o. So, now the time is not 9 oclock.
Using statement II If the time now is 9 oclock, then 15 min before the hour and minute hand of the clock can never coincide with each
other. Instead they will have an angle of 7.5o. So, the time now is not 9 oclock.

STATEMENT AND CONCLUSION:


73. (d) I is too far fetched. II is irrelevant. So, none follows.
74. (d) None follows.
75. (c) Supposing numbers of Sandal trees and Ashoka trees are 50 each. Therefore, total will be 100. Out of which 75
trees are old. Hence, it is confirmed that at least one-half of the Ashoka trees are old.
76. (d) It is given in the statement (1) that only students can participate in the race. Hence, it directly implies that all
participants in the race will be students.
77. (c) In view of the top management order, either the remaining manager will resign or the management will
terminate the service of one manager. Hence, either I or II follows.

VISUAL REASONING:
78. (a) In one step, the symbols square, circle and + sign, interchange positions, in an ACW direction and in the next
step, all the four arcs get laterally inverted.
79. (e) In each step, two 'L' shaped symbols are added to the figure. Also, each one of the odd numbered i.e. first, third,
fifth..,elements rotates 90ACW while each one of the even numbered i.e., second, fourth, sixth.elements
rotates 90CW in each step.
80. (e) In one step, the figure rotates 45ACW and a half-leaf is added at the CW-end of the existing leaves. In the next
step, the figure rotates 90ACW and a half-leaf is added at the CW-end of the existing leaves.
81. (d) In one step, one side is removed from each one of the two upper elements and in the next step, one side is
removed from each one of the two lower elements. The removal of sides in any element goes on in a CW direction.
82. (d) Similar figure repeats in every second step.

PRATHAM: BMS Special Booklet

99

CUBES & DICES:

83. (c) The symbols which are adjacent to


are
, ,
,
. Hence, the remaining symbol ? will be opposite to
.
84. (d) Colours adjacent to green colours are Black, Brown, Red and White.
85. (a) Black is opposite to Red. Hence, if Red is at the bottom then black is on the top.
86. (b) Combining B and F, C can clearly be ascertained.
87. (d) Letters M, Y and U are adjacent to letter W. So, they will not be opposite to letter W. Hence, letter S is opposite
to Letter W.

INPUT - OUTPUT:
Solution for Questions 88 - 92: After careful study of the given input and various steps of rearrangement, we may
conclude that in the first step one word which comes first in dictionary is placed at the first position from the left and
other terms are kept as they are. In the second step , the highest number is placed at the second place keeping rest of the
terms as they are and the same process is continued till the rearrangement ends.
88. (c) Step II : cut 97 38 end for 29 46 down
Step III : cut 97 down 38 end for 29 46
Step IV : cut 97 down 46 38 end for 29
Step V : cut 97 down 46 end 38 for 29
89. (b) Step I : car 17 vas tiger 92 87 like 52
Step II : car 92 17 vas tiger 87 like 52
Step III : car 92 like 17 vas tiger 87 52
Step IV : car 92 like 87 17 vas tiger 52
90. (a) Input : zeal for 49 31 high 22 track 12
Step I : for zeal 49 31 high 22 track 12
Step II : for 49 zeal 31 high 22 track 12
Step III : for 49 high zeal 31 22 track 12
91. (d) Input : 19 feet 34 28 dog bag 43
Step I : bag 19 feet 34 28 dog 43
Step II : bag 43 19 feet 34 28 dog
Step III : bag 43 dog 19 feet 34 28
92. (d) Cannot be determined.
Solution for Questions 93 97: All words become arranged in alphabetical order (dictionary format).
93. (e)Do not imitate others work.
This statement will be arranged in only one statement. So there is no 3rd step.
94. (a) Adequate safety arrangement are must for kids. So it is arranged in 5 step so second last step is 4th step which is
adequate are arrangement for kids safety must.
95. (b) Marcus Brauchli gives inaccurate information. So total required steps are 4.
96. (d) In last step it should be arranged in alphabetical order means cannot even free India Indian leaders most
visualize.
97. Cannot be determined.

PRATHAM: BMS Special Booklet

100

ANSWERS & EXPLANATIONS

STATEMENT & ASSUMPTIONS:


98. (a) Since the person is asked to check only the availability, hence only assumption I is implicit.
99. (a) I is an implicit assumption. II is too farfetched.
100. (b) II is an implicit assumption.
101. (e) Both are valid assumptions.
102. (d) (i) VALID:
It must have been assumed that people want to sell an old car that is why the advertisement was given.
(ii) VALID:
It must have been assumed that the advertisement will be read by needy people (in need of selling or buying a car)
that is why it was published.
(iii) VALID:
The people are dealing in such cars assuming that used cars may not be useless. Some usefulness is desired from
used cars.

STATEMENT & COURSE OF ACTION:


103. (c) I and III are possible courses of action. Closing down schools doesnt help the case.
104. (d) Stopping the sales of chloroquine is not a solution. The remaining two are possible courses of action.
105. (b) II is a possible course of action.
106. (e) None of the three follows.
107. (b) Only II is a feasible course of action.

STATEMENT & ARGUMENT:


108. (d) Both are weak as they are mere opinions and not prevailing notions of truth.
109. (d) Both are weak. I is an opinion which states that this matter can be solved only by legal means, II predicts a result
which may or may not follow.
110. (e) I is an established fact. II predicts a negative result
111. (b) I is weak as it is an opinion, II is strong as it is an established fact
112. (a) I is strong as it is an accepted truth, II is weak because it is not directly related to the statement

ASSERTION & REASONING:


113. (a) Both (A) and (R) are true because children need protection otherwise they might go on the path where their
parents went. Without parents, they are punished for the crime which their parents did. But (R) is not the correct
explanation of (A).
114. (c) (A) is true while (R) is false because it is the condition at plus zero level temperature.
115. (a) (A) is true but (R) is false and not related to (A).
116. (d) (A) is false but (R) is true because stenographer must know typing.
117. (d) (R) is true but (A) is false because marriage is a social obligation.

PRATHAM: BMS Special Booklet

101

INFERENCE FROM PASSAGE:


118. (c) Data inadequate.
119. (a)
120. (b)
121. (c) No data regarding the statement is given. Hence, data inadequate.
122. (b)
123. (a) The passage talks about western technological model based on heavy use of energy which is an inherently high
toxic model. Hence the inference that industries using heavy energy create more pollution is definitely true.
124. (a) The passage mentions phrases like pollution growing faster than economy and rapidly shows that pollution
has gathered momentum in recent past. Inference is definitely true.
125. (c) There is no data regarding pollution levels in the western world. The passage talks about use of western
technological model in India which may or may not be used in Western countries. Hence, data inadequate.
126. (a) The last line of the passage talks about careful choice of technology which would control the pollution, hence the
inference is definitely true.

UNIT 3: VERBAL ABILITY


SYNONYMS:
1. (d)

2. (c)

3. (c)

4. (a)

5. (d)

6. (d)

7. (c)

8. (c)

9. (d)

13. (d)

14. (b)

15. (c)

16. (a)

17. (a)

18. (b)

19. (b)

10. (a)

ANTONYMS:
11. (a)

12. (b)

20. (c)

ANALOGIES:
21. (b) Raucous means loud, it is a synonym of Dulcet. Same as crazy / insane.
22. (b) Second word is a follow-up of first one. Reaction comes after action and Assail (attack) is followed by defence.
23. (a) Malapropism is using words in wrong places. Anachronism is placing something in wrong period of time.
24. (b) Gourmet is person who has a deep knowledge about food. Likewise Connoisseur is a person who has a sense
of judgement about Art.
25. (c) Both are pair of antonyms.
26. (a) The relationship between the two words here is that of intensity. A saunter is a leisurely stroll and a perambulate
is a leisurely walk. Thus, the correct choice is (a).
27. (d) Vandalism is the desecration of property just as slander is the desecration of a reputation.
28. (c) An artist creates a canvas just as a composer creates a symphony.
29. (b) Sword goes into scabbard and any beverage gets poured into a cup.
30. (c)

IDIOMS:
31. (c)

32. (d)

33. (b)

PRATHAM: BMS Special Booklet

34. (b)

35. (b)

36. (b)

37. (c)

38. (a)

39. (c)

40. (a)

102

ANSWERS & EXPLANATIONS

SENTENCE COMPLETION:
41. (b) The use of the word groveled in the question suggests that the blanks should contain words with negative
connotation. Thus option (a), (c) and (d) get eliminated. Option (b) is the best.
42. (b) The second blank needs as adjective for nerves which are no longer working properly. So damaged is the best
option. Also note that nerves cannot be detrimental or complex or involuntary.
43. (c) In general, a loose fiscal policy would lead to an increase in demand. So assess, outstrip and restrain do not fit
the first blank as well as the word stimulate does. So option (c) is the correct option.
44. (d) Going by option makes choice easier. Reports or books are not written for tools. Practices are not traded or dye
but they fade with time.
45. (c) A personality always emerges rather than arise or comes, and when two things are together, they combine.
46. (d) Option gives us clear choice. Being subordinate, does not given boast. Talking about current cannot be
ancestors. World do not move with cliques, which mean a gang of some people. So option is (d).
47. (d) Touts means middle men and it is clear that poor people are not once taken care of.
48. (c) Option (a) and (c) could be in choice, rest is rejected as does not go with words. Out of available tow options,
prices be spiraling rather being escalating.
49. (d) Morals go with manners and theme should well and suits rather than a story.
50. (b) Sounds are audible as symbols are visual.

SENTENCE IMPROVEMENT:
51. (d) Focusing on forging is a better option which mean to create lasting relationship.
52. (b) An important clause is always noted rather than being noticed.
53. (b) A good novel can be a good combination of realistic details with romantic temperament.
54. (c) The sentence if followed by option (c) gives that exercising any other option will lead to failure.
55. (a) .... in her option, she thought and passed on are redundancy errors, incorrect usage. So answer is (a).

SENTENCE CORRECTION:
56. (a) No error. Poor product quality is singular, s the singular noun it must also
57. (d) This corrects the double negative (hadnt hardly) and also uses those with kinds correctly.
58. (d) This is an incomplete sentence since the verb is missing. Option (d) provides the verb (is recognised) and presents
the only complete sentence among the options.
59. (e) None of the sentence are correct.
60. (a) Choice (a) is correct because the subject of verb had must be who, not whom. Which is choice (e) should not be
used to refer to a person.
61. (a) Already is an adverb; all ready is an adjectival construction. Allready is a misspelling. Choices (d) and (e) do not
convey the thought of the sentence.
62. (c) This is also an error in agreement: Kind is singular and requires a singular modifier (this).
63. (d) This choice eliminates the error in parallel structure.
64. (b) The preposition with is needed to complete the phrase has gone hand in hand with. Choice (c) unnecessarily loses
the parallel structure.
65. (a) The correct answer is Option (a).
66. (b) Referred back is redundant. The prefix re means back.
67. (d) The either..... or structure should always be as close as possible to the things the they are attached to.
68. (a) The original sentence is correct and needs no change.
69. (d) Despite should be used as a preposition, not as a word joining clauses.

PRATHAM: BMS Special Booklet

103

70. (c) Entrust with is the correct phrase. This eliminates option (a) and (d). The noun upliftment has to be used
instead of the verb uplift.

JUMBLED SENTENCES:
71. (c) C is the opening statement as it introduces the topic of the paragraph i.e., Bushs mission to restore power to
the presidency. Statement E presents the problem that he must tackle i.e., a legislature of second guessers and time
wasters. Hence, CE. Further A tells us what Bush did not tackle the problem. He bypassed the Congress (the U.S.
legislative houses). Thus CEA also DB tells us that he wasnt merely fighting the entire Congress. Hence CEADB.
72. (d) The link is EDB. This is present only in option (d) and hence that is the best choice.
73. (d) The para can begin with A or B. A is a better option being broader in concept with B using areas. C and D
successively supports B & C. D falls in line with one state and assessment.
74. (b) B clearly follows (1) with oil and water. A continues to explain the concept. C has to be preceded by (6) by
rays and colour. D has to settle in between, after A.
75. (b) C clearly follows (1) commenting on salvation. B tells about the fate of human society in the Christian tradition.
AD explains Buddhism as analytical dissolving individuals into component parts.

READING COMPREHENSION:
Passage I:
76. (c) Last paragraph, line 6, 7 and 9 support choice (c).
77. (b) Refer paragraph 3, lines 2 and 3, and paragraph 4, line 1.
78. (d) Paragraph 1, line 4 supports choice (d).
79. (b) Paragraph 1, line 2 and paragraph 4, line 1 indicates that choice (b) is correct.

Passage II:
80. (b) It is clear from the last paragraph in the last four lines that philosophy teaches us how to live in uncertainty
without being paralyzed by hesitation.
81. (b) Paragraph 3 clearly states that philosophy appeals to human reasons whether they are of tradition or that of
revelation like science.
82. (d) It is evident from the passage that author's profession can not be of a theologian because according to him the
definiteness of theologies presented by theologian cause modern minds to view them with suspicion. Theologizes
create a kind of insolence towards the universe.
83. (d) Options (a), (b) and (c) are nowhere mentioned in the passage.

Passage III:
84. (c) The passage clearly states that printed sheets of notated music might travel from the composer, it still remains his
property; the notion of property remains at heart of western conception of genius' which derives from latin gignere
or 'to beget'.
85. (a) The passage reveals that teaching of north Indian classical music is achieved by oral means. Saussures conception
of language as a communication between addresser and addressee is new, exotic complexity and glamourous.
86. (d) The passage stipulates the fact that middle class play a significant role in continuing ancient tradition. Moreover,
tape-recorded serves as 'Handy technological slave' and preserves from oblivion, vanishing, elusive moment of oral
transmission. The author's part of education was conducted via ugly but beneficial rectangle of plastic.

PRATHAM: BMS Special Booklet

104

ANSWERS & EXPLANATIONS


87. (c) The passage states that oral transmission of North Indian classical music remains a testament to the fact that the
human brains can absorb, remember and reproduce structures of great complexity and sophistication without the
help of hieroglyph or written mark of a system of notation.
88. (a) The passage itself explains that conductor itself is a custodian and guardian of his property. The Raga is no one's
property. It is not easy to pin down its source or to know exactly where it's provenance or origin lies.
89. (d) ; (a) The passage says that printed sheet of music might travel from composer but remains his property.
(c) Can be taken from passage as connection between composer and piece of music is unambigous because
composer writes down his composition as poets write their poems.
90. (d) The passage 6 illustrates that style of teaching followed in north Indian classical music has produced no
noteworthy student; the creative musicians still emerge from guru-shishya relationships, their understanding of musk
developed through communication.
91. (b) The passage states that this idea is reflected way strongly in the passage. In fact the last four paragraphs are
devoted completely in this theme.

Passage IV:
92. (c) It is clear from the passage.
93. (b) the paragraph gives the traditional method of bereavement handling in which farmers and town people mourne at
the death of mother, brother son and the bereaved is joined by neighbours and kin. They meet grief together in
lamentation, prayer and song. They also sourround themselves in community.
94. (c) The author says that tools of bereavement counselling will create a desert where community once flourished, the
bereavement counselor will see the impossibility of restoring hope in clients once they are alone with nothing but a
service for consulting. In failure of service bereavement, counselor will find desert in herself.
95. (d) According to the passage bereavement counseller is a person who meets the need to those experiencing the
death of loved ones and one who can process the grief of the people.
96. (a) According to the passage when the Europeans were puzzled by the environment also called it 'GREAT DESERT'. It
seemed untellable. The earth was wet and it was covered with centuries of tangled and matted grass.
97. (d) The passage clearly says that it took Europeans just one generation to make their homeland into a desert. The
sauk Indians banished themselves to another desert called reservation. The counsellor's new tool will cut through the
social fabric throwing aside kinship, care, neighbourly obligations and community ways of coming together and going
on. The tools will create desert where community once flourished.
98. (a) Breavement counsellor knows how to process grief the proper way. He can make people learn the correct form
of service to deal with guilt and grief. Moreover grieving daughter knows that it is breavement counsellor who comes
when death visits this family on the prairie of sauk.
99. (b) According to the passage 'Migration of communities is not mentioned anywhere in the passage whereas other
three parallels are mentioned at different points in the passage.

Passage V:
100. (d) The passage reveals that people are not capable of technical drafting skills and hence such artists create an art
form that anyone is capable of and less time consuming. Secondly, they feel that art deals with beauty. Moreover,
modern abstractionists compose their pieces irrationally.
101. (c) The passage explains that people feel more comfortable with something that they can relate to and understand
immediately without too much thought.
102. (b) This can be understood from the paragraph that if Guernica was a representational painting it would not have
been able to express emotions that well.
103. (c) Passage 3 explains that Mondrian was trying to present a system of simplicity, logic, and rational order. As a result
his pieces didn't end up looking like a scrabble board.

PRATHAM: BMS Special Booklet

105

104. (a) The passage says that both classes of painters have their ways of depicting 'reality'. To an representational artist
reality is what he sees with his eyes. To an abstract artist reality is what he feels about what his eyes see.

Passage VI:
105. (d) The passage clarifies that American army was designed to fight on the plains of Europe and thus lost the war
because it was not used to fight in Jungles while Vietnam was on their own land which they understood very well.
106. (b) The passage reveals all the ways of war except the option (b)
107. (a) The passage clearly states that ever since Sun-tzu, the Chinese have been seen as masters of subtlety who take
measure action to manipulate an adversary without his knowledge.
108. (b) The second last paragraph illustrates that Clausewitz view the way to achieve a larger political purpose is through
destruction of enemy's army whereas Sun-tzu believes that war proceeds along a fixed course and occupy a finite
extent of time like a lay in three acts with beginning. A middle and an end.
109. (a) The passage clarifies that to the Americans strategic mind, the Viet Cong guerilla did not fight fairly. They should
have come out in open and fought like men, instead of hiding in the jungle and sneaking around like a cat in the night.
110. (c) The passage states that Chinese intervention in Korea and jet offensive in Vietnam came out of tradition of
surprise and stealth. The exception that proves the point was the Soviet Unions invasion of Afghanistan in 1979.

Passage VII:
111. (c) This can be understood from the entire passage.
112. (b) The passage states that author thinks himself essential in relation to creation.
113. (d) The passage clarifies that there is difference between perception and creation as in perception object is essential
and subject is inessential whereas in creation subject is essential and object is inessential.
114. (a) The passage reveals the truth that reading is necessary and it lasts as long as this act can last. Beyond that there
are only black marks on paper, but the writer can't read what he had written.
115. (b) The passage itself reveals that a writer as an artist makes us feel essential vis-a-vis nature.

Passage VIII:
116. (a) The passage itself describes that writer's attitude towards the government is critical.
117. (c) The passage defines that government viewed that businessmen were little more than crooks who were to be
prevented from entering important areas of economy. The rates of tax, licensing of laws, reservation of whole
swathes of Industry for public sector, granting monopoly to public sector governed this attitude.
118. (c) The passage explains that changes that took place were forced by situations and partly by foreign exchange
bankruptcy of 1991 and the recognition that government would no longer muster the funds to support the public
sector.
119. (b) The author says that in another 50 years the World would have moved even further ahead.
120. (b) The passage explains that infrastructure in India is far better than most of developing countries have.
121. (d) The passage tells India didn't suffer during II World War. It was because it had advantages like an English speaking
elite, quality scientific manpower (including a Nobel laureate and other's who could be ranked among the world's
best) and excellent business accumen.
122. (a) The passage reveals that topmost reason for India's poor performance was economic isolationism. The
government discouraged imports and encouraged self-sufficiency. The result was creation of inefficient industry that
to keep pace with global trends and proved to be uncompetitive.
123. (d) It can be understood from passage that government discouraged imports and encouraged self-sufficiency.
124. (b) The passage states that symbols to show how far India have fallen back can be inferred as Korean Cellos are sold
in India, while no one in South Korea is rushing to buy an Indian car.

PRATHAM: BMS Special Booklet

106

ANSWERS & EXPLANATIONS


125. (a) The passage tells that Indian politicians are unable to see beyond their noses. Hence a no-exit policy for labour is
equivalent to no-entry policy for business.

Passage IX:
126. (b) The passage explains that how capitalism has led to disintegration of labour.
127. (b) The passage describes that author feels that Adam Smith boasted about something that was undesirable.
128. (d) The passage describes that there was a time when pinmakers would buy the material; shape it; make the head
and the point; ornament it; and take it to the market; sell it. They knew each and every process from beginning to
end but they couldn't afford to sell anybody a piper of pins for the farthing.
129. (b) It can be understood that pins are so cheap that if a child steals it, it would not be considered as stealing.
130. (a) The author is clearly against machines taking the place of men.
131. (c) The passage explains that Adam smith was the supporter of mass production.
132. (b) The passage explains that as people become richer they lose out on individual abilities.
133. (c) He is attaching the fact by making fun of it.
134. (d) None of the given statements continue with what the author has said in the last paragraph.

Passage X:
135. (b) The passage says that moral principles are said to share general transformation. As to the principles of ethics
there is no sign of revolution.
136. (c) The passage says that Darwin may need not change our ultimate moral ideas.
137. (c) The passage reveals that Chief Good refers to the welfare of community realised in its members.
138. (b) The passage states that Darwinism may force in some points a correction of our moral views and a return to
Non-Christian and perhaps a Hellenic Ideal. Moreover, passage says that Christian ideals are false.

PRATHAM: BMS Special Booklet

107

Das könnte Ihnen auch gefallen